The battle of yorktown

Lakukan tugas rumah & ujian kamu dengan baik sekarang menggunakan Quizwiz!

The nurse employed in a neonatal intensive care nursery receives a telephone call from the delivery room and is told that a newborn with spina bifida (myelomeningocele type) will be transported to the nursery. The maternity nurse prepares for the arrival of the newborn and places which priority item at the newborn's bedside? 1. A rectal thermometer 2. A blood pressure cuff 3. A specific gravity urinometer 4. A bottle of sterile normal saline

*4. A bottle of sterile normal saline*

The nurse is providing care to patient with the following laboratory values: pH - 7.31; PaCO2 - 48 mmHg; and a normal HCO3. Which condition should the nurse plan care for based on the current data? 1) Metabolic acidosis 2) Metabolic alkalosis 3) Respiratory acidosis 4) Respiratory alkalosis

3) Respiratory acidosis If the pH is decreased and the PaCO2 is increased with a normal HCO3, it is uncompensated respiratory acidosis

A nurse is preparing to start peripheral intravenous (IV) therapy. In which order will the nurse perform the steps starting with the first one? 1. Clean site. 2. Select vein. 3. Apply tourniquet. 4. Release tourniquet. 5. Reapply tourniquet. 6. Advance and secure. 7. Insert vascular access device.

3, 2, 4, 1, 5, 7, 6

A client with cardiovascular disease is scheduled to receive a daily dose of furosemide (Lasix). Which potassium level would cause the nurse, reviewing the client's electrolyte values, to contact the physician before administering the dose? A. 3.0 mEq/L B. 3.8 mEq/L C. 4.2 mEq/L D. 5.1 mEq/L

A. 3.0 mEq/L Rationale: The normal serum potassium level in the adult is 3.5 to 5.1 mEq/L. A result of 3.0 mEq/L is low, 3.8 and 4.2 mEq/L are normal, and 5.1 mEq/L is high. Administering furosemide to a client with a low potassium level and a history of cardiovascular disease could precipitate ventricular dysrhythmias in the client. The normal and high levels do not require withholding of the dose. In fact, the high level may be lowered by administration of the medication, which is a potassium-losing diuretic.

The nurse and a student nurse are discussing the effects of bed immobility on patients. The nurse knows that the student nurse understands the concept of mobility when she states, "Patients with impaired bed mobility a.have an increased risk for pressure ulcers." b.like to have extra visitors." c.need to have a mechanical soft diet." d.are prone to constipation."

ANS : A

Patient with chronic renal failure who will require dialysis three times a week for the rest of his life says to the nurse, "Why should I even bother to watch what I eat and drink? It doesn't really matter what I do if I'm never going to get better!" On the basis of the client's statement, the nurse determines that the client is experiencing which problem? A. Anxiety B. Ineffective coping C. Powerlessness D. Disturbed body image

ANS : C

The nurse is talking to the unlicensed assistive personnel about moving a patient in bed. The nurse knows the unlicensed assistive personnel understands the concept of mobility and proper moving techniques when he or she states, "Patients must a.have a trapeze over the bed to move properly." b.move themselves in bed to prevent immobility." c.always have a two-person assist to move in bed." d.be moved correctly in bed to prevent shearing."

ANS : D

A client with a history of lung disease is at risk for respiratory acidosis. For which of the following signs and symptoms does the nurse assess this client? A. Disorientation and dyspnea B. Drowsiness, headache, and tachypnea C. Tachypnea, dizziness, and paresthesias D. Dysrhythmias and decreased respiratory rate and depth

ANS :A. The client with respiratory acidosis would exhibit the symptoms identified in the correct option. The client will experience dyspnea and may be disoriented as a result of hypoxia and retention of carbon dioxide. Metabolic acidosis and alkalosis are marked by drowsiness, headache, and tachypnea and dysrhythmias and decreased respiratory rate and depth, respectively. The client with respiratory alkalosis is likely to exhibit tachypnea, dizziness, and paresthesias of the extremities.

A patient who has been in the hospital for several weeks is about to be discharged. The patient is weak from the hospitalization and asks the nurse to explain why this is happening. The nurse's best response is "You are weak because a.your iron level is low. This is known as anemia." b.of your immobility in the hospital. This is known as deconditioning." c.of your poor appetite. This is known as malnutrition." d.of your medications. This is known as drug induced weakness."

ANS :B

A client tells the nurse that he has been experiencing frequent heartburn and has been "living on antacids." For which acid-base disturbance does the nurse recognize a risk? A. Metabolic acidosis B. Metabolic alkalosis C. Respiratory acidosis D. Respiratory alkalosis

ANS :B Oral antacids commonly contain sodium or calcium bicarbonate or other alkaline components. These substances bind to the hydrochloric acid in the stomach to neutralize it. Excessive use of oral antacids containing sodium or calcium bicarbonate can cause metabolic alkalosis over time.

A patient who is suspected of having an epidural hematoma is admitted to the emergency department. Which action will the nurse expect to take? a. Administer IV furosemide (Lasix). b. Prepare the patient for craniotomy. c. Initiate high-dose barbiturate therapy. D. Type and cross match for blood transfusion.

ANS :B The patient will need a craniotomy to release some of the pressure that is coming from the hematoma

A nurse is teaching a group of business people about disease transmission. He knows that he needs to re-educate when one of the participants states which of the following? A. "When traveling outside of the country, I need to be sure that I receive appropriate vaccinations." B. "Food and water supplies in foreign countries can contain microorganisms to which my body is not accustomed and has no resistance." C. "If I don't feel sick, then I don't have to worry about transmitted diseases." D. "I need to be sure to have good hygiene practices when traveling in crowded planes and trains."

ANS :C

In order to provide the best intervention for a patient, the nurse is often responsible for obtaining a sample of exudate for culture. What information will this provide? A. What city/state the infection was contracted from. B. Where an infection is located. C. What cells are being utilized by the body to attack an infection. D. What specific type of pathogen is causing an infection.

ANS :D

Mobility for the patient changes throughout the life span; this is known as the process of a.aging and illness. b.illness and disease. c.health and wellness. d.growth and development.

ANS :D

A 38-yr-old patient who has had a spinal cord injury returned home following a stay in a rehabilitation facility. The home care nurse notes the spouse is performing many of the activities that the patient had been managing unassisted during rehabilitation. The appropriate nursing action at this phase of rehabilitation is to A.remind the patient about the importance of preforming ADL B.tell the spouse to stop helping because the patient is able to do them C.recognize that it is important for the spouse to be involved in the patients care and encourage participation D. develop a plan to increase the patient's independence in consultation with the patient and the spouse.

ANS :D You want to get the patient back into preforming the ADLs independently but its also very important to use therapeutic communication with both the patient and the spouse. Including both of them in the plan will make sure that everyone feels involved.

A patient who complains of chest pain is hospitalized with hyperkalemia. Which information will the nurse monitor to evaluate the effectiveness of the prescribed calcium gluconate IV? A. Cardiac rhythm B. Neurological status C. Calcium levels D. Urine volume

ANS A

A home health registered nurse is planning care for patient with autonomic dysreflexia related to a recent spinal injury Which nursing action can be delegated to license practical nurse (LPN) A. Assess for use of medications that may precipitate episode B. Place medications in the home medications organizer C. Make referrals to appropriate community agencies D. Teach the patient and family how to manage autonomic dysreflexia

ANS B This is the only action that is within the LPN scope of practice

which of these nursing actions for a patient with a spinal cord injury is appropriate for the nurse to delegate to the UAP A. Assessment of bladder distendtion B. NG tube feeding every 4 hours C. Passive range of motion every 4 hours D artificial tears administer every 2 hours

ANS C This is the least intrusive task and a UAP is not able to preform skilled task, assessments, or administer and form of medication.

Sodium polystyrene sulfonate (Kayexalate) is ordered for a patient with hyperkalemia. Before administering the medication, the nurse should assess the A.BUN B. Blood glucose C. Level of consciousness D. Bowel sounds

ANS D

the nurse is caring for a client who is undergoing peritoneal dialysis the nurse notes that the outflow is less than the inflow on the first exchange what should the nurse do first A. Hang the second exchange and continue to monitor B. Reposition the patient C. Irrigate the catheter D. Check the system for kinks

ANS D The first intervention should be to check for kinks and obstructions because that could be preventing drainage. After checking for kinks, have the client change position to promote drainage. Don't give the next scheduled exchange until the dialysate is drained because abdominal distention will occur, unless the output is within parameters set by the physician. If unable to get more output despite checking for kinks and changing the client's position, the nurse should then call the physician to determine the proper intervention.

The nurse has taught family member of a client undergoing the peritoneal dialysis method to deal with problems of inflow of the dialysate the nurse realizes that the family member understood the provided instructions if the family member mentions taking which action first if there is difficulty with installation of the dialysate? A. Reposition the client B. Milk the dialysis tube C. Ask the client about recent problems with constipation D. Place the client in supine Fowler position

ANS: A

The nurse should assess the client with severe diarrhea for which acid-base imbalance? a) Metabolic acidosis. b) Metabolic alkalosis. c) Respiratory alkalosis. d) Respiratory acidosis.

ANS: A A client with severe diarrhea loses large amounts of bicarbonate, resulting in metabolic acidosis. Metabolic alkalosis does not result in this situation. Diarrhea does not affect the respiratory system.

A nurse is studying the results of periodic serum laboratory studies in a client with diabetic ketoacidosis (DKA) who is receiving an intravenous insulin infusion. Which finding should prompt the nurse to contact the health care provider? A. Potassium 3.1 mEq/L (3.1 mmol/L) B. Serum pH 7.33 (7.33) C. Blood glucose 290 mg/dL (16.15 mmol/L) D. Sodium 137 mEq/L (137 mmol/L)

ANS: A Rationale: The client with DKA initially becomes hyperkalemic as potassium leaves the cells in response to a lowered pH. Once fluid replacement and insulin therapy are started, the potassium level drops quickly because potassium is carried into the cells along with glucose and insulin and because potassium is excreted in the urine once rehydration has occurred. Therefore the nurse must carefully monitor the client's serum potassium results and report hypokalemia (i.e., potassium 3.1 mEq/L, 3.1 mmol/L) immediately. A blood glucose reading of 290 mg/dL (16.2 mmol/L)is high and is the reason that the client is receiving the insulin infusion. Normally the blood glucose level is higher than 300 mg/dL (16.7 mmol/L)in DKA, so a value of 290 mg/dL (16.2 mmol/L)indicates improvement in the client's condition. A serum pH of 7.33 is slightly low, reflecting the metabolic acidosis that accompanies DKA. A sodium value of 137 mEq/L (137 mmol/L)is within the normal range; serum sodium values in DKA fluctuate and may be low, normal, or high. Test-Taking Strategy: Use the process of elimination and recall the normal reference ranges for the tests listed in the options. Eliminate the sodium level, because it is a normal value. To select from the remaining options, recall that an increased blood glucose level and low pH are expected in DKA. Review care of the client with DKA if you had difficulty with this question.

Which action can the registered nurse (RN) who is caring for a critically ill patient experiencing acute kidney injury with multiple IV lines and medications delegate to a licensed practical/vocational nurse (LPN/LVN)? a. Flush a saline lock with normal saline. b. Verify blood products prior to administration. c. Remove the patient's central venous catheter. d. Titrate the flow rate of vasoactive IV medications.

ANS: A A LPN/LVN has the education, experience, and scope of practice to flush a saline lock with normal saline. Administration of blood products, adjustment of vasoactive infusion rates, and removal of central catheters in critically ill patients require RN level education and scope of practice.

Which action can the registered nurse (RN) who is caring for a critically ill patient with multiple IV lines delegate to an experienced licensed practical/vocational nurse (LPN/LVN)? a. Flush a saline lock with normal saline. b. Verify blood products prior to administration. c. Remove the patient's central venous catheter. d. Titrate the flow rate of vasoactive IV medications.

ANS: A A LPN/LVN has the education, experience, and scope of practice to flush a saline lock with normal saline. Administration of blood products, adjustment of vasoactive infusion rates, and removal of central catheters in critically ill patients require RN level education and scope of practice. DIF: Cognitive Level: Apply (application) REF: 294 OBJ: Special Questions: Delegation TOP: Nursing Process: Planning MSC: NCLEX: Safe and Effective Care Environment

Which assessment finding would be the earliest and most sensitive indicator that there is an alteration in intracranial regulation? a. Change in level of consciousness b. Inability to focus visually c. Loss of primitive reflexes d. Unequal pupil size

ANS: A A change in level of consciousness is the earliest and most sensitive indication of a change in intracranial processing. This is assessed with the Glasgow Coma Scale (GCS), which assesses eye opening and verbal and motor response. The inability to focus may indicate a change, but it is not one of the earliest indicators or a component of the GCS. Primitive reflexes refer to those reflexes found in a normal infant that disappear with maturation. These reflexes may reappear with frontal lobe dysfunction and may be tested for with a suspected brain injury, so it would be the reappearance of primitive reflexes. A change in pupil size or unequal pupils may indicate a change, but they are not one of the earliest indicators or a component of the GCS.

A nurse is caring for a patient with peripheral intravenous (IV) therapy. Which task will the nurse assign to the nursing assistive personnel? a. Recording intake and output b. Regulating intravenous flow rate c. Starting peripheral intravenous therapy d. Changing a peripheral intravenous dressing

ANS: A A nursing assistive personnel (NAP) can record intake and output. An RN cannot delegate regulating flow rate, starting an IV, or changing an IV dressing to an NAP.

The nurse is admitting an older adult with decompensated congestive heart failure. The nursing assessment reveals adventitious lung sounds, dyspnea, and orthopnea. The nurse should question which doctor's order? a. Intravenous (IV) 500 mL of 0.9% NaCl at 125 mL/hr b. Furosemide (Lasix) 20 mg PO now c. Oxygen via face mask at 8 L/min d. KCl 20 mEq PO two times per day

ANS: A A patient with decompensated heart failure has extracellular fluid volume (ECV) excess. The IV of 0.9% NaCl is normal saline, which should be questioned because it would expand ECV and place an additional load on the failing heart. Diuretics such as furosemide are appropriate to decrease the ECV during heart failure. Increasing the potassium intake with KCl is appropriate, because furosemide increases potassium excretion. Oxygen administration is appropriate in this situation of near pulmonary edema from ECV excess.

A patient arrives in the emergency department with facial and chest burns caused by a house fire. Which action should the nurse take first? a. Auscultate the patient's lung sounds. b. Determine the extent and depth of the burns. c. Infuse the ordered lactated Ringer's solution. d. Administer the ordered hydromorphone (Dilaudid).

ANS: A A patient with facial and chest burns is at risk for inhalation injury, and assessment of airway and breathing is the priority. The other actions will be completed after airway management is assured.

A 68-yr-old male patient is brought to the emergency department (ED) by ambulance after being found unconscious on the bathroom floor by his spouse. Which action will the nurse take first? a. Check oxygen saturation. b. Assess pupil reaction to light. c. Palpate the head for injuries d. Verify Glasgow Coma Scale (GCS) score.

ANS: A Airway patency and breathing are the most vital functions and should be assessed first. The neurologic assessments should be accomplished next and additional assessment after that. DIF: Cognitive Level: Analyze (analysis)

Which patient is most appropriate for the burn unit charge nurse to assign to a registered nurse (RN) who has floated from the hospital medical unit? a. A 34-year-old patient who has a weight loss of 15% from admission and requires enteral feedings. b. A 67-year-old patient who has blebs under an autograft on the thigh and has an order for bleb aspiration c. A 46-year-old patient who has just come back to the unit after having a cultured epithelial autograft to the chest d. A 65-year-old patient who has twice-daily burn debridements and dressing changes to partial-thickness facial burns

ANS: A An RN from a medical unit would be familiar with malnutrition and with administration and evaluation of response to enteral feedings. The other patients require burn assessment and care that is more appropriate for staff who regularly care for burned patients

Which patient is most appropriate for the intensive care unit (ICU) charge nurse to assign to a registered nurse (RN) who has floated from the medical unit? a. A 45-yr-old patient receiving IV antibiotics for meningococcal meningitis b. A 35-yr-old patient with intracranial pressure (ICP) monitoring after a head injury c. A 25-yr-old patient admitted with a skull fracture and craniotomy the previous day d. A 55-yr-old patient who has increased intracranial pressure (ICP) and is receiving hyperventilation therapy

ANS: A An RN who works on a medical unit will be familiar with administration of IV antibiotics and with meningitis. The patient recovering from a craniotomy, the patient with an ICP monitor, and the patient on a ventilator should be assigned to an RN familiar with the care of critically ill patients. DIF: Cognitive Level: Analyze (analysis)

What is the nurse's first action when planning to teach the parents of an infant with a congenital heart defect (CHD)? a. Assess the parents' anxiety level and readiness to learn. b. Gather literature for the parents. c. Secure a quiet place for teaching. d. Discuss the plan with the nursing team.

ANS: A Any effort to organize the right environment, plan, or literature is of no use if the parents are not ready to learn or have high anxiety. Decreasing their level of anxiety is often needed before new information can be processed. A baseline assessment of prior knowledge should be taken into consideration before developing any teaching plan. Locating a quiet place for meeting with parents is appropriate; however, an assessment should be done before any teaching is done. Discussing a teaching plan with the nursing team is appropriate after an assessment of the parents' knowledge and readiness.

A patient will need vascular access for hemodialysis. Which statement by the nurse accurately describes an advantage of a fistula over a graft? a. A fistula is much less likely to clot. b. A fistula increases patient mobility. c. A fistula can accommodate larger needles. d. A fistula can be used sooner after surgery.

ANS: A Arteriovenous (AV) fistulas are much less likely to clot than grafts, although it takes longer for them to mature to the point where they can be used for dialysis. The choice of an AV fistula or a graft does not have an impact on needle size or patient mobility. DIF: Cognitive Level: Understand (comprehension) REF: 1088 TOP: Nursing Process: Implementation MSC: NCLEX: Physiological Integrity

A patient has a tumor in the cerebellum. The nurse will plan interventions to a. prevent falls. . b. stabilize mood. c. avoid aspiration d. improve memory.

ANS: A Because functions of the cerebellum include coordination and balance, the patient with dysfunction is at risk for falls. The cerebellum does not affect memory, mood, or swallowing ability.

A patient with septic shock has a BP of 70/46 mm Hg, pulse 136, respirations 32, temperature 104° F, and blood glucose 246 mg/dL. Which intervention ordered by the health care provider should the nurse implement first? a. Give normal saline IV at 500 mL/hr. b. Give acetaminophen (Tylenol) 650 mg rectally. c. Start insulin drip to maintain blood glucose at 110 to 150 mg/dL. d. Start norepinephrine (Levophed) to keep systolic blood pressure >90 mm Hg.

ANS: A Because of the low systemic vascular resistance (SVR) associated with septic shock, fluid resuscitation is the initial therapy. The other actions also are appropriate, and should be initiated quickly as well.

A patient is admitted to the emergency department with possible renal trauma after an automobile accident. Which prescribed intervention will the nurse implement first? a. Check blood pressure and heart rate. b. Administer morphine sulfate 4 mg IV. c. Transport to radiology for an intravenous pyelogram. d. Insert a urethral catheter and obtain a urine specimen.

ANS: A Because the kidney is very vascular, the initial action with renal trauma will be assessment for bleeding and shock. The other actions are also important once the patient's cardiovascular status has been determined and stabilized.

The patient has an intravenous (IV) line and the nurse needs to remove the gown. In which order will the nurse perform the steps, starting with the first one? 1. Remove the sleeve of the gown from the arm without the IV. 2. Remove the sleeve of the gown from the arm with the IV. 3. Remove the IV solution container from its stand. 4. Pass the IV bag and tubing through the sleeve. a. 1, 2, 3, 4 b. 2, 3, 4, 1 c. 3, 4, 1, 2 d. 4, 1, 2, 3

ANS: A Change regular gowns by following these steps for maximum speed and arm mobility: (1) To remove a gown, remove the sleeve of the gown from the arm without the IV line, maintaining the patient's privacy. (2) Remove the sleeve of the gown from the arm with the IV line. (3) Remove the IV solution container from its stand, and pass it and the tubing through the sleeve. (If this involves removing the tubing from an EID, use the roller clamp to slow the infusion to prevent the accidental infusion of a large volume of solution or medication.)

A female patient who had a stroke 24 hours ago has expressive aphasia. An appropriate nursing intervention to help the patient communicate is to a. ask questions that the patient can answer with "yes" or "no." b. develop a list of words that the patient can read and practice reciting. c. have the patient practice her facial and tongue exercises with a mirror. d. prevent embarrassing the patient by answering for her if she does not respond.

ANS: A Communication will be facilitated and less frustrating to the patient when questions that require a "yes" or "no" response are used. When the language areas of the brain are injured, the patient might not be able to read or recite words, which will frustrate the patient without improving communication. Expressive aphasia is caused by damage to the language areas of the brain, not by the areas that control the motor aspects of speech. The nurse should allow time for the patient to respond.

A college athlete is seen in the clinic 6 weeks after a concussion. Which assessment information will the nurse collect to determine whether the patient is developing postconcussion syndrome? a. Short-term memory b. Muscle coordination c. Glasgow Coma Scale d. Pupil reaction to light

ANS: A Decreased short-term memory is one indication of postconcussion syndrome. The other data may be assessed but are not indications of postconcussion syndrome.

A nurse is caring for a patient who is orally intubated and receiving mechanical ventilation. To decrease the risk for ventilator-associated pneumonia, which action will the nurse include in the plan of care? a. Elevate head of bed to 30 to 45 degrees. b. Give enteral feedings at no more than 10 mL/hr. c. Suction the endotracheal tube every 2 to 4 hours. d. Limit the use of positive end-expiratory pressure.

ANS: A Elevation of the head decreases the risk for aspiration. Positive end-expiratory pressure is frequently needed to improve oxygenation in patients receiving mechanical ventilation. Suctioning should be done only when the patient assessment indicates that it is necessary. Enteral feedings should provide adequate calories for the patient's high energy needs.

A patient has just been admitted with a 40% total body surface area (TBSA) burn injury. To maintain adequate nutrition, the nurse should plan to take which action? a. Insert a feeding tube and initiate enteral feedings. b. Infuse total parenteral nutrition via a central catheter. c. Encourage an oral intake of at least 5000 kcal per day. d. Administer multiple vitamins and minerals in the IV solution.

ANS: A Enteral feedings can usually be initiated during the emergent phase at low rates and increased over 24 to 48 hours to the goal rate. During the emergent phase, the patient will be unable to eat enough calories to meet nutritional needs and may have a paralytic ileus that prevents adequate nutrient absorption. Vitamins and minerals may be administered during the emergent phase, but these will not assist in meeting the patient's caloric needs. Parenteral nutrition increases the infection risk, does not help preserve gastrointestinal function, and is not routinely used in burn patients.

Which intervention should be included in the plan of care for an infant with the nursing diagnosis of Excess Fluid Volume related to congestive heart failure? a. Weigh the infant every day on the same scale at the same time. b. Notify the physician when weight gain exceeds more than 20 g/day. c. Put the infant in a car seat to minimize movement. d. Administer digoxin (Lanoxin) as ordered by the physician.

ANS: A Excess fluid volume may not be overtly visible. Weight changes may indicate fluid retention. Weighing the infant on the same scale at the same time each day ensures consistency. An excessive weight gain for an infant is an increase of more than 50 g/day. With fluid volume excess, skin will be edematous. The infant's position should be changed frequently to prevent undesirable pooling of fluid in certain areas. Lanoxin is used in the treatment of congestive heart failure to improve cardiac function. Diuretics will help the body get rid of excess fluid.

A patient in the outpatient clinic is diagnosed with acute hepatitis C (HCV) infection. Which action by the nurse is appropriate? a. Schedule the patient for HCV genotype testing. b. Administer the HCV vaccine and immune globulin. c. Teach the patient about ribavirin (Rebetol) treatment. d. Explain that the infection will resolve over a few months.

ANS: A Genotyping of HCV has an important role in managing treatment and is done before drug therapy is initiated. Because most patients with acute HCV infection convert to the chronic state, the nurse should not teach the patient that the HCV will resolve in a few months. Immune globulin or vaccine is not available for HCV. Ribavirin is used for chronic HCV infection.

A patient with acute respiratory distress syndrome (ARDS) and acute kidney injury has the following drugs ordered. Which drug should the nurse discuss with the health care provider before giving? a. gentamicin 60 mg IV b. pantoprazole (Protonix) 40 mg IV c. sucralfate (Carafate) 1 g per nasogastric tube d. methylprednisolone (Solu-Medrol) 60 mg IV

ANS: A Gentamicin, which is one of the aminoglycoside antibiotics, is potentially nephrotoxic, and the nurse should clarify the drug and dosage with the health care provider before administration. The other drugs are appropriate for the patient with ARDS.

Which problem is most often associated with myelomeningocele? a. Hydrocephalus b. Craniosynostosis c. Biliary atresia d. Esophageal atresia

ANS: A Hydrocephalus is an associated anomaly in 80% to 90% of children. Craniosynostosis is the premature closing of the cranial sutures and is not associated with myelomeningocele. Biliary and esophageal atresias are not associated with myelomeningocele.

When caring for a patient after a head injury, the nurse would be most concerned with assessment findings which included respiratory changes along with what other findings? a. Hypertension and bradycardia b. Hypertension and tachycardia c. Hypotension and bradycardia d. Hypotension and tachycardia

ANS: A Hypertension with widening pulse pressure, bradycardia, and respiratory changes are the ominous late signs of increased intracranial pressure and indications of impending herniation (Cushing's triad). It is bradycardia, not tachycardia, which is the component of this ominous triad. It is hypertension, not hypotension, which is the component of this ominous triad.

A patient receives 3% NaCl solution for correction of hyponatremia. Which assessment is most important for the nurse to monitor for while the patient is receiving this infusion? a. Lung sounds b. Urinary output c. Peripheral pulses d. Peripheral edema

ANS: A Hypertonic solutions cause water retention, so the patient should be monitored for symptoms of fluid excess. Crackles in the lungs may indicate the onset of pulmonary edema and are a serious manifestation of fluid excess. Bounding peripheral pulses, peripheral edema, or changes in urine output are also important to monitor when administering hypertonic solutions, but they do not indicate acute respiratory or cardiac decompensation

The nurse administers an intravenous (IV) hypertonic solution to a patient. In which direction will the fluid shift? a. From intracellular to extracellular b. From extracellular to intracellular c. From intravascular to intracellular d. From intravascular to interstitial

ANS: A Hypertonic solutions will move fluid from the intracellular to the extracellular (intravascular). A hypertonic solution has a concentration greater than normal body fluids, so water will shift out of cells because of the osmotic pull of the extra particles. Movement of water from the extracellular (intravascular) into cells (intracellular) occurs when hypotonic fluids are administered. Distribution of fluid between intravascular and interstitial spaces occurs by filtration, the net sum of hydrostatic and osmotic pressures.

A patient with renal failure has a serum calcium level of 7.0 mEq/L. Which assessment finding is most important for the nurse to report to the health care provider? a. The patient is experiencing laryngeal stridor. b. The patient complains of generalized fatigue. c. The patient's bowels have not moved for 4 days. d. The patient has numbness and tingling of the lips.

ANS: A Hypocalcemia can cause laryngeal stridor, which may lead to respiratory arrest. Rapid action is required to correct the patient's calcium level. The other data are also consistent with hypocalcemia, but do not indicate a need for as immediate action as laryngospasm

A patient who has been receiving diuretic therapy is admitted to the emergency department with a serum potassium level of 3.0 mEq/L. The nurse should alert the health care provider immediately that the patient is on which medication? a. Oral digoxin (Lanoxin) 0.25 mg daily b. Ibuprofen (Motrin) 400 mg every 6 hours c. Metoprolol (Lopressor) 12.5 mg orally daily d. Lantus insulin 24 U subcutaneously every evening

ANS: A Hypokalemia increases the risk for digoxin toxicity, which can cause serious dysrhythmias. The nurse will also need to do more assessment regarding the other medications, but they are not of as much concern with the potassium level

A patient who has been receiving diuretic therapy is admitted to the emergency department with a serum potassium level of 3.0 mEq/L. The nurse should alert the health care provider immediately that the patient is on which medication? a. Oral digoxin (Lanoxin) 0.25 mg daily b. Ibuprofen (Motrin) 400 mg every 6 hours c. Metoprolol (Lopressor) 12.5 mg orally daily d. Lantus insulin 24 U subcutaneously every evening

ANS: A Hypokalemia increases the risk for digoxin toxicity, which can cause serious dysrhythmias. The nurse will also need to do more assessment regarding the other medications, but they are not of as much concern with the potassium level

A 20-yr-old patient has a mandatory electrocardiogram (ECG) before participating on a college soccer team and is found to have sinus bradycardia, rate 52. Blood pressure (BP) is 114/54 mm Hg, and the student denies any health problems. What action by the nurse is most appropriate? a. Allow the student to participate on the soccer team. b. Refer the student to a cardiologist for further testing. c. Tell the student to stop playing immediately if any dyspnea occurs. d. Obtain more detailed information about the student's family health history.

ANS: A In an aerobically trained individual, sinus bradycardia is normal. The student's normal BP and negative health history indicate that there is no need for a cardiology referral or for more detailed information about the family's health history. Dyspnea during an aerobic activity such as soccer is normal. DIF: Cognitive Level: Apply (application)

When admitting a patient with possible respiratory failure and a high PaCO2, which assessment information should be immediately reported to the health care provider? a. The patient is very somnolent. b. The patient complains of weakness. c. The patient's blood pressure is 164/98. d. The patient's oxygen saturation is 90%.

ANS: A Increasing somnolence will decrease the patient's respiratory rate and further increase the PaCO2 and respiratory failure. Rapid action is needed to prevent respiratory arrest. An SpO2 of 90%, weakness, and elevated blood pressure all require ongoing monitoring but are not indicators of possible impending respiratory arrest.

A patient has dehydration. While planning care, the nurse considers that the majority of the patient's total water volume exists in with compartment? A. Intracellular B. Extracellular C. Intravascular D. Transcellular

ANS: A Intracellular (inside the cells) fluid accounts for approximately two thirds of total body water. Extracellular (outside the cells) is approximately one third of the total body water. Intravascular fluid (liquid portion of the blood) and transcellular fluid are two major divisions of the extracellular compartment.

The nurse is making a home visit to a child who has a chronic disease. Which finding has the greatest implication for acid-base aspects of this patient's care? a. Urine output is very small today. b. Whites of the eyes appear more yellow. c. Skin around the mouth is very chapped. d. Skin is sweaty under three blankets.

ANS: A Oliguria decreases the excretion of metabolic acids and is a risk factor for metabolic acidosis. Jaundice requires follow-up but is not an acid-base problem. Perioral chapped skin needs intervention but is not an acid-base issue. With three blankets, diaphoresis is not unusual.

A patient experiences a chest wall contusion as a result of being struck in the chest with a baseball bat. The emergency department nurse would be most concerned if which finding is observed during the initial assessment? a. Paradoxic chest movement b. Complaint of chest wall pain c. Heart rate of 110 beats/minute d. Large bruised area on the chest

ANS: A Paradoxic chest movement indicates that the patient may have flail chest, which can severely compromise gas exchange and can rapidly lead to hypoxemia. Chest wall pain, a slightly elevated pulse rate, and chest bruising all require further assessment or intervention, but the priority concern is poor gas exchange

Following a cauda equina spinal cord injury, which action will the nurse include in the plan of care? a. Catheterize patient every 3 to 4 hours. b. Assist patient to ambulate several times daily. c. Administer medications to reduce bladder spasm. d. Stabilize the neck when repositioning the patient.

ANS: A Patients with cauda equina syndrome have areflexic bladder, and intermittent catheterization will be used for emptying the bladder. Because the bladder is flaccid, antispasmodic medications will not be used. The legs are flaccid with cauda equina syndrome and the patient will be unable to ambulate. The head and neck will not need to be stabilized following a cauda equina injury, which affects the lumbar and sacral nerve roots.

A patient has been taking phenytoin (Dilantin) for 2 years. Which action will the nurse take when evaluating for adverse effects of the medication? a. Inspect the oral mucosa. b. Listen to the lung sounds. c. Auscultate the bowel tones. d. Check pupil reaction to light.

ANS: A Phenytoin can cause gingival hyperplasia, but does not affect bowel tones, lung sounds, or pupil reaction to light. DIF: Cognitive Level: Apply (application) REF: 1424 TOP: Nursing Process: Evaluation MSC: NCLEX: Physiological Integrity

The acid-base status of a patient is dependent on normal gas exchange. Which patient would the nurse identify as having an increased risk for the development of respiratory acidosis? A patient with a. chronic lung disease with increased carbon dioxide retention b. acute anxiety, hyperventilation, and decreased carbon dioxide retention c. decreased cardiac output with increased serum lactic acid production d. gastric drainage with increased removal of gastric acid

ANS: A Respiratory acidosis is caused by an increase in retention of carbon dioxide, regardless of the underlying disease. A decrease in carbon dioxide retention may lead to respiratory alkalosis. An increase in production of lactic acid leads to metabolic acidosis. Removal of an acid (gastric secretions) will lead to a metabolic alkalosis.

The nurse is planning care for a patient with acute severe pancreatitis. The highest priority patient outcome is a. maintaining normal respiratory function. b. expressing satisfaction with pain control. c. developing no ongoing pancreatic disease. d. having adequate fluid and electrolyte balance.

ANS: A Respiratory failure can occur as a complication of acute pancreatitis and maintenance of adequate respiratory function is the priority goal. The other outcomes would also be appropriate for the patient.

Sodium polystyrene sulfonate (Kayexalate) is ordered for a patient with hyperkalemia. Before administering the medication, the nurse should assess the a. bowel sounds. b. blood glucose. c. blood urea nitrogen (BUN). d. level of consciousness (LOC).

ANS: A Sodium polystyrene sulfonate (Kayexalate) should not be given to a patient with a paralytic ileus (as indicated by absent bowel sounds) because bowel necrosis can occur. The BUN and creatinine, blood glucose, and LOC would not affect the nurse's decision to give the medication. DIF: Cognitive Level: Apply (application) REF: 1080 TOP: Nursing Process: Assessment MSC: NCLEX: Physiological Integrity

The charge nurse observes the following actions being taken by a new nurse on the burn unit. Which action by the new nurse would require an intervention by the charge nurse? a. The new nurse uses clean latex gloves when applying antibacterial cream to a burn wound. b. The new nurse obtains burn cultures when the patient has a temperature of 95.2° F (35.1° C). c. The new nurse administers PRN fentanyl (Sublimaze) IV to a patient 5 minutes before a dressing change. d. The new nurse calls the health care provider for a possible insulin order when a nondiabetic patient's serum glucose is elevated.

ANS: A Sterile gloves should be worn when applying medications or dressings to a burn. Hypothermia is an indicator of possible sepsis, and cultures are appropriate. Nondiabetic patients may require insulin because stress and high calorie intake may lead to temporary hyperglycemia. Fentanyl peaks 5 minutes after IV administration, and should be used just before and during dressing changes for pain management

A nurse is caring for a patient with acute respiratory distress syndrome (ARDS) who is receiving mechanical ventilation using synchronized intermittent mandatory ventilation (SIMV). The settings include fraction of inspired oxygen (FIO2) of 80%, tidal volume of 450, rate of 16/minute, and positive end-expiratory pressure (PEEP) of 5 cm. Which assessment finding is most important for the nurse to report to the health care provider? a. O2 saturation of 99% b. Heart rate 106 beats/minute c. Crackles audible at lung bases d. Respiratory rate 22 breaths/minute

ANS: A The FIO2 of 80% increases the risk for O2 toxicity. Because the patient's O2 saturation is 99%, a decrease in FIO2 is indicated to avoid toxicity. The other patient data would be typical for a patient with ARDS and would not be the most important data to report to the health care provider. DIF: Cognitive Level: Analyze (analysis)

The nurse is caring for a patient who arrived in the emergency department with acute respiratory distress. Which assessment finding by the nurse requires the most rapid action? a. The patient's PaO2 is 45 mm Hg. b. The patient's PaCO2 is 33 mm Hg. c. The patient's respirations are shallow. d. The patient's respiratory rate is 32 breaths/min.

ANS: A The PaO2 indicates severe hypoxemia and respiratory failure. Rapid action is needed to prevent further deterioration of the patient. Although the shallow breathing, rapid respiratory rate, and low PaCO2 also need to be addressed, the most urgent problem is the patient's poor oxygenation.

Which assessment finding obtained by the nurse when caring for a patient with a right radial arterial line indicates a need for the nurse to take action? a. The right hand feels cooler than the left hand. b. The mean arterial pressure (MAP) is 77 mm Hg. c. The system is delivering 3 mL of flush solution per hour. d. The flush bag and tubing were last changed 2 days previously.

ANS: A The change in temperature of the right hand suggests that blood flow to the right hand is impaired. The flush system needs to be changed every 96 hours. A mean arterial pressure (MAP) of 75 mm Hg is normal. Flush systems for hemodynamic monitoring are set up to deliver 3 to 6 mL/hr of flush solution. DIF: Cognitive Level: Apply (application)

During change-of-shift report, the nurse is told that a patient has been admitted with dehydration and hypotension after having vomiting and diarrhea for 4 days. Which finding is most important for the nurse to report to the health care provider? a. New onset of confusion b. Heart rate 112 beats/minute c. Decreased bowel sounds d. Pale, cool, and dry extremities

ANS: A The changes in mental status are indicative that the patient is in the progressive stage of shock and that rapid intervention is needed to prevent further deterioration. The other information is consistent with compensatory shock

A patient with circumferential burns of both legs develops a decrease in dorsalis pedis pulse strength and numbness in the toes. Which action should the nurse take? a. Notify the health care provider. b. Monitor the pulses every 2 hours. c. Elevate both legs above heart level with pillows. d. Encourage the patient to flex and extend the toes on both feet.

ANS: A The decrease in pulse in a patient with circumferential burns indicates decreased circulation to the legs and the need for an escharotomy. Monitoring the pulses is not an adequate response to the decrease in circulation. Elevating the legs or increasing toe movement will not improve the patient's circulation

Which data collected by the nurse caring for a patient who has cardiogenic shock indicate that the patient may be developing multiple organ dysfunction syndrome (MODS)? a. The patient's serum creatinine level is elevated. b. The patient complains of intermittent chest pressure. c. The patient's extremities are cool and pulses are weak. d. The patient has bilateral crackles throughout lung fields.

ANS: A The elevated serum creatinine level indicates that the patient has renal failure as well as heart failure. The crackles, chest pressure, and cool extremities are all symptoms consistent with the patient's diagnosis of cardiogenic shock. DIF: Cognitive Level: Apply (application)

Which action will the nurse include in the plan of care for a patient with impaired functioning of the left glossopharyngeal nerve (CN IX) and the vagus nerve (CN X)? a. Withhold oral fluid or foods. b. Provide highly seasoned foods. c. Insert an oropharyngeal airway. d. Apply artificial tears every hour.

ANS: A The glossopharyngeal and vagus nerves innervate the pharynx and control the gag reflex. A patient with impaired function of these nerves is at risk for aspiration. An oral airway may be needed when a patient is unconscious and unable to maintain the airway, but it will not decrease aspiration risk. Taste and eye blink are controlled by the facial nerve.

A patient with renal failure has been taking aluminum hydroxide/magnesium hydroxide suspension (Maalox) at home for indigestion. The patient arrives for outpatient hemodialysis and is unresponsive to questions and has decreased deep tendon reflexes. Which action should the dialysis nurse take first? a. Notify the patient's health care provider. b. Obtain an order to draw a potassium level. c. Review the magnesium level on the patient's chart. d. Teach the patient about the risk of magnesium-containing antacids

ANS: A The health care provider should be notified immediately. The patient has a history and manifestations consistent with hypermagnesemia. The nurse should check the chart for a recent serum magnesium level and make sure that blood is sent to the laboratory for immediate electrolyte and chemistry determinations. Dialysis should correct the high magnesium levels. The patient needs teaching about the risks of taking magnesium-containing antacids. Monitoring of potassium levels also is important for patients with renal failure, but the patient's current symptoms are not consistent with hyperkalemia

Four patients arrive at the emergency department at the same time. Which patient will the nurse see first? a. An infant with temperature of 102.2° F and diarrhea for 3 days b. A teenager with a sprained ankle and excessive edema c. A middle-aged adult with abdominal pain who is moaning and holding her stomach d. An older adult with nausea and vomiting for 3 days with blood pressure 112/60

ANS: A The infant should be seen first. An infant's proportion of total body water (70% to 80% total body weight) is greater than that of children or adults. Infants and young children have greater water needs and immature kidneys. They are at greater risk for extracellular volume deficit and hypernatremia because body water loss is proportionately greater per kilogram of weight. A teenager with excessive edema from a sprained ankle can wait. A middle-aged adult moaning in pain can wait as can an older adult with a blood pressure of 112/60.

A patient is admitted to the emergency department (ED) for shock of unknown etiology. The first action by the nurse should be to a. administer oxygen. b. obtain a 12-lead electrocardiogram (ECG). c. obtain the blood pressure. d. check the level of consciousness.

ANS: A The initial actions of the nurse are focused on the ABCs—airway, breathing, and circulation—and administration of oxygen should be done first. The other actions should be accomplished as rapidly as possible after oxygen administration.

A patient reports feeling numbness and tingling of the left arm before experiencing a tonic-clonic seizure. The nurse determines that this history is consistent with what type of seizure? a. Focal b. Atonic c. Absence d. Myoclonic

ANS: A The initial symptoms of a focal seizure involve clinical manifestations that are localized to a particular part of the body or brain. Symptoms of an absence seizure are staring and a brief loss of consciousness. In an atonic seizure, the patient loses muscle tone and (typically) falls to the ground. Myoclonic seizures are characterized by a sudden jerk of the body or extremities. DIF: Cognitive Level: Understand (comprehension) REF: 1421 TOP: Nursing Process: Assessment MSC: NCLEX: Physiological Integrity

A patient is experiencing respiratory acidosis. Which organ system is responsible for compensation in this patient? a. Renal b. Endocrine c. Respiratory d. Gastrointestinal

ANS: A The kidneys (renal) are responsible for respiratory acidosis compensation. A problem with the respiratory system causes respiratory acidosis, so another organ system (renal) needs to compensate. Problems with the gastrointestinal and endocrine systems can cause acid-base imbalances, but these systems cannot compensate for an existing imbalance.

The nurse is caring for a diabetic patient in renal failure who is in metabolic acidosis. Which laboratory findings are consistent with metabolic acidosis? a. pH 7.3, PaCO2 36 mm Hg, HCO3- 19 mEq/L b. pH 7.5, PaCO2 35 mm Hg, HCO3- 35 mEq/L c. pH 7.32, PaCO2 47 mm Hg, HCO3- 23 mEq/L d. pH 7.35, PaCO2 40 mm Hg, HCO3- 25 mEq/L

ANS: A The laboratory values that reflect metabolic acidosis are pH 7.3, PaCO2 36 mm Hg, HCO3- 19 mEq/L. A laboratory finding of pH 7.5, PaCO2 35 mm Hg, HCO3- 35 mEq/L is metabolic alkalosis. pH 7.32, PaCO2 47 mm Hg, HCO3- 23 mEq/L is respiratory acidosis. pH 7.35, PaCO2 40 mm Hg, HCO3- 25 mEq/L values are within normal range.

A 39-year-old patient is being evaluated for a possible spinal cord tumor. Which finding by the nurse requires the most immediate action? a. The patient has new onset weakness of both legs. b. The patient complains of chronic severe back pain. c. The patient starts to cry and says, "I feel hopeless." d. The patient expresses anxiety about having surgery.

ANS: A The new onset of symptoms indicates cord compression, which is an emergency that requires rapid treatment to avoid permanent loss of function. The other patient assessments also indicate a need for nursing action but do not require intervention as rapidly as the new onset weakness

The nurse is caring for a group of patients. Which patient will the nurse see first? a. A patient with D5W hanging with the blood b. A patient with type A blood receiving type O blood c. A patient with intravenous potassium chloride that is diluted d. A patient with a right mastectomy and an intravenous site in the left arm

ANS: A The nurse will see the patient with D5W and blood to prevent a medication error. When preparing to administer blood, prime the tubing with 0.9% sodium chloride (normal saline) to prevent hemolysis or breakdown of RBCs. All the rest are normal. A patient with type A blood can receive type O. Type O is considered the universal donor. A patient with a mastectomy should have the IV in the other arm. Potassium chloride should be diluted, and it is never given IV push.

The best chance of survival for a child with cirrhosis is: a. liver transplantation. b. treatment with corticosteroids. c. treatment with immune globulin. d. provision of nutritional support.

ANS: A The only successful treatment for end-stage liver disease and liver failure may be liver transplantation, which has improved the prognosis for many children with cirrhosis. Liver transplantation has revolutionized the approach to cirrhosis. Liver failure and cirrhosis are indications for transplantation. Liver transplantation reflects the failure of other medical and surgical measures, such as treatment with corticosteroids or immune globulin and nutritional support, to prevent or treat cirrhosis.

A patient who is lethargic and exhibits deep, rapid respirations has the following arterial blood gas (ABG) results: pH 7.32, PaO2 88 mm Hg, PaCO2 37 mm Hg, and HCO3 16 mEq/L. How should the nurse interpret these results? a. Metabolic acidosis b. Metabolic alkalosis c. Respiratory acidosis d. Respiratory alkalosis

ANS: A The pH and HCO3 indicate that the patient has a metabolic acidosis. The ABGs are inconsistent with the other responses. DIF: Cognitive Level: Apply (application) REF: 288

A patient with new onset confusion and is being admitted . When making room assignments, the charge nurse should take which action? a. Assign the patient to a room near the nurse's station. b. Place the patient in a room nearest to the water fountain. c. Place the patient on telemetry to monitor for peaked T waves. d. Assign the patient to a semi-private room and place an order for a low-salt diet.

ANS: A The patient should be placed near the nurse's station if confused in order for the staff to closely monitor the patient. To help improve serum sodium levels, water intake is restricted. Therefore a confused patient should not be placed near a water fountain. Peaked T waves are a sign of hyperkalemia, not hyponatremia. A confused patient could be distracting and disruptive for another patient in a semiprivate room. This patient needs sodium replacement, not restriction.

A patient has a ruptured cerebral aneurysm and subarachnoid hemorrhage. Which intervention will the nurse include in the plan of care? a. Apply intermittent pneumatic compression stockings. b. Assist to dangle on edge of bed and assess for dizziness. c. Encourage patient to cough and deep breathe every 4 hours. d. Insert an oropharyngeal airway to prevent airway obstruction.

ANS: A The patient with a subarachnoid hemorrhage usually has minimal activity to prevent cerebral vasospasm or further bleeding and is at risk for venous thromboembolism. Activities such as coughing and sitting up that might increase intracranial pressure or decrease cerebral blood flow are avoided. Because there is no indication that the patient is unconscious, an oropharyngeal airway is inappropriate.

The nurse caring for a patient admitted with burns over 30% of the body surface assesses that urine output has dramatically increased. Which action by the nurse would best ensure adequate kidney function? a. Continue to monitor the urine output. b. Monitor for increased white blood cells (WBCs). c. Assess that blisters and edema have subsided. d. Prepare the patient for discharge from the burn unit.

ANS: A The patient's urine output indicates that the patient is entering the acute phase of the burn injury and moving on from the emergent stage. At the end of the emergent phase, capillary permeability normalizes and the patient begins to diurese large amounts of urine with a low specific gravity. Although this may occur at about 48 hours, it may be longer in some patients. Blisters and edema begin to resolve, but this process requires more time. White blood cells may increase or decrease, based on the patient's immune status and any infectious processes. The WBC count does not indicate kidney function. The patient will likely remain in the burn unit during the acute stage of burn injury

When caring for a patient with a left arm arteriovenous fistula, which action will the nurse include in the plan of care to maintain the patency of the fistula? a. Auscultate for a bruit at the fistula site. b. Assess the quality of the left radial pulse. c. Compare blood pressures in the left and right arms. d. Irrigate the fistula site with saline every 8 to 12 hours.

ANS: A The presence of a thrill and bruit indicates adequate blood flow through the fistula. Pulse rate and quality are not good indicators of fistula patency. Blood pressures should never be obtained on the arm with a fistula. Irrigation of the fistula might damage the fistula, and typically only dialysis staff would access the fistula. DIF: Cognitive Level: Understand (comprehension) REF: 1087 TOP: Nursing Process: Planning MSC: NCLEX: Physiological

Which finding indicates to the nurse that lactulose is effective for an older adult who has advanced cirrhosis? a. The patient is alert and oriented. b. The patient denies nausea or anorexia. c. The patient's bilirubin level decreases. d. The patient has at least one stool daily.

ANS: A The purpose of lactulose in the patient with cirrhosis is to lower ammonia levels and prevent encephalopathy. Although lactulose may be used to treat constipation, that is not the purpose for this patient. Lactulose will not decrease nausea and vomiting or lower bilirubin levels

When prone positioning is used for a patient with acute respiratory distress syndrome (ARDS), which information obtained by the nurse indicates that the positioning is effective? a. The patient's PaO2 is 89 mm Hg, and the SaO2 is 91%. b. Endotracheal suctioning results in clear mucous return. c. Sputum and blood cultures show no growth after 48 hours. d. The skin on the patient's back is intact and without redness.

ANS: A The purpose of prone positioning is to improve the patient's oxygenation as indicated by the PaO2 and SaO2. The other information will be collected but does not indicate whether prone positioning has been effective.

To assess the functioning of the trigeminal and facial nerves (CNs V and VII), the nurse should a. shine a light into the patient's pupil. b. check for unilateral eyelid drooping. c. touch a cotton wisp strand to the cornea. d. have the patient read a magazine or book.

ANS: A The trigeminal and facial nerves are responsible for the corneal reflex. The optic nerve is tested by having the patient read a Snellen chart or a newspaper. Assessment of pupil response to light and ptosis are used to check function of the oculomotor nerve. DIF: Cognitive Level: Understand (comprehension) REF: 1347 TOP: Nursing Process: Assessment MSC: NCLEX: Physiological Integrity

Which patient should the nurse assess first? a. A patient with smoke inhalation who has wheezes and altered mental status b. A patient with full-thickness leg burns who has a dressing change scheduled c. A patient with abdominal burns who is complaining of level 8 (0 to 10 scale) pain d. A patient with 40% total body surface area (TBSA) burns who is receiving IV fluids at 500 mL/hour

ANS: A This patient has evidence of lower airway injury and hypoxemia and should be assessed immediately to determine the need for oxygen or intubation. The other patients should also be assessed as rapidly as possible, but they do not have evidence of life-threatening complications.

The nurse is caring for a patient who has a right-sided chest tube after a right lower lobectomy. Which nursing action can the nurse delegate to the unlicensed assistive personnel (UAP)? a. Document the amount of drainage every 8 hours. b. Obtain samples of drainage for culture from the system. c. Assess patient pain level associated with the chest tube. d. Check the water-seal chamber for the correct fluid level.

ANS: A UAP education includes documentation of intake and output. The other actions are within the scope of practice and education of licensed nursing personnel.

When admitting an acutely confused 20-year-old patient with a head injury, which action should the nurse take? a. Ask family members about the patient's health history. b. Ask leading questions to assist in obtaining health data. c. Wait until the patient is better oriented to ask questions. d. Obtain only the physiologic neurologic assessment data.

ANS: A When admitting a patient who is likely to be a poor historian, the nurse should obtain health history information from others who have knowledge about the patient's health. Waiting until the patient is oriented or obtaining only physiologic data will result in incomplete assessment data, which could adversely affect decision making about treatment. Asking leading questions may result in inaccurate or incomplete information.

An employee spills industrial acids on both arms and legs at work. What is the priority action that the occupational health nurse at the facility should take? a. Remove nonadherent clothing and watch. b. Apply an alkaline solution to the affected area. c. Place cool compresses on the area of exposure. d. Cover the affected area with dry, sterile dressings.

ANS: A With chemical burns, the initial action is to remove the chemical from contact with the skin as quickly as possible. Remove nonadherent clothing, shoes, watches, jewelry, glasses, or contact lenses (if face was exposed). Flush chemical from wound and surrounding area with copious amounts of saline solution or water. Covering the affected area or placing cool compresses on the area will leave the chemical in contact with the skin. Application of an alkaline solution is not recommended.

During change-of-shift report, the nurse learns that a patient with a head injury has decorticate posturing to noxious stimulation. Which positioning shown in the accompanying figure will the nurse expect to observe? a. 1 b. 2 c. 3 d. 4

ANS: A With decorticate posturing, the patient exhibits internal rotation and adduction of the arms with flexion of the elbows, wrists, and fingers. The other illustrations are of decerebrate, mixed decorticate/decerebrate posturing, and opisthotonic posturing. DIF: Cognitive Level: Understand (comprehension) REF: 1360 TOP: Nursing Process: Assessment MSC: NCLEX: Physiological Integrity

After receiving change-of-shift report on the following four patients, which patient should the nurse see first? a. A 60-year-old patient with right-sided weakness who has an infusion of tPA prescribed b. A 50-year-old patient who has atrial fibrillation and a new order for warfarin (Coumadin) c. A 40-year-old patient who experienced a transient ischemic attack yesterday who has a dose of aspirin due d. A 30-year-old patient with a subarachnoid hemorrhage 2 days ago who has nimodipine (Nimotop) scheduled

ANS: A tPA needs to be infused within the first few hours after stroke symptoms start in order to be effective in minimizing brain injury. The other medications should also be given as quickly as possible, but timing of the medications is not as critical. DIF: Cognitive Level: Apply (application) REF: 1398 OBJ: Special Questions: Prioritization; Multiple Patients TOP: Nursing Process: Implementation MSC: NCLEX: Safe and Effective Care Environment

Latex allergy is suspected in a child with spina bifida. Appropriate nursing interventions include: a. Avoiding using any latex product. b. Using only nonallergenic latex products. c. Administering medication for long-term desensitization. d. Teaching the family about long-term management of asthma.

ANS: A Feedback A Care must be taken that individuals who are at high risk for latex allergies do not come in direct or secondary contact with products or equipment containing latex at any time during medical treatment. Latex allergy is estimated to occur in 75% of this patient population. B There are no nonallergic latex products. C At this time, desensitization is not an option. D The child does not have asthma. The parents must be taught about allergy and the risk of anaphylaxis.

The most common problem of children born with a myelomeningocele is: a. Neurogenic bladder b. Intellectual impairment. c. Respiratory compromise. d. Cranioschisis

ANS: A Feedback A Myelomeningocele is one of the most common causes of neuropathic (neurogenic) bladder dysfunction among children. B Risk of intellectual impairment is minimized through early intervention and management of hydrocephalus. C Respiratory compromise is not a common problem in myelomeningocele. D Cranioschisis is a skull defect through which various tissues protrude. It is not associated with myelomeningocele.

Which preventive actions by the nurse will help limit the development of systemic inflammatory response syndrome (SIRS) in patients admitted to the hospital (select all that apply)? a.Use aseptic technique when caring for invasive lines or devices. b.Ambulate postoperative patients as soon as possible after surgery. c.Remove indwelling urinary catheters as soon as possible after surgery. d.Advocate for parenteral nutrition for patients who cannot take oral feedings. e.Administer prescribed antibiotics within 1 hour for patients with possible sepsis.

ANS: A, B, C, E Because sepsis is the most frequent etiology for SIRS, measures to avoid infection such as removing indwelling urinary catheters as soon as possible, use of aseptic technique, and early ambulation should be included in the plan of care. Adequate nutrition is important in preventing SIRS. Enteral, rather than parenteral, nutrition is preferred when patients are unable to take oral feedings because enteral nutrition helps maintain the integrity of the intestine, thus decreasing infection risk. Antibiotics should be administered within 1 hour after being prescribed to decrease the risk of sepsis progressing to SIRS.

After an unimmunized individual is exposed to hepatitis B through a needle-stick injury, which actions will the nurse plan to take (select all that apply)? a. Administer hepatitis B vaccine. b. Test for antibodies to hepatitis B. c. Teach about -interferon therapy. d. Give hepatitis B immune globulin. e. Teach about choices for oral antiviral therapy.

ANS: A, B, D The recommendations for hepatitis B exposure include both vaccination and immune globulin administration. In addition, baseline testing for hepatitis B antibodies will be needed. Interferon and oral antivirals are not used for hepatitis B prophylaxis. 2.21.44

A 27-year-old patient who has been treated for status epilepticus in the emergency department will be transferred to the medical nursing unit. Which equipment should the nurse have available in the patient's assigned room (select all that apply)? a. Side-rail pads b. Tongue blade c. Oxygen mask d. Suction tubing e. Urinary catheter f. Nasogastric tube

ANS: A, C, D The patient is at risk for further seizures, and oxygen and suctioning may be needed after any seizures to clear the airway and maximize oxygenation. The bed's side rails should be padded to minimize the risk for patient injury during a seizure. Use of tongue blades during a seizure is contraindicated. Insertion of a nasogastric (NG) tube is not indicated because the airway problem is not caused by vomiting or abdominal distention. A urinary catheter is not required unless there is urinary retention .DIF: Cognitive Level: Apply (application) REF: 1426 TOP: Nursing Process: Planning MSC: NCLEX: Physiological Integrity

IV potassium chloride (KCl) 60 mEq is prescribed for treatment of a patient with severe hypokalemia. Which action should the nurse take? a. Administer the KCl as a rapid IV bolus. b. Infuse the KCl at a rate of 10 mEq/hour. c. Only give the KCl through a central venous line. d. Discontinue cardiac monitoring during the infusion.

ANS: B IV KCl is administered at a maximal rate of 10 mEq/hr. Rapid IV infusion of KCl can cause cardiac arrest. KCl can cause inflammation of peripheral veins, but it can be administered by this route. Cardiac monitoring should be continued while patient is receiving potassium because of the risk for dysrhythmias.

A nurse begins infusing a 250-mL bag of IV fluid at 1845 on Monday and programs the pump to infuse at 50 mL/hr. At what time should the infusion be completed? a. 2300 Monday b. 2345 Monday c. 0015 Tuesday d. 0045 Tuesday

ANS: B 250 mL ÷ 50 mL/hr = 5 hr 1845 + 5 hr = 2345, which would be 2345 on Monday.

A 38-year-old patient who had a kidney transplant 8 years ago is receiving the immunosuppressants tacrolimus (Prograf), cyclosporine (Sandimmune), and prednisone (Deltasone). Which assessment data will be of most concern to the nurse? a. The blood glucose is 144 mg/dL. b. There is a nontender axillary lump. c. The patient's skin is thin and fragile. d. The patient's blood pressure is 150/92.

ANS: B A nontender lump suggests a malignancy such as a lymphoma, which could occur as a result of chronic immunosuppressive therapy. The elevated glucose, skin change, and hypertension are possible side effects of the prednisone and should be addressed, but they are not as great a concern as the possibility of a malignancy

Which action will the emergency department nurse anticipate for a patient diagnosed with a concussion who did not lose consciousness? a. Coordinate the transfer of the patient to the operating room. b. Provide discharge instructions about monitoring neurologic status. c. Transport the patient to radiology for magnetic resonance imaging (MRI). d. Arrange to admit the patient to the neurologic unit for 24 hours of observation.

ANS: B A patient with a minor head trauma is usually discharged with instructions about neurologic monitoring and the need to return if neurologic status deteriorates. MRI, hospital admission, or surgery are not usually indicated in a patient with a concussion

An unconscious male patient has just arrived in the emergency department after a head injury caused by a motorcycle crash. Which order should the nurse question? a. Obtain x-rays of the skull and spine. b. Prepare the patient for lumbar puncture. c. Send for computed tomography (CT) scan. d. Perform neurologic checks every 15 minutes.

ANS: B After a head injury, the patient may be experiencing intracranial bleeding and increased intracranial pressure, which could lead to herniation of the brain if a lumbar puncture is performed. The other orders are appropriate.

Which statement by patient who is being discharged from the emergency department (ED) after a concussion indicates a need for intervention by the nurse? a. "I will return if I feel dizzy or nauseated." b. "I am going to drive home and go to bed." c. "I do not even remember being in an accident." d. "I can take acetaminophen (Tylenol) for my headache."

ANS: B After a head injury, the patient should avoid driving and operating heavy machinery. Retrograde amnesia is common after a concussion. The patient can take acetaminophen for headache and should return if symptoms of increased intracranial pressure such as dizziness or nausea occur. DIF: Cognitive Level: Apply (application)

The nurse recognizes that teaching a patient following a laparoscopic cholecystectomy has been effective when the patient makes which statement? a. "I can expect yellow-green drainage from the incision for a few days." b. "I can remove the bandages on my incisions tomorrow and take a shower." c. "I should plan to limit my activities and not return to work for 4 to 6 weeks." d. "I will need to maintain a low-fat diet for life because I no longer have a gallbladder."

ANS: B After a laparoscopic cholecystectomy, the patient will have Band-Aids in place over the incisions. Patients are discharged the same (or next) day and have few restrictions on activities of daily living. Drainage from the incisions would be abnormal, and the patient should be instructed to call the health care provider if this occurs. A low-fat diet may be recommended for a few weeks after surgery but will not be a lifelong requirement.

Before administration of captopril to a patient with stage 2 chronic kidney disease (CKD), the nurse will check the patient's a. glucose. b. potassium. c. creatinine. d. phosphate.

ANS: B Angiotensin-converting enzyme (ACE) inhibitors are frequently used in patients with CKD because they delay the progression of the CKD, but they cause potassium retention. Therefore careful monitoring of potassium levels is needed in patients who are at risk for hyperkalemia. The other laboratory values would also be monitored in patients with CKD but would not affect whether the captopril was given or not. DIF: Cognitive Level: Apply (application) REF: 1075 TOP: Nursing Process: Assessment MSC: NCLEX: Physiological Integrity

After change-of-shift report in the progressive care unit, who should the nurse care for first? a. Patient who had an inferior myocardial infarction 2 days ago and has crackles in the lung bases b. Patient with suspected urosepsis who has new orders for urine and blood cultures and antibiotics c. Patient who had a T5 spinal cord injury 1 week ago and currently has a heart rate of 54 beats/minute d. Patient admitted with anaphylaxis 3 hours ago who now has clear lung sounds and a blood pressure of 108/58 mm Hg

ANS: B Antibiotics should be administered within the first hour for patients who have sepsis or suspected sepsis in order to prevent progression to systemic inflammatory response syndrome (SIRS) and septic shock. The data on the other patients indicate that they are more stable. Crackles heard only at the lung bases do not require immediate intervention in a patient who has had a myocardial infarction. Mild bradycardia does not usually require atropine in patients who have a spinal cord injury. The findings for the patient admitted with anaphylaxis indicate resolution of bronchospasm and hypotension.

A patient is being treated with an antibiotic for an infection what explination should the nurse give to this patient about this medication? A.antibiotics will decrease the pain at the site B.an antibiotic helps to kill the infection causing the inflammation C.an antibiotic inhibits cyclooxygenase an enzyme in the body D.antibiotics will reduce the patients fever

ANS: B Antimicrobials treat the underlying cause of the infection which leads to inflammation. Analgesics and nonsteroidal antiinflammatory drugs (NSAIDs) help to treat pain. NSAIDs and other antipyretics are cyclooxygenase inhibitors. Antipyretics help to reduce fever.

Which information given by a 70-yr-old patient during a health history indicates to the nurse that the patient should be screened for hepatitis C? a. The patient had a blood transfusion in 2005. b. The patient used IV drugs about 20 years ago. c. The patient frequently eats in fast-food restaurants. d. The patient traveled to a country with poor sanitation.

ANS: B Any patient with a history of IV drug use should be tested for hepatitis C. Blood transfusions given after 1992 (when an antibody test for hepatitis C became available) do not pose a risk for hepatitis C. Hepatitis C is not spread by the oral-fecal route and therefore is not caused by contaminated food or by traveling in underdeveloped countries.

Which finding is the best indicator that the fluid resuscitation for a 90 kg patient with hypovolemic shock has been effective? a. Hemoglobin is within normal limits. b. Urine output is 65 mL over the last hour. c. Central venous pressure (CVP) is normal. d. Mean arterial pressure (MAP) is 72 mm Hg.

ANS: B Assessment of end organ perfusion, such as an adequate urine output, is the best indicator that fluid resuscitation has been successful. The hemoglobin level, CVP, and MAP are useful in determining the effects of fluid administration, but they are not as useful as data indicating good organ perfusion.

The nurse is caring for a patient who has cirrhosis. Which data obtained by the nurse during the assessment will be of most concern? a. The patient complains of right upper-quadrant pain with palpation. b. The patient's hands flap back and forth when the arms are extended. c. The patient has ascites and a 2-kg weight gain from the previous day. d. The patient's abdominal skin has multiple spider-shaped blood vessels.

ANS: B Asterixis indicates that the patient has hepatic encephalopathy, and hepatic coma may occur. The spider angiomas and right upper quadrant abdominal pain are not unusual for the patient with cirrhosis and do not require a change in treatment. The ascites and weight gain indicate the need for treatment but not as urgently as the changes in neurologic status. DIF: Cognitive Level: Analyze (analysis) REF: 990 OBJ: Special Questions: Prioritization TOP: Nursing Process: Assessment MSC: NCLEX: Physiological Integrity

Several patients have been hospitalized for diagnosis of neurologic problems. Which patient will the nurse assess first? a. Patient with a transient ischemic attack (TIA) returning from carotid duplex studies b. Patient with a brain tumor who has just arrived on the unit after a cerebral angiogram c. Patient with a seizure disorder who has just completed an electroencephalogram (EEG) d. Patient prepared for a lumbar puncture whose health care provider is waiting for assistance

ANS: B Because cerebral angiograms require insertion of a catheter into the femoral artery, bleeding is a possible complication. The nurse will need to check the pulse, blood pressure, and the catheter insertion site in the groin as soon as the patient arrives. Carotid duplex studies and EEG are noninvasive. The nurse will need to assist with the lumbar puncture as soon as possible, but monitoring for hemorrhage after cerebral angiogram has a higher priority.

A 42-yr-old patient admitted with acute kidney injury due to dehydration has oliguria, anemia, and hyperkalemia. Which prescribed action should the nurse take first? a. Insert a urinary retention catheter. b. Place the patient on a cardiac monitor. c. Administer epoetin alfa (Epogen, Procrit). d. Give sodium polystyrene sulfonate (Kayexalate).

ANS: B Because hyperkalemia can cause fatal cardiac dysrhythmias, the initial action should be to monitor the cardiac rhythm. Kayexalate and Epogen will take time to correct the hyperkalemia and anemia. The catheter allows monitoring of the urine output but does not correct the cause of the renal failure. DIF: Cognitive Level: Analyze (analysis) REF: 1073 OBJ: Special Questions: Prioritization TOP: Nursing Process: Implementation MSC: NCLEX: Physiological Integrity

The nurse is caring for a patient who has septic shock. Which assessment finding is most important for the nurse to report to the health care provider? a. Blood pressure (BP) 92/56 mm Hg b. Skin cool and clammy c. Oxygen saturation 92% d. Heart rate 118 beats/minute

ANS: B Because patients in the early stage of septic shock have warm and dry skin, the patient's cool and clammy skin indicates that shock is progressing. The other information will also be reported, but does not indicate deterioration of the patient's status.

A 36-yr-old female patient is receiving treatment for chronic hepatitis C with pegylated interferon (PEG-Intron, Pegasys), ribavirin (Rebetol), and telaprevir (Incivek). Which finding is important to communicate to the health care provider to suggest a change in therapy? a. Weight loss of 2 lb (1 kg) b. Positive urine pregnancy test c. Hemoglobin level of 10.4 g/dL d. Complaints of nausea and anorexia

ANS: B Because ribavirin is teratogenic, the medication will need to be discontinued immediately. Anemia, weight loss, and nausea are common adverse effects of the prescribed regimen and may require actions such as patient teaching, but they would not require immediate cessation of the therapy. DIF: Cognitive Level: Apply (application) REF: 981 OBJ: Special Questions: Prioritization TOP: Nursing Process: Assessment MSC: NCLEX: Physiological Integrity

After endotracheal suctioning, the nurse notes that the intracranial pressure (ICP) for a patient with a traumatic head injury has increased from 14 to 17 mm Hg. Which action should the nurse take first? a. Document the increase in intracranial pressure. b. Ensure that the patient's neck is in neutral position. c. Notify the health care provider about the change in pressure. d. Increase the rate of the prescribed propofol (Diprivan) infusion.

ANS: B Because suctioning will cause a transient increase in ICP, the nurse should initially check for other factors that might be contributing to the increase and observe the patient for a few minutes. Documentation is needed, but this is not the first action. There is no need to notify the health care provider about this expected reaction to suctioning. Propofol is used to control patient anxiety or agitation. There is no indication that anxiety has contributed to the increase in ICP.

A patient with paraplegia resulting from a T9 spinal cord injury has a neurogenic reflexic bladder. Which action will the nurse include in the plan of care? a. Teach the patient the Credé method. b. Instruct the patient how to self-catheterize. c. Catheterize for residual urine after voiding. d. Assist the patient to the toilet every 2 hours.

ANS: B Because the patient's bladder is spastic and will empty in response to overstretching of the bladder wall, the most appropriate method is to avoid incontinence by emptying the bladder at regular intervals through intermittent catheterization. Assisting the patient to the toilet will not be helpful because the bladder will not empty. The Credé method is more appropriate for a bladder that is flaccid, such as occurs with areflexic neurogenic bladder. Catheterization after voiding will not resolve the patient's incontinence

Which interventions should the nurse include in the plan of care for the infant awaiting surgical closure of a myelomeningocele sac? a. Open to air b. Covered with a sterile, moist, nonadherent dressing c. Reinforcement of the original dressing if drainage noted d. A diaper secured over the dressing

ANS: B Before surgical closure, the myelomeningocele is prevented from drying by the application of a sterile, moist, nonadherent dressing over the defect. The moistening solution is usually sterile normal saline. Dressings are changed frequently (every 2 to 4 hours), and the sac is closely inspected for leaks, abrasions, irritation, and any signs of infection. The sac must be carefully cleansed if it becomes soiled or contaminated. The original dressing would not be reinforced but changed as needed. A diaper is not placed over the dressing because stool contamination can occur.

One of the most frequent causes of hypovolemic shock in children is: a. Myocardial infarction. b. Blood loss. c. Anaphylaxis. d. Congenital heart disease.

ANS: B Blood loss and extracellular fluid loss are two of the most frequent causes of hypovolemic shock in children. Myocardial infarction is rare in a child; if it occurred, the resulting shock would be cardiogenic, not hypovolemic. Anaphylaxis results in distributive shock from extreme allergy or hypersensitivity to a foreign substance. Congenital heart disease tends to contribute to hypervolemia, not hypovolemia.

Which statement by the nurse when explaining the purpose of positive end-expiratory pressure (PEEP) to the patient's caregiver is accurate? a. "PEEP will push more air into the lungs during inhalation." b. "PEEP prevents the lung air sacs from collapsing during exhalation." c. "PEEP will prevent lung damage while the patient is on the ventilator." d. "PEEP allows the breathing machine to deliver 100% O2 to the lungs."

ANS: B By preventing alveolar collapse during expiration, PEEP improves gas exchange and oxygenation. PEEP will not prevent lung damage (e.g., fibrotic changes that occur with ARDS), push more air into the lungs, or change the fraction of inspired oxygen (FIO2) delivered to the patient.

Which information will the nurse monitor in order to determine the effectiveness of prescribed calcium carbonate (Caltrate) for a patient with chronic kidney disease (CKD)? a. Blood pressure b. Phosphate level c. Neurologic status d. Creatinine clearance

ANS: B Calcium carbonate is prescribed to bind phosphorus and prevent mineral and bone disease in patients with CKD. The other data will not be helpful in evaluating the effectiveness of calcium carbonate DIF: Cognitive Level: Apply (application) REF: 1081 TOP: Nursing Process: Evaluation MSC: NCLEX: Physiological Integrity

A patient admitted with a diffuse axonal injury has a systemic blood pressure (BP) of 106/52 mm Hg and an intracranial pressure (ICP) of 14 mm Hg. Which action should the nurse take first? a. Document the BP and ICP in the patient's record. b. Report the BP and ICP to the health care provider. c. Elevate the head of the patient's bed to 60 degrees. d. Continue to monitor the patient's vital signs and ICP.

ANS: B Calculate the cerebral perfusion pressure (CPP): (CPP = mean arterial pressure [MAP] - ICP). MAP = DBP + 1/3 (systolic blood pressure [SBP] - diastolic blood pressure [DBP]). Therefore the (MAP) is 70 and the CPP is 56 mm Hg, which is below the normal of 60 to 100 mm Hg and approaching the level of ischemia and neuronal death. Immediate changes in the patient's therapy such as fluid infusion or vasopressor administration are needed to improve the cerebral perfusion pressure. Adjustments in the head elevation should only be done after consulting with the health care provider. Continued monitoring and documentation will also be done, but they are not the first actions that the nurse should take

Which nursing action is a priority for a patient who has suffered a burn injury while working on an electrical power line? a. Obtain the blood pressure. b. Stabilize the cervical spine. c. Assess for the contact points. d. Check alertness and orientation.

ANS: B Cervical spine injuries are commonly associated with electrical burns. Therefore stabilization of the cervical spine takes precedence after airway management. The other actions are also included in the emergent care after electrical burns, but the most important action is to avoid spinal cord injury.

A 20-yr-old male patient is admitted with a head injury after a collision while playing football. After noting that the patient has developed clear nasal drainage, which action should the nurse take? a. Have the patient gently blow the nose. b. Check the drainage for glucose content. c. Teach the patient that rhinorrhea is expected after a head injury. d. Obtain a specimen of the fluid to send for culture and sensitivity.

ANS: B Clear nasal drainage in a patient with a head injury suggests a dural tear and cerebrospinal fluid (CSF) leakage. If the drainage is CSF, it will test positive for glucose. Fluid leaking from the nose will have normal nasal flora, so culture and sensitivity will not be useful. Blowing the nose is avoided to prevent CSF leakage.

When teaching about clopidogrel (Plavix), the nurse will tell the patient with cerebral atherosclerosis a. to monitor and record the blood pressure daily. b. to call the health care provider if stools are tarry. c. that clopidogrel will dissolve clots in the cerebral arteries. d. that clopidogrel will reduce cerebral artery plaque formation.

ANS: B Clopidogrel inhibits platelet function and increases the risk for gastrointestinal bleeding, so patients should be advised to notify the health care provider about any signs of bleeding. The medication does not lower blood pressure, decrease plaque formation, or dissolve clots.

A female patient with chronic kidney disease (CKD) is receiving peritoneal dialysis with 2-L inflows. Which information should the nurse report promptly to the health care provider? a. The patient has an outflow volume of 1800 mL. b. The patient's peritoneal effluent appears cloudy. c. The patient's abdomen appears bloated after the inflow. d. The patient has abdominal pain during the inflow phase.

ANS: B Cloudy-appearing peritoneal effluent is a sign of peritonitis and should be reported immediately so that treatment with antibiotics can be started. The other problems can be addressed through nursing interventions such as slowing the inflow and repositioning the patient. DIF: Cognitive Level: Apply (application) REF: 1087 TOP: Nursing Process: Assessment MSC: NCLEX: Physiological Integrity

Several weeks after a stroke, a 50-yr-old male patient has impaired awareness of bladder fullness, resulting in urinary incontinence. Which nursing intervention should be planned to begin an effective bladder training program? a. Limit fluid intake to 1200 mL daily to reduce urine volume. b. Assist the patient onto the bedside commode every 2 hours. c. Perform intermittent catheterization after each voiding to check for residual urine. d. Use an external "condom" catheter to protect the skin and prevent embarrassment.

ANS: B Developing a regular voiding schedule will prevent incontinence and may increase patient awareness of a full bladder. A 1200-mL fluid restriction may lead to dehydration. Intermittent catheterization and use of a condom catheter are appropriate in the acute phase of stroke, but should not be considered solutions for long-term management because of the risks for urinary tract infection and skin breakdown.

A patient has arrived for a scheduled hemodialysis session. Which nursing action is most appropriate for the registered nurse (RN) to delegate to a dialysis technician? a. Teach the patient about fluid restrictions. b. Check blood pressure before starting dialysis. c. Assess for causes of an increase in predialysis weight. d. Determine the ultrafiltration rate for the hemodialysis.

ANS: B Dialysis technicians are educated in monitoring for blood pressure. Assessment, adjustment of the appropriate ultrafiltration rate, and patient teaching require the education and scope of practice of an RN. DIF: Cognitive Level: Apply (application) REF: 1089 OBJ: Special Questions: Delegation TOP: Nursing Process: Planning MSC: NCLEX: Safe and Effective Care Environment

The patient had diarrhea for 5 days and developed an acid-base imbalance. Which statement would indicate that the nurse's teaching about the acid-base imbalance has been effective? a. "To prevent another problem, I should eat less sodium during diarrhea." b. "My blood became too acid because I lost some base in the diarrhea fluid." c. "Diarrhea removes fluid from the body, so I should drink more ice water." d. "I should try to slow my breathing so my acids and bases will be balanced."

ANS: B Diarrhea causes metabolic acidosis through loss of bicarbonate, which is a base. Eating less sodium during diarrhea increases the risk of ECV deficit. Although diarrhea does remove fluid from the body, it also removes sodium and bicarbonate which need to be replaced. Rapid deep respirations are the compensatory mechanism for metabolic acidosis and should be encouraged rather than stopped.

Which information in a patient's history indicates to the nurse that the patient is not an appropriate candidate for kidney transplantation? a. The patient has type 1 diabetes. b. The patient has metastatic lung cancer. c. The patient has a history of chronic hepatitis C infection. d. The patient is infected with human immunodeficiency virus.

ANS: B Disseminated malignancies are a contraindication to transplantation. The conditions of the other patients are not contraindications for kidney transplant. DIF: Cognitive Level: Understand (comprehension) REF: 1092 TOP: Nursing Process: Assessment MSC: NCLEX: Physiological Integrity

To verify the correct placement of an oral endotracheal tube (ET) after insertion, the best initial action by the nurse is to a. obtain a portable chest x-ray. b. use an end-tidal CO2 monitor. c. auscultate for bilateral breath sounds. d. observe for symmetrical chest movement.

ANS: B End-tidal CO2 monitors are currently recommended for rapid verification of ET placement. Auscultation for bilateral breath sounds and checking chest expansion are also used, but they are not as accurate as end-tidal CO2 monitoring. A chest x-ray confirms the placement but is done after the tube is secured. DIF: Cognitive Level: Analyze (analysis)

The following interventions are ordered by the health care provider for a patient who has respiratory distress and syncope after eating strawberries. Which will the nurse complete first? a. Start a normal saline infusion. b. Give epinephrine (Adrenalin). c. Start continuous ECG monitoring. d. Give diphenhydramine (Benadryl).

ANS: B Epinephrine rapidly causes peripheral vasoconstriction, dilates the bronchi, and blocks the effects of histamine and reverses the vasodilation, bronchoconstriction, and histamine release that cause the symptoms of anaphylaxis. The other interventions are also appropriate but would not be the first ones completed.

While close family members are visiting, a patient has a respiratory arrest, and resuscitation is started. Which action by the nurse is best? a. Tell the family members that watching the resuscitation will be very stressful. b. Ask family members if they wish to remain in the room during the resuscitation. c. Take the family members quickly out of the patient room and remain with them. d. Assign a staff member to wait with family members just outside the patient room.

ANS: B Evidence indicates that many family members want the option of remaining in the room during procedures such as cardiopulmonary resuscitation (CPR) and that this decreases anxiety and facilitates grieving. The other options may be appropriate if the family decides not to remain with the patient. DIF: Cognitive Level: Analyze (analysis)

Which action should the nurse take to evaluate treatment effectiveness for a patient who has hepatic encephalopathy? a. Request that the patient stand on one foot. b. Ask the patient to extend both arms forward. c. Request that the patient walk with eyes closed. d. Ask the patient to perform the Valsalva maneuver.

ANS: B Extending the arms allows the nurse to check for asterixis, a classic sign of hepatic encephalopathy. The other tests might also be done as part of the neurologic assessment but would not be diagnostic for hepatic encephalopathy.

a patient with a history of chronic kidney injury is seen in the clinic for bladder infection describes the following symptoms which information is most important for the nurse to report to the health care provider? A. Burning with urination B. Left sided flank pain C. Intermittent Hematuria D. Urinary urgency

ANS: B Flank pain indicates that the patient may have developed pyelonephritis as a complication of the bladder infection. The other clinical manifestations are consistent with a lower urinary tract infection (UTI).

The nurse is titrating the IV fluid infusion rate immediately after a patient has had kidney transplantation. Which parameter will be most important for the nurse to consider? a. Heart rate b. Urine output c. Creatinine clearance d. Blood urea nitrogen (BUN) level

ANS: B Fluid volume is replaced based on urine output after transplant because the urine output can be as high as a liter an hour. The other data will be monitored but are not the most important determinants of fluid infusion rate.

When caring for a patient with renal failure on a low phosphate diet, the nurse will inform unlicensed assistive personnel (UAP) to remove which food from the patient's food tray? a. Grape juice b. Milk carton c. Mixed green salad d. Fried chicken breast

ANS: B Foods high in phosphate include milk and other dairy products, so these are restricted on low-phosphate diets. Green, leafy vegetables; high-fat foods; and fruits/juices are not high in phosphate and are not restricted

The intensive care unit (ICU) nurse educator determines that teaching a new staff nurse about arterial pressure monitoring has been effective when the nurse a. balances and calibrates the monitoring equipment every 2 hours. b. positions the zero-reference stopcock line level with the phlebostatic axis. c. ensures that the patient is supine with the head of the bed flat for all readings. d. rechecks the location of the phlebostatic axis with changes in the patient's position.

ANS: B For accurate measurement of pressures, the zero-reference level should be at the phlebostatic axis. There is no need to rebalance and recalibrate monitoring equipment every 2 hours. Accurate hemodynamic readings are possible with the patient's head raised to 45 degrees or in the prone position. The anatomic position of the phlebostatic axis does not change when patients are repositioned. DIF: Cognitive Level: Apply (application)

A nurse is caring for a client with a diagnosis of chronic kidney disease who is receiving dialysis epoetin have been prescribed to be administered subcutaneously the nurse is drawing the medication from a single use vial what should the nurse do to prepare the medication A. Obtain the medication from the medication freezer and allow to thaw B. Draw up the medications and discard the unused portion C. Mix the medication with 0.1 ml heparin before administration to avoid clotting D. Shake the vile before drawing up the medication

ANS: B If you use the vials that have one dose, you might not use all of the medicine in each vial. Use each vial only once and throw any extra medicine away. Do not save an opened vial. Do not shake the vial before using it. Do not use this medicine if it has been frozen.

Which clinical changes occur as a result of septic shock? a. Hypothermia b. Increased cardiac output c. Vasoconstriction d. Angioneurotic edema

ANS: B Increased cardiac output, which results in warm, flushed skin, is one of the manifestations of septic shock. Fever and chills are characteristic of septic shock. Vasodilation is more common in septic shock. Angioneurotic edema occurs as a manifestation in anaphylactic shock.

After evacuation of an epidural hematoma, a patient's intracranial pressure (ICP) is being monitored with an intraventricular catheter. Which information obtained by the nurse requires urgent communication with the health care provider? a. Pulse of 102 beats/min b. Temperature of 101.6° F c. Intracranial pressure of 15 mm Hg d. Mean arterial pressure of 90 mm Hg

ANS: B Infection is a serious consideration with ICP monitoring, especially with intraventricular catheters. The temperature indicates the need for antibiotics or removal of the monitor. The ICP, arterial pressure, and apical pulse only require ongoing monitoring at this time.

Propranolol (Inderal), a β-adrenergic blocker that inhibits sympathetic nervous system activity, is prescribed for a patient who has extreme anxiety about public speaking. The nurse monitors the patient for a. dry mouth. b. bradycardia. c. constipation. d. urinary retention.

ANS: B Inhibition of the fight or flight response leads to a decreased heart rate. Dry mouth, constipation, and urinary retention are associated with peripheral nervous system blockade. DIF: Cognitive Level: Understand (comprehension) REF: eTable 56-2 TOP: Nursing Process: Evaluation MSC: NCLEX: Physiological Integrity

The nurse is caring for a patient with increased intracranial pressure. Which action is considered unsafe? A. Aligning the neck with the body B. Clustering many nursing activities C. Elevating the head of the bed 30 degrees D. Providing stool softeners or laxatives as ordered

ANS: B It is important to minimize stress and activities that could increase intracranial pressure. Combining many nursing activities could increase oxygen demand and intracranial pressure. This would not be safe. Interventions which can promote venous outflow can help decrease intracranial pressure. The stress of constipation or bowel movements can increase intracranial pressure; stool softeners or laxatives can minimize this.

A diabetic patient's arterial blood gas (ABG) results are pH 7.28; PaCO2 34 mm Hg; PaO2 85 mm Hg; HCO3- 18 mEq/L. The nurse would expect which finding? a. Intercostal retractions b. Kussmaul respirations c. Low oxygen saturation (SpO2) d. Decreased venous O2 pressure

ANS: B Kussmaul (deep and rapid) respirations are a compensatory mechanism for metabolic acidosis. The low pH and low bicarbonate result indicate metabolic acidosis. Intercostal retractions, a low oxygen saturation rate, and a decrease in venous O2 pressure would not be caused by acidosis.

The home health registered nurse (RN) is planning care for a patient with a seizure disorder related to a recent head injury. Which nursing action can be delegated to a licensed practical/vocational nurse (LPN/LVN)? a. Make referrals to appropriate community agencies. b. Place medications in the home medication organizer. c. Teach the patient and family how to manage seizures. d. Assess for use of medications that may precipitate seizures.

ANS: B LPN/LVN education includes administration of medications. The other activities require RN education and scope of practice. DIF: Cognitive Level: Apply (application) REF: 1426 OBJ: Special Questions: Delegation TOP: Nursing Process: Planning MSC: NCLEX: Safe and Effective Care Environment

A nurse is caring for a client who is vomiting. For which acid-base imbalance does the nurse assess the client? A. Metabolic acidosis B. Metabolic alkalosis C. Respiratory acidosis D. Respiratory alkalosis

ANS: B Loss of gastric fluid by way of nasogastric suction or vomiting results in metabolic alkalosis. This is because of the loss of hydrochloric acid, a potent acid. The situation results in an alkalotic condition. The respiratory system is not involved.

The nurse will teach a patient with chronic pancreatitis to take the prescribed pancrelipase (Viokase) a. at bedtime. c. in the morning. b. with meals. d. for abdominal pain.

ANS: B Pancreatic enzymes are used to help with digestion of nutrients and should be taken with every meal.

Which action will the nurse include in the plan of care for a patient who has been diagnosed with chronic hepatitis B? a. Advise limiting alcohol intake to 1 drink daily. b. Schedule for liver cancer screening every 6 months. c. Initiate administration of the hepatitis C vaccine series. d. Monitor anti-hepatitis B surface antigen (anti-HBs) levels.

ANS: B Patients with chronic hepatitis are at higher risk for development of liver cancer, and should be screened for liver cancer every 6 to 12 months. Patients with chronic hepatitis are advised to completely avoid alcohol. There is no hepatitis C vaccine. Because anti-HBs is present whenever there has been a past hepatitis B infection or vaccination, there is no need to regularly monitor for this antibody.

When a patient with acute kidney injury (AKI) has an arterial blood pH of 7.30, the nurse will expect an assessment finding of a. persistent skin tenting b. rapid, deep respirations. c. hot, flushed face and neck. d. bounding peripheral pulses.

ANS: B Patients with metabolic acidosis caused by AKI may have Kussmaul respirations as the lungs try to regulate carbon dioxide. Bounding pulses and vasodilation are not associated with metabolic acidosis. Because the patient is likely to have fluid retention, poor skin turgor would not be a finding in AKI. DIF: Cognitive Level: Apply (application) REF: 1072 TOP: Nursing Process: Assessment MSC: NCLEX: Physiological Integrity

The nurse observes that the patient's calcium is elevated. When checking the phosphate level,what does the nurse expect to see? a. Increased b. Decreased c. Equal to calcium d. No change in phosphate

ANS: B Phosphate will decrease. Serum calcium and phosphate have an inverse relationship. When one is elevated, the other decreases, except in some patients with end-stage renal disease. DIF:Apply (application)REF:941

A nurse is assessing a patient. Which assessment finding should cause a nurse to further assess for extracellular fluid volume deficit? a. Moist mucous membranes b. Postural hypotension c. Supple skin turgor d. Pitting edema

ANS: B Physical examination findings of deficit include postural hypotension, tachycardia, thready pulse, dry mucous membranes, and poor skin turgor. Pitting edema indicates that the patient may be retaining excess extracellular fluid. DIF:Apply (application)REF:940

A nurse is administering a diuretic to a patient and teaching the patient about foods to increase. Which food choices by the patient will best indicate successful teaching? a. Milk and cheese b. Potatoes and fresh fruit c. Canned soups and vegetables d. Whole grains and dark green leafy vegetables

ANS: B Potatoes and fruits are high in potassium. Milk and cheese are high in calcium. Canned soups and vegetables are high in sodium. Whole grains and dark green leafy vegetables are high in magnesium. DIF:Apply (application)REF:941

For a patient with cirrhosis, which nursing action can the registered nurse (RN) delegate to unlicensed assistive personnel (UAP)? a. Assessing the patient for jaundice b. Providing oral hygiene after a meal c. Palpating the abdomen for distention d. Teaching the patient the prescribed diet

ANS: B Providing oral hygiene is within the scope of UAP. Assessments and assisting patients to choose therapeutic diets are nursing actions that require higher level nursing education and scope of practice and would be delegated to licensed practical/vocational nurses (LPNs/LVNs) or RNs.

The nurse is caring for a patient who has an arterial catheter in the left radial artery for arterial pressure-based cardiac output (APCO) monitoring. Which information obtained by the nurse requires a report to the health care provider? a. The patient has a positive Allen test result. b. There is redness at the catheter insertion site. c. The mean arterial pressure (MAP) is 86 mm Hg. d. The dicrotic notch is visible in the arterial waveform.

ANS: B Redness at the catheter insertion site indicates possible infection. The Allen test is performed before arterial line insertion, and a positive test result indicates normal ulnar artery perfusion. A MAP of 86 mm Hg is normal, and the dicrotic notch is normally present on the arterial waveform. DIF: Cognitive Level: Apply (application)

Which blood gas result will the nurse expect to observe in a patient with respiratory alkalosis? a. pH 7.60, PaCO2 40 mm Hg, HCO3- 30 mEq/L b. pH 7.53, PaCO2 30 mm Hg, HCO3- 24 mEq/L c. pH 7.35, PaCO2 35 mm Hg, HCO3- 26 mEq/L d. pH 7.25, PaCO2 48 mm Hg, HCO3- 23 mEq/L

ANS: B Respiratory alkalosis should show an alkalotic pH and decreased CO2 (respiratory) values, with a normal HCO3-. In this case, pH 7.53 is alkaline (normal = 7.35 to 7.45), PaCO2 is 30 (normal 35 to 45 mm Hg), and HCO3- is 24 (normal = 22 to 26 mEq/L). A result of pH 7.60, PaCO2 40 mm Hg, HCO3- 30 mEq/L is metabolic alkalosis. pH 7.35, PaCO2 35 mm Hg, HCO3- 26 mEq/L is within normal limits. pH 7.25, PaCO2 48 mm Hg, HCO3- 23 mEq/L is respiratory acidosis.

A 2-year-old child is brought into the emergency department after ingesting a medication that causes respiratory depression. For which acid-base imbalance will the nurse most closely monitor this child? a. Respiratory alkalosis b. Respiratory acidosis c. Metabolic acidosis d. Metabolic alkalosis

ANS: B Respiratory depression leads to hypoventilation. Hypoventilation results in retention of CO2 and respiratory acidosis. Respiratory alkalosis would result from hyperventilation, causing a decrease in CO2 levels. Metabolic acid-base imbalance would be a result of kidney dysfunction, vomiting, diarrhea, or other conditions that affect metabolic acids.

The nurse is admitting a patient with a basal skull fracture. The nurse notes ecchymoses around both eyes and clear drainage from the patient's nose. Which admission order should the nurse question? a. Keep the head of bed elevated. b. Insert nasogastric tube to low suction. c. Turn patient side to side every 2 hours. d. Apply cold packs intermittently to face.

ANS: B Rhinorrhea may indicate a dural tear with cerebrospinal fluid (CSF) leakage. Insertion of a nasogastric tube will increase the risk for infections such as meningitis. Turning the patient, elevating the head, and applying cold packs are appropriate orders

A patient with Parkinson's disease has a nursing diagnosis of impaired physical mobility related to bradykinesia. Which action will the nurse include in the plan of care? a. Instruct the patient in activities that can be done while lying or sitting. b. Suggest that the patient rock from side to side to initiate leg movement. c. Have the patient take small steps in a straight line directly in front of the feet. d. Teach the patient to keep the feet in contact with the floor and slide them forward.

ANS: B Rocking the body from side to side stimulates balance and improves mobility. The patient will be encouraged to continue exercising because this will maintain functional abilities. Maintaining a wide base of support will help with balance. The patient should lift the feet and avoid a shuffling gait.

Which hemodynamic parameter best reflects the effectiveness of drugs that the nurse gives to reduce a patient's left ventricular afterload? a. Mean arterial pressure (MAP) b. Systemic vascular resistance (SVR) c. Pulmonary vascular resistance (PVR) d. Pulmonary artery wedge pressure (PAWP)

ANS: B SVR reflects the resistance to ventricular ejection, or afterload. The other parameters may be monitored but do not reflect afterload as directly. DIF: Cognitive Level: Apply (application)

Which intervention will the nurse include in the plan of care for a patient who has cardiogenic shock? a. Check temperature every 2 hours. b. Monitor breath sounds frequently. c. Maintain patient in supine position. d. Assess skin for flushing and itching.

ANS: B Since pulmonary congestion and dyspnea are characteristics of cardiogenic shock, the nurse should assess the breath sounds frequently. The head of the bed is usually elevated to decrease dyspnea in patients with cardiogenic shock. Elevated temperature and flushing or itching of the skin are not typical of cardiogenic shock.

Which assessment information will be most important for the nurse to report to the health care provider about a patient with acute cholecystitis? a. The patient's urine is bright yellow. b. The patient's stools are tan colored. c. The patient has increased pain after eating. d. The patient complains of chronic heartburn.

ANS: B Tan or gray stools indicate biliary obstruction, which requires rapid intervention to resolve. The other data are not unusual for a patient with this diagnosis, although the nurse would also report the other assessment information to the health care provider.

After shunt procedure, the nurse would monitor the patient's neurologic status by using which test? a. Electroencephalogram b. Glasgow Coma Scale c. National Institutes of Health Stroke Scale d. Monro-Kellie doctrine

ANS: B The GCS gives a standardized numeric score of the neurologic patient assessment. An electroencephalogram is used in diagnosing and localizing the area of seizure origin. This scale is an example of one type of specific tool for nurses to use when assessing a patient following stroke. The Monroe-Kellie doctrine is not an assessment or monitoring strategy; it describes the interrelationship of volume and compliance of the three cranial components, brain tissue, cerebral spinal fluid, and blood.

The nurse is assessing a patient 4 hours after a kidney transplant. Which information is most important to communicate to the health care provider? a. The urine output is 900 to 1100 mL/hr. b. The patient's central venous pressure (CVP) is decreased. c. The patient has a level 7 (0 to 10 point scale) incisional pain. d. The blood urea nitrogen (BUN) and creatinine levels are elevated.

ANS: B The decrease in CVP suggests hypovolemia, which must be rapidly corrected to prevent renal hypoperfusion and acute tubular necrosis. The other information is not unusual in a patient after a transplant

When a 74-year-old patient is seen in the health clinic with new development of a stooped posture, shuffling gait, and pill rolling-type tremor, the nurse will anticipate teaching the patient about a. oral corticosteroids. b. antiparkinsonian drugs. c. magnetic resonance imaging (MRI). d. electroencephalogram (EEG) testing.

ANS: B The diagnosis of Parkinson's is made when two of the three characteristic manifestations of tremor, rigidity, and bradykinesia are present. The confirmation of the diagnosis is made on the basis of improvement when antiparkinsonian drugs are administered. This patient has symptoms of tremor and bradykinesia. The next anticipated step will be treatment with medications. MRI and EEG are not useful in diagnosing Parkinson's disease, and corticosteroid therapy is not used to treat it.

A 62-yr-old female patient has been hospitalized for 4 days with acute kidney injury (AKI) caused by dehydration. Which information will be most important for the nurse to report to the health care provider? a. The creatinine level is 3.0 mg/dL. b. Urine output over an 8-hour period is 2500 mL. c. The blood urea nitrogen (BUN) level is 67 mg/dL. d. The glomerular filtration rate is less than 30 mL/min/1.73 m2.

ANS: B The high urine output indicates a need to increase fluid intake to prevent hypovolemia. The other information is typical of AKI and will not require a change in therapy. DIF: Cognitive Level: Analyze (analysis) REF: 1072 OBJ: Special Questions: Prioritization TOP: Nursing Process: Assessment MSC: NCLEX: Physiological Integrity

Which goal has the highest priority in the plan of care for a 26-yr-old patient who is homeless who was admitted with viral hepatitis who has severe anorexia and fatigue? a. Increase activity level. b. Maintain adequate nutrition. c. Establish a stable environment. d. Identify source of hepatitis exposure.

ANS: B The highest priority outcome is to maintain nutrition because adequate nutrition is needed for hepatocyte regeneration. Finding a home for the patient and identifying the source of the infection would be appropriate activities, but they do not have as high a priority as ensuring adequate nutrition. Although the patient's activity level will be gradually increased, rest is indicated during the acute phase of hepatitis.

A 25-yr-old male patient has been admitted with a severe crushing injury after an industrial accident. Which laboratory result will be most important to report to the health care provider? a. Serum creatinine level of 2.1 mg/dL b. Serum potassium level of 6.5 mEq/L c. White blood cell count of 11,500/µL d. Blood urea nitrogen (BUN) of 56 mg/dL

ANS: B The hyperkalemia associated with crushing injuries may cause cardiac arrest and should be treated immediately. The nurse also will report the other laboratory values, but abnormalities in these are not immediately life threatening. DIF: Cognitive Level: Analyze (analysis) REF: 1072 OBJ: Special Questions: Prioritization TOP: Nursing Process: Assessment MSC: NCLEX: Physiological Integrity

The nurse monitors a patient after chest tube placement for a hemopneumothorax. The nurse is most concerned if which assessment finding is observed? a. A large air leak in the water-seal chamber b. 400 mL of blood in the collection chamber c. Complaint of pain with each deep inspiration d. Subcutaneous emphysema at the insertion site

ANS: B The large amount of blood may indicate that the patient is in danger of developing hypovolemic shock. An air leak would be expected immediately after chest tube placement for a pneumothorax. Initially, brisk bubbling of air occurs in this chamber when a pneumothorax is evacuated. The pain should be treated but is not as urgent a concern as the possibility of continued hemorrhage. Subcutaneous emphysema should be monitored but is not unusual in a patient with pneumothorax. A small amount of subcutaneous air is harmless and will be reabsorbed.

A patient is being treated for bleeding esophageal varices with balloon tamponade. Which nursing action will be included in the plan of care? a. Instruct the patient to cough every hour. b. Monitor the patient for shortness of breath. c. Verify the position of the balloon every 4 hours. d. Deflate the gastric balloon if the patient reports nausea.

ANS: B The most common complication of balloon tamponade is aspiration pneumonia. In addition, if the gastric balloon ruptures, the esophageal balloon may slip upward and occlude the airway. Coughing increases the pressure on the varices and increases the risk for bleeding. Balloon position is verified after insertion and does not require further verification. Balloons may be deflated briefly every 8 to 12 hours to avoid tissue necrosis, but if only the gastric balloon is deflated, the esophageal balloon may occlude the airway. Balloons are not deflated for nausea.

An older adult patient who is malnourished presents to the emergency department with a serum protein level of 5.2 g/dL. The nurse would expect which clinical manifestation? a. Pallor b. Edema c. Confusion d. Restlessness

ANS: B The normal range for total protein is 6.4 to 8.3 g/dL. Low serum protein levels cause a decrease in plasma oncotic pressure and allow fluid to remain in interstitial tissues, causing edema. Confusion, restlessness, and pallor are not associated with low serum protein levels

A high school teacher who has just been diagnosed with epilepsy after having a generalized tonic-clonic seizure tells the nurse, "I cannot teach anymore, it will be too upsetting if I have a seizure at work." Which response by the nurse is best? a. "You might benefit from some psychologic counseling." b. "Epilepsy usually can be well controlled with medications." c. "You will want to contact the Epilepsy Foundation for assistance." d. "The Department of Vocational Rehabilitation can help with work retraining."

ANS: B The nurse should inform the patient that most patients with seizure disorders are controlled with medication. The other information may be necessary if the seizures persist after treatment with antiseizure medications is implemented.

A 74-yr-old patient who is progressing to stage 5 chronic kidney disease asks the nurse, "Do you think I should go on dialysis? Which initial response by the nurse is best? a. "It depends on which type of dialysis you are considering." b. "Tell me more about what you are thinking regarding dialysis." c. "You are the only one who can make the decision about dialysis." d. "Many people your age use dialysis and have a good quality of life."

ANS: B The nurse should initially clarify the patient's concerns and questions about dialysis. The patient is the one responsible for the decision, and many people using dialysis do have good quality of life, but these responses block further assessment of the patient's concerns. Referring to which type of dialysis the patient might use only indirectly responds to the patient's question. DIF: Cognitive Level: Analyze (analysis) REF: 1091 OBJ: Special Questions: Prioritization TOP: Nursing Process: Assessment MSC: NCLEX: Psychosocial Integrity

A patient with a head injury opens his eyes to verbal stimulation, curses when stimulated, and does not respond to a verbal command to move but attempts to push away a painful stimulus. The nurse records the patient's Glasgow Coma Scale score as a. 9. b. 11 c. 13. d. 15.

ANS: B The patient has scores of 3 for eye opening, 3 for best verbal response, and 5 for best motor response. DIF: Cognitive Level: Apply (application)

A 20-yr-old patient who sustained a T2 spinal cord injury 10 days ago tells the nurse, "I want to be transferred to a hospital where the nurses know what they are doing." Which action by the nurse is appropriate? a. Respond that abusive language will not be tolerated. b. Request that the patient provide input for the plan of care. c. Perform care without responding to the patient's comments. d. Reassure the patient about the competence of the nursing staff.

ANS: B The patient is demonstrating behaviors consistent with the anger phase of the grief process, and the nurse should allow expression of anger and seek the patient's input into care. Expression of anger is appropriate at this stage, and should be accepted by the nurse. Reassurance about the competency of the staff will not be helpful in responding to the patient's concerns. Ignoring the patient's comments will increase the patient's anger and sense of helplessness. DIF: Cognitive Level: Apply (application)

A patient is admitted for a bowel obstruction and has had a nasogastric tube set to low intermittent suction for the past 3 days. Which arterial blood gas values will the nurse expect to observe? A. Respiratory alkalosis B. Metabolic alkalosis C. Metabolic acidosis D. Respiratory acidosis

ANS: B The patient is losing acid from the nasogastric tube so the patient will have metabolic alkalosis. Lung problems will produce respiratory alkalosis or acidosis. Metabolic acidosis will occur when too much acid is in the body like kidney failure.

A patient with new-onset confusion and hyponatremia is being admitted. When making room assignments, the charge nurse should take which action? a. Assign the patient to a semi-private room. b. Assign the patient to a room near the nurse's station. c. Place the patient in a room nearest to the water fountain. d. Place the patient on telemetry to monitor for peaked T waves..

ANS: B The patient should be placed near the nurse's station if confused for the staff to closely monitor the patient. To help improve serum sodium levels, water intake is restricted. Therefore a confused patient should not be placed near a water fountain. Peaked T waves are a sign of hyperkalemia, not hyponatremia. A confused patient could be distracting and disruptive for another patient in a semiprivate room.

When the nurse is developing a rehabilitation plan for a 30-year-old patient with a C6 spinal cord injury, an appropriate goal is that the patient will be able to a. drive a car with powered hand controls. b. push a manual wheelchair on a flat surface. c. turn and reposition independently when in bed. d. transfer independently to and from a wheelchair.

ANS: B The patient with a C6 injury will be able to use the hands to push a wheelchair on flat, smooth surfaces. Because flexion of the thumb and fingers is minimal, the patient will not be able to grasp a wheelchair during transfer, drive a car with powered hand controls, or turn independently in bed

The nurse will explain to the patient who has a T2 spinal cord transection injury that a. use of the shoulders will be limited. b. function of both arms should be retained. c. total loss of respiratory function may occur. d. tachycardia is common with this type of injury.

ANS: B The patient with a T2 injury can expect to retain full motor and sensory function of the arms. Use of only the shoulders is associated with cervical spine injury. Loss of respiratory function occurs with cervical spine injuries. Bradycardia is associated with injuries above the T6 level

Which intervention will be included in the plan of care for a patient with acute kidney injury (AKI) who has a temporary vascular access catheter in the left femoral vein? a. Start continuous pulse oximetry. b. Restrict physical activity to bed rest. c. Restrict the patient's oral protein intake. d. Discontinue the urethral retention catheter.

ANS: B The patient with a femoral vein catheter must be on bed rest to prevent trauma to the vein. Protein intake is likely to be increased when the patient is receiving dialysis. The retention catheter is likely to remain in place because accurate measurement of output will be needed. There is no indication that the patient needs continuous pulse oximetry. DIF: Cognitive Level: Apply (application) REF: 1088 TOP: Nursing Process: Planning MSC: NCLEX: Physiological Integrity

For a patient who had a right hemisphere stroke, the nurse anticipates planning interventions to manage a. impaired physical mobility related to right-sided hemiplegia. b. risk for injury related to denial of deficits and impulsiveness. c. impaired verbal communication related to speech-language deficits. d. ineffective coping related to depression and distress about disability.

ANS: B The patient with right-sided brain damage typically denies any deficits and has poor impulse control, leading to risk for injury when the patient attempts activities such as transferring from a bed to a chair. Right-sided brain damage causes left hemiplegia. Left-sided brain damage typically causes language deficits. Left-sided brain damage is associated with depression and distress about the disability.

Which set of assessment data is consistent for a patient with severe infection that could lead to system failure? a. Blood pressure (BP) 92/52, pulse (P) 56 beats/min, respiratory rate (RR) 10 breaths/min, urine output 1200 mL in past 24 hours b. BP 90/48, P 112 beats/min, RR 26 breaths/min, urine output 240 mL in past 24 hours c. BP 112/64, P 98 beats/min, RR 18 breaths/min, urine output 2400 mL in past 24 hours d. BP 152/90, P 52 beats/min, RR 12 breaths/min, urine output 4800 mL in past 24 hours

ANS: B The patient with severe infection presents with low BP and compensating elevations in pulse to move lower volumes of blood more rapidly and respiration to increase access to oxygen. Urine output decreases to counteract the decreased circulating blood volume and hypotension. These vital signs are all too low: Blood pressure (BP) 92/52, pulse (P) 56 beats/min, respiratory rate (RR) 10 breaths/min, urine output 1200 mL in past 24 hours. The patient with severe infection does have a low BP, but the pulse and respiratory rate increase to compensate. This data is all within normal limits: BP 112/64, P 98 beats/min, RR 18 breaths/min, urine output 2400 mL in past 24 hours. This set of data reflects an elevated BP with a decrease in pulse and respiratory rates along with normal urine output: BP 152/90, P 52 beats/min, RR 12 breaths/min, urine output 4800 mL in past 24 hours. None of these is a typical response to severe infection.

A patient whose heart monitor shows sinus tachycardia, rate 132, is apneic, and has no palpable pulses. What action should the nurse take next? a. Perform synchronized cardioversion. b. Start cardiopulmonary resuscitation (CPR). c. Give atropine per agency dysrhythmia protocol. d. Provide supplemental O2 via non-rebreather mask

ANS: B The patient's clinical manifestations indicate pulseless electrical activity, and the nurse should immediately start CPR. The other actions would not be of benefit to this patient. DIF: Cognitive Level: Apply (application)

During routine hemodialysis, a patient complains of nausea and dizziness. Which action should the nurse take first? a. Slow down the rate of dialysis. b. Check the blood pressure (BP). c. Review the hematocrit (Hct) level. d. Give prescribed PRN antiemetic drugs.

ANS: B The patient's complaints of nausea and dizziness suggest hypotension, so the initial action should be to check the BP. The other actions may also be appropriate based on the blood pressure obtained. DIF: Cognitive Level: Analyze (analysis) REF: 1090 OBJ: Special Questions: Prioritization TOP: Nursing Process: Implementation MSC: NCLEX: Physiological Integrity

A patient is admitted to the burn unit with burns to the head, face, and hands. Initially, wheezes are heard, but an hour later, the lung sounds are decreased and no wheezes are audible. What is the best action for the nurse to take? a. Encourage the patient to cough and auscultate the lungs again. b. Notify the health care provider and prepare for endotracheal intubation. c. Document the results and continue to monitor the patient's respiratory rate. d. Reposition the patient in high-Fowler's position and reassess breath sounds.

ANS: B The patient's history and clinical manifestations suggest airway edema and the health care provider should be notified immediately, so that intubation can be done rapidly. Placing the patient in a more upright position or having the patient cough will not address the problem of airway edema. Continuing to monitor is inappropriate because immediate action should occur

A patient with respiratory failure has a respiratory rate of 6 breaths/min and an oxygen saturation (SpO2) of 88%. The patient is increasingly lethargic. Which intervention will the nurse anticipate? a. Administration of 100% O2 by non-rebreather mask b. Endotracheal intubation and positive pressure ventilation c. Insertion of a mini-tracheostomy with frequent suctioning d. Initiation of continuous positive pressure ventilation (CPAP)

ANS: B The patient's lethargy, low respiratory rate, and SpO2 indicate the need for mechanical ventilation with ventilator-controlled respiratory rate. Giving high-flow O2 will not be helpful because the patient's respiratory rate is so low. Insertion of a mini-tracheostomy will facilitate removal of secretions, but it will not improve the patient's respiratory rate or oxygenation. CPAP requires that the patient initiate an adequate respiratory rate to allow adequate gas exchange. DIF: Cognitive Level: Apply (application)

The nurse reviews the electronic health record for a patient scheduled for a total hip replacement. Which assessment data shown in the accompanying figure increase the patient's risk for respiratory complications after surgery? a. Older age and anemia b. Albumin level and weight loss c. Recent arthroscopic procedure d. Confusion and disorientation to time

ANS: B The patient's recent weight loss and low protein stores indicate possible muscle weakness, which make it more difficult for an older patient to recover from the effects of general anesthesia and immobility associated with the hip surgery. The other information will also be noted by the nurse but does not place the patient at higher risk for respiratory failure.

A patient who had a transverse colectomy for diverticulosis 18 hours ago has nasogastric suction and is complaining of anxiety and incisional pain. The patient's respiratory rate is 32 breaths/minute and the arterial blood gases (ABGs) indicate respiratory alkalosis. Which action should the nurse take first? a. Discontinue the nasogastric suction. b. Give the patient the PRN IV morphine sulfate 4 mg. c. Notify the health care provider about the ABG results. d. Teach the patient how to take slow, deep breaths when anxious.

ANS: B The patient's respiratory alkalosis is caused by the increased respiratory rate associated with pain and anxiety. The nurse's first action should be to medicate the patient for pain. Although the nasogastric suction may contribute to the alkalosis, it is not appropriate to discontinue the tube when the patient needs gastric suction. The health care provider may be notified about the ABGs but is likely to instruct the nurse to medicate for pain. The patient will not be able to take slow, deep breaths when experiencing pain. DIF: Cognitive Level: Analyze (analysis) REF: 288 OBJ: Special Questions: Prioritization TOP: Nursing Process: Implementation MSC: NCLEX: Physiological Integrity

A patient admitted with an abrupt onset of jaundice and nausea has abnormal liver function studies but serologic testing is negative for viral causes of hepatitis. Which question by the nurse is appropriate? a. "Do you have a history of IV drug use?" b. "Do you use any over-the-counter drugs?" c. "Have you used corticosteroids for any reason?" d. "Have you recently traveled to a foreign country?"

ANS: B The patient's symptoms, lack of antibodies for hepatitis, and the abrupt onset of symptoms suggest toxic hepatitis, which can be caused by commonly used over-the-counter drugs such as acetaminophen (Tylenol). Travel to a foreign country and a history of IV drug use are risk factors for viral hepatitis. Corticosteroid use does not cause the symptoms listed.

The nurse is planning care for a patient with severe heart failure who has developed elevated blood urea nitrogen (BUN) and creatinine levels. The primary treatment goal in the plan will be a. augmenting fluid volume. b. maintaining cardiac output. c. diluting nephrotoxic substances. d. preventing systemic hypertension.

ANS: B The primary goal of treatment for acute kidney injury (AKI) is to eliminate the cause and provide supportive care while the kidneys recover. Because this patient's heart failure is causing AKI, the care will be directed toward treatment of the heart failure. For renal failure caused by hypertension, hypovolemia, or nephrotoxins, the other responses would be correct. DIF: Cognitive Level: Apply (application) REF: 1073 TOP: Nursing Process: Planning MSC: NCLEX: Physiological Integrity

To detect possible complications in a patient with severe cirrhosis who has bleeding esophageal varices, it is most important for the nurse to monitor a. bilirubin levels. c. potassium levels. b. ammonia levels. d. prothrombin time.

ANS: B The protein in the blood in the gastrointestinal tract will be absorbed and may result in an increase in the ammonia level because the liver cannot metabolize protein very well. The prothrombin time, bilirubin, and potassium levels should also be monitored, but they will not be affected by the bleeding episode.

Which cerebrospinal fluid analysis result should the nurse recognize as abnormal and communicate to the health care provider? a. Specific gravity of 1.007 b. Protein of 65 mg/dL (0.65 g/L) c. Glucose of 45 mg/dL (1.7 mmol/L) d. White blood cell (WBC) count of 4 cells/μL

ANS: B The protein level is high. The specific gravity, WBCs, and glucose values are normal. DIF: Cognitive Level: Understand (comprehension) REF: 1298 TOP: Nursing Process: Implementation MSC: NCLEX: Physiological Integrity

The nurse is caring for a patient who has a central venous access device (CVAD). Which action by the nurse is appropriate? a. Avoid using friction when cleaning around the CVAD insertion site. b. Use the push-pause method to flush the CVAD after giving medications. c. Obtain an order from the health care provider to change CVAD dressing. d. Position the patient's face toward the CVAD during injection cap changes.

ANS: B The push-pause enhances the removal of debris from the CVAD lumen and decreases the risk for clotting. To decrease infection risk, friction should be used when cleaning the CVAD insertion site. The dressing should be changed whenever it becomes damp, loose, or visibly soiled. The patient should turn away from the CVAD during cap changes

A nurse is caring for a patient who has burns of the ears, head, neck, and right arm and hand. The nurse should place the patient in which position? a. Place the right arm and hand flexed in a position of comfort. b. Elevate the right arm and hand on pillows and extend the fingers. c. Assist the patient to a supine position with a small pillow under the head. d. Position the patient in a side-lying position with rolled towel under the neck.

ANS: B The right hand and arm should be elevated to reduce swelling and the fingers extended to avoid flexion contractures (even though this position may not be comfortable for the patient). The patient with burns of the ears should not use a pillow for the head because this will put pressure on the ears, and the pillow may stick to the ears. Patients with neck burns should not use a pillow because the head should be maintained in an extended position in order to avoid contractures.

A patient is admitted to the emergency department with severe fatigue and confusion. Laboratory studies are done. Which laboratory value will require the most immediate action by the nurse? a. Arterial blood pH is 7.32. b. Serum calcium is 18 mg/dL. c. Serum potassium is 5.1 mEq/L. d. Arterial oxygen saturation is 91%.

ANS: B The serum calcium is well above the normal level and puts the patient at risk for cardiac dysrhythmias. The nurse should initiate cardiac monitoring and notify the health care provider. The potassium, oxygen saturation, and pH are also abnormal, and the nurse should notify the health care provider about these values as well, but they are not immediately life threatening

The home health nurse is caring for an 81-year-old who had a stroke 2 months ago. Based on information shown in the accompanying figure from the history, physical assessment, and physical therapy/occupational therapy, which nursing diagnosis is the highest priority for this patient? a. Impaired transfer ability b. Risk for caregiver role strain c. Ineffective health maintenance d. Risk for unstable blood glucose level

ANS: B The spouse's household and patient care responsibilities, in combination with chronic illnesses, indicate a high risk for caregiver role strain. The nurse should further assess the situation and take appropriate actions. The data about the control of the patient's diabetes indicates that ineffective health maintenance and risk for unstable blood glucose are not priority concerns at this time. Because the patient is able to ambulate with a cane, the nursing diagnosis of impaired transfer ability is not supported. DIF: Cognitive Level: Analyze (analysis) REF: 1409 OBJ: Special Questions: Prioritization TOP: Nursing Process: Diagnosis MSC: NCLEX: Psychosocial Integrity

A nurse is caring for a patient with ARDS who is being treated with mechanical ventilation and high levels of positive end-expiratory pressure (PEEP). Which assessment finding by the nurse indicates that the PEEP may need to be reduced? a. The patient's PaO2 is 50 mm Hg and the SaO2 is 88%. b. The patient has subcutaneous emphysema on the upper thorax. c. The patient has bronchial breath sounds in both the lung fields. d. The patient has a first-degree atrioventricular heart block with a rate of 58 beats/min.

ANS: B The subcutaneous emphysema indicates barotrauma caused by positive pressure ventilation and PEEP. Bradycardia, hypoxemia, and bronchial breath sounds are all concerns and will need to be addressed, but they are not specific indications that PEEP should be reduced.

A young adult patient who is in the rehabilitation phase 6 months after a severe face and neck burn tells the nurse, "I'm sorry that I'm still alive. My life will never be normal again." Which response by the nurse is best? a. "Most people recover after a burn and feel satisfied with their lives." b. "It's true that your life may be different. What concerns you the most?" c. "It is really too early to know how much your life will be changed by the burn." d. "Why do you feel that way? You will be able to adapt as your recovery progresses."

ANS: B This response acknowledges the patient's feelings and asks for more assessment data that will help in developing an appropriate plan of care to assist the patient with the emotional response to the burn injury. The other statements are accurate, but do not acknowledge the anxiety and depression that the patient is expressing.

The patient has severe hyperthyroidism and will have surgery tomorrow. What assessment is most important for the nurse to assess in order to detect development of the acid-base imbalance for which the patient has highest risk? a. Urine output and color b. Level of consciousness c. Heart rate and blood pressure d. Lung sounds in lung bases

ANS: B Thyroid hormone increases metabolic rate, causing a patient with severe hyperthyroidism to have high risk of metabolic acidosis from increased production of metabolic acids. Metabolic acidosis decreases level of consciousness. Changes in urine output, urine color, and lung sounds are not signs of metabolic acidosis. Although metabolic acidosis often causes tachycardia, many other factors influence heart rate and blood pressure, including thyroid hormone.

Which nursing action can the registered nurse (RN) delegate to experienced unlicensed assistive personnel (UAP) working as telemetry technicians on the cardiac care unit? a. Decide whether a patient's heart rate of 116 requires urgent treatment. b. Observe heart rhythms for multiple patients who have telemetry monitoring. c. Monitor a patient's level of consciousness during synchronized cardioversion. d. Select the best lead for monitoring a patient admitted with acute coronary syndrome.

ANS: B UAP serving as telemetry technicians can monitor heart rhythms for individuals or groups of patients. Nursing actions such as assessment and choice of the most appropriate lead based on ST segment elevation location require RN-level education and scope of practice. DIF: Cognitive Level: Apply (application)

Which vaccine is now recommended for the immunization of all newborns? a. Hepatitis A vaccine b. Hepatitis B vaccine c. Hepatitis C vaccine d. Hepatitis A, B, and C vaccines

ANS: B Universal vaccination for hepatitis B is now recommended for all newborns. A vaccine is available for hepatitis A, but it is not yet universally recommended. No vaccine is currently available for hepatitis C. Only hepatitis B vaccine is recommended for newborns.

While the patient's full-thickness burn wounds to the face are exposed, what is the best nursing action to prevent cross contamination? a. Use sterile gloves when removing old dressings. b. Wear gowns, caps, masks, and gloves during all care of the patient. c. Administer IV antibiotics to prevent bacterial colonization of wounds. d. Turn the room temperature up to at least 70° F (20° C) during dressing changes.

ANS: B Use of gowns, caps, masks, and gloves during all patient care will decrease the possibility of wound contamination for a patient whose burns are not covered. When removing contaminated dressings and washing the dirty wound, use nonsterile, disposable gloves. The room temperature should be kept at approximately 85° F for patients with open burn wounds to prevent shivering. Systemic antibiotics are not well absorbed into deep burns because of the lack of circulation.

The priority nursing assessment for a patient being admitted with a brainstem infarction is a. pupil reaction. b. respiratory rate. c. reflex reaction time. d. level of consciousness.

ANS: B Vital centers that control respiration are located in the medulla and part of the brainstem, and will require priority assessments because changes in respiratory function may be life threatening. The other information will also be obtained by the nurse but is not as urgent. DIF: Cognitive Level: Apply (application) REF: 1297 OBJ: Special Questions: Prioritization TOP: Nursing Process: Assessment MSC: NCLEX: Physiological Integrity

The nurse is evaluating a child who is taking digoxin for her cardiac condition. The nurse is cognizant that a common sign of digoxin toxicity is: a. Seizures. b. Vomiting. c. Bradypnea d. Tachycardia.

ANS: B Vomiting is a common sign of digoxin toxicity. Seizures are not associated with digoxin toxicity. The child will have a slower heart rate, not respiratory rate. PTS: 1 DIF: Cognitive Level: Comprehension REF: 1335 OBJ: Nursing Process: Implementation MSC: Client Needs: Physiologic Integrity

Which action should the nurse perform when preparing a patient with supraventricular tachycardia for cardioversion who is alert and has a blood pressure of 110/66 mm Hg? a. Turn the synchronizer switch to the "off" position. b. Give a sedative before cardioversion is implemented. c. Set the defibrillator/cardioverter energy to 360 joules. d. Provide assisted ventilations with a bag-valve-mask device.

ANS: B When a patient has a nonemergency cardioversion, sedation is used just before the procedure. The synchronizer switch is turned "on" for cardioversion. The initial level of joules for cardioversion is low (e.g., 50). Assisted ventilations are not indicated for this patient

A patient with acute shortness of breath is admitted to the hospital. Which action should the nurse take during the initial assessment of the patient? a. Ask the patient to lie down to complete a full physical assessment. b. Briefly ask specific questions about this episode of respiratory distress. c. Complete the admission database to check for allergies before treatment. d. Delay the physical assessment to first complete pulmonary function tests.

ANS: B When a patient has severe respiratory distress, only information pertinent to the current episode is obtained, and a more thorough assessment is deferred until later. Obtaining a comprehensive health history or full physical examination is unnecessary until the acute distress has resolved. Brief questioning and a focused physical assessment should be done rapidly to help determine the cause of the distress and suggest treatment. Checking for allergies is important, but it is not appropriate to complete the entire admission database at this time. The initial respiratory assessment must be completed before any diagnostic tests or interventions can be ordered.

After receiving 2 L of normal saline, the central venous pressure for a patient who has septic shock is 10 mm Hg, but the blood pressure is still 82/40 mm Hg. The nurse will anticipate an order for a. nitroglycerine (Tridil). b. norepinephrine (Levophed). c. sodium nitroprusside (Nipride). d. methylprednisolone (Solu-Medrol).

ANS: B When fluid resuscitation is unsuccessful, vasopressor drugs are administered to increase the systemic vascular resistance (SVR) and blood pressure, and improve tissue perfusion. Nitroglycerin would decrease the preload and further drop cardiac output and BP. Methylprednisolone (Solu-Medrol) is considered if blood pressure does not respond first to fluids and vasopressors. Nitroprusside is an arterial vasodilator and would further decrease SVR.

A patient who has had progressive chronic kidney disease (CKD) for several years has just begun regular hemodialysis. Which information about diet will the nurse include in patient teaching? a. Increased calories are needed because glucose is lost during hemodialysis. b. More protein is allowed because urea and creatinine are removed by dialysis. c. Dietary potassium is not restricted because the level is normalized by dialysis. d. Unlimited fluids are allowed because retained fluid is removed during dialysis.

ANS: B When the patient is started on dialysis and nitrogenous wastes are removed, more protein in the diet is encouraged. Fluids are still restricted to avoid excessive weight gain and complications such as shortness of breath. Glucose is not lost during hemodialysis. Sodium and potassium intake continues to be restricted to avoid the complications associated with high levels of these electrolytes. DIF: Cognitive Level: Apply (application) REF: 1087 TOP: Nursing Process: Implementation MSC: NCLEX: Physiological Integrity

When assessing a patient who spilled hot oil on the right leg and foot, the nurse notes that the skin is dry, pale, hard skin. The patient states that the burn is not painful. What term would the nurse use to document the burn depth? a. First-degree skin destruction b. Full-thickness skin destruction c. Deep partial-thickness skin destruction d. Superficial partial-thickness skin destruction

ANS: B With full-thickness skin destruction, the appearance is pale and dry or leathery and the area is painless because of the associated nerve destruction. Erythema, swelling, and blisters point to a deep partial-thickness burn. With superficial partial-thickness burns, the area is red, but no blisters are present. First-degree burns exhibit erythema, blanching, and pain.

Which assessments will the nurse make to monitor a patient's cerebellar function (select all that apply)? a. Assess for graphesthesia. b. Observe arm swing with gait. c. Perform the finger-to-nose test. d. Check ability to push against resistance. e. Determine ability to sense heat and cold.

ANS: B, C The cerebellum is responsible for coordination and is assessed by looking at the patient's gait and the finger-to-nose test. The other assessments will be used for other parts of the neurologic assessment. DIF: Cognitive Level: Analyze (analysis) REF: 1348 TOP: Nursing Process: Assessment MSC: NCLEX: Health Promotion and Maintenance

Which statements said by patients indicate that the nurse's teaching regarding prevention of acid-base imbalances is successful? (Select all that apply.) a. "Baking soda is an effective inexpensive antacid." b. "I shall take my insulin on time every day." c. "My aspirin is on a high shelf away from children." d. "I have reliable transportation to dialysis sessions." e. "Fasting is a great way to lose weight rapidly."

ANS: B, C, D Taking insulin as prescribed helps prevent diabetic ketoacidosis. Safeguarding aspirin from children prevents metabolic acidosis from increased acid intake. Regular dialysis reduces the risk of metabolic acidosis from decreased renal excretion of metabolic acid. Baking soda is sodium bicarbonate and should not be used as an antacid due to the risk of metabolic alkalosis. Fasting without carbohydrate intake is a risk factor for starvation ketoacidosis.

The patient is hyperventilating from anxiety and abdominal pain. Which assessment findings should the nurse attribute to respiratory alkalosis? (Select all that apply.) a. Skin pale and cold b. Tingling of fingertips c. Heart rate of 102 d. Numbness around mouth e. Cramping in feet

ANS: B, D, E Hyperventilation is a risk factor for respiratory alkalosis. Respiratory alkalosis can cause perioral and digital paresthesias and pedal spasms. Pallor, cold skin, and tachycardia are characteristic of activation of the sympathetic nervous system, not respiratory alkalosis.

The nurse is monitoring an infant for signs of increased intracranial pressure (ICP). Which are late signs of increased ICP in an infant? (Select all that apply.) a. Tachycardia b. Alteration in pupil size and reactivity c. Increased motor response d. Extension or flexion posturing e. Cheyne-Stokes respirations

ANS: B, D, E Late signs of ICP in an infant or child include bradycardia, alteration in pupil size and reactivity, decreased motor response, extension or flexion posturing, and Cheyne-Stokes respirations.

A hospitalized client with chronic renal failure has returned to the nursing unit after a hemodialysis treatment. Which parameters contained in the predialysis and postdialysis documentation does the nurse utilize to determine if the procedure was effective? A. Potassium and creatinine levels B. Blood pressure and weight C. Weight and BUN D. BUN and creatinine levels

ANS: B. Rationale: After hemodialysis, the client's vital signs are monitored to determine whether the client is remaining hemodynamically stable and for comparison with predialysis measurements. The client's blood pressure and weight are expected to be reduced as a result of fluid removal. Laboratory studies are performed as per protocol but are not necessarily done after the hemodialysis treatment has ended.

A patient has cellulitis on the right forearm. The nurse would anticipate orders to administer medications to eradicate which organism? a. Candida albicans b. Group A beta-hemolytic streptococci c. Staphylococcus aureus d. E. Coli

ANS: C Staphylococcus aureus is the usual cause of cellulitis, although other pathogens may be responsible. A small abrasion or lesion can provide a portal for opportunistic or pathogenic infectious organisms to infect deeper tissues.

After reviewing the information shown in the accompanying figure for a patient with pneumonia and sepsis, which information is most important to report to the health care provider? a. Temperature and IV site appearance b. Oxygen saturation and breath sounds c. Platelet count and presence of petechiae d. Blood pressure, pulse rate, respiratory rate.

ANS: C The low platelet count and presence of petechiae suggest that the patient may have disseminated intravascular coagulation and that multiple organ dysfunction syndrome (MODS) is developing. The other information will also be discussed with the health care provider but does not indicate that the patients condition is deteriorating or that a change in therapy is needed immediately.

The nurse notes that a patient has incisional pain, a poor cough effort, and scattered rhonchi after a thoracotomy. Which action should the nurse take first? a. Assist the patient to sit upright in a chair. b. Splint the patient's chest during coughing. c. Medicate the patient with prescribed morphine. d. Observe the patient use the incentive spirometer.

ANS: C A major reason for atelectasis and poor airway clearance in patients after chest surgery is incisional pain (which increases with deep breathing and coughing). The first action by the nurse should be to medicate the patient to minimize incisional pain. The other actions are all appropriate ways to improve airway clearance but should be done after the morphine is given

A patient with burns covering 40% total body surface area (TBSA) is in the acute phase of burn treatment. Which snack would be best for the nurse to offer to this patient? a. Bananas b. Orange gelatin c. Vanilla milkshake d. Whole grain bagel

ANS: C A patient with a burn injury needs high protein and calorie food intake, and the milkshake is the highest in these nutrients. The other choices are not as nutrient-dense as the milkshake. Gelatin is likely high in sugar. The bagel is a good carbohydrate choice, but low in protein. Bananas are a good source of potassium, but are not high in protein and calories.

The emergency department (ED) nurse receives report that a patient involved in a motor vehicle crash is being transported to the facility with an estimated arrival in 1 minute. In preparation for the patient's arrival, the nurse will obtain a. hypothermia blanket. b. lactated Ringer's solution. c. two 14-gauge IV catheters. d. dopamine (Intropin) infusion.

ANS: C A patient with multiple trauma may require fluid resuscitation to prevent or treat hypovolemic shock, so the nurse will anticipate the need for 2 large bore IV lines to administer normal saline. Lactated Ringer's solution should be used cautiously and will not be ordered until the patient has been assessed for possible liver abnormalities. Vasopressor infusion is not used as the initial therapy for hypovolemic shock. Patients in shock need to be kept warm not cool.

A patient is being admitted with a possible stroke. Which information from the assessment indicates that the nurse should consult with the health care provider before giving the prescribed aspirin? a. The patient has dysphasia. b. The patient has atrial fibrillation. c. The patient reports that symptoms began with a severe headache. d. The patient has a history of brief episodes of right-sided hemiplegia.

ANS: C A sudden onset headache is typical of a subarachnoid hemorrhage, and aspirin is contraindicated. Atrial fibrillation, dysphasia, and transient ischemic attack are not contraindications to aspirin use.

The nurse is assessing a patient before hanging an IV solution of 0.9% NaCl with KCl in it. Which assessment finding should cause the nurse to hold the IV solution and contact the physician? a. Weight gain of 2 pounds since last week b. Dry mucous membranes and skin tenting c. Urine output 8 mL/hr d. Blood pressure 98/58

ANS: C Administering IV potassium to a patient who has oliguria is not safe, because potassium intake faster than potassium output can cause hyperkalemia with dangerous cardiac dysrhythmias. Dry mucous membranes, skin tenting, and blood pressure 98/58 are consistent with the need for IV 0.9% NaCl. Weight gain of 2 pounds in a week does not necessarily indicate fluid overload, because it can be from increased nutritional intake. Only an overnight weight gain indicates a fluid gain.

The nurse is assessing a patient before hanging an IV solution of 0.9% NaCl with KCl in it. Which assessment finding should cause the nurse to hold the IV solution and contact the physician? a. Weight gain of 2 pounds since last week b. Dry mucous membranes and skin tenting c. Urine output 8 mL/hr d. Blood pressure 98/58

ANS: C Administering IV potassium to a patient who has oliguria is not safe, because potassium intake faster than potassium output can cause hyperkalemia with dangerous cardiac dysrhythmias. Dry mucous membranes, skin tenting, and blood pressure 98/58 are consistent with the need for IV 0.9% NaCl. Weight gain of 2 pounds in a week does not necessarily indicate fluid overload, because it can be from increased nutritional intake. Only an overnight weight gain indicates a fluid gain.

The nurse is caring for a patient who has been experiencing stroke symptoms for 60 minutes. Which action can the nurse delegate to a licensed practical/vocational nurse (LPN/LVN)? a. Assess the patient's gag and cough reflexes. b. Determine when the stroke symptoms began. c. Administer the prescribed short-acting insulin. d. Infuse the prescribed IV metoprolol (Lopressor).

ANS: C Administration of subcutaneous medications is included in LPN/LVN education and scope of practice. The other actions require more education and scope of practice and should be done by the registered nurse (RN).

A patient with extensive electrical burn injuries is admitted to the emergency department. Which prescribed intervention should the nurse implement first? a. Assess oral temperature. b. Check a potassium level. c. Place on cardiac monitor. d. Assess for pain at contact points.

ANS: C After an electrical burn, the patient is at risk for fatal dysrhythmias and should be placed on a cardiac monitor. Assessing the oral temperature is not as important as assessing for cardiac dysrhythmias. Checking the potassium level is important. However, it will take time before the laboratory results are back. The first intervention is to place the patient on a cardiac monitor and assess for dysrhythmias, so that they can be treated if occurring. A decreased or increased potassium level will alert the nurse to the possibility of dysrhythmias. The cardiac monitor will alert the nurse immediately of any dysrhythmias. Assessing for pain is important, but the patient can endure pain until the cardiac monitor is attached. Cardiac dysrhythmias can be lethal.

A child is brought to the emergency department experiencing an anaphylactic reaction to a bee sting. While an airway is being established, what medication should the nurse prepare for immediate administration? a. Diphenhydramine (Benadryl) b. Dopamine c. Epinephrine d. Calcium chloride

ANS: C After the first priority of establishing an airway, epinephrine is the drug of choice. Benadryl is not a strong enough antihistamine for this severe a reaction. Dopamine and calcium chloride are not appropriate drugs for this type of reaction.

During change-of-shift report on a medical unit, the nurse learns that a patient with aspiration pneumonia who was admitted with respiratory distress has become increasingly agitated. Which action should the nurse take first? a. Give the prescribed PRN sedative drug. b. Offer reassurance and reorient the patient. c. Use pulse oximetry to check the oxygen saturation. d. Notify the health care provider about the patient's status.

ANS: C Agitation may be an early indicator of hypoxemia. The other actions may also be appropriate, depending on the findings about O2 saturation.

A patient has just arrived in the emergency department after an electrical burn from exposure to a high-voltage current. What is the priority nursing assessment? a. Oral temperature b. Peripheral pulses c. Extremity movement d. Pupil reaction to light

ANS: C All patients with electrical burns should be considered at risk for cervical spine injury, and assessments of extremity movement will provide baseline data. The other assessment data are also necessary but not as essential as determining the cervical spine status

A 33-year-old patient with a T4 spinal cord injury asks the nurse whether he will be able to be sexually active. Which initial response by the nurse is best? a. Reflex erections frequently occur, but orgasm may not be possible. b. Sildenafil (Viagra) is used by many patients with spinal cord injury. c. Multiple options are available to maintain sexuality after spinal cord injury. d. Penile injection, prostheses, or vacuum suction devices are possible options.

ANS: C Although sexuality will be changed by the patient's spinal cord injury, there are options for expression of sexuality and for fertility. The other information also is correct, but the choices will depend on the degrees of injury and the patient's individual feelings about sexuality.

A patient in metabolic alkalosis is admitted to the emergency department, and pulse oximetry (SpO2) indicates that the O2 saturation is 94%. Which action should the nurse take next? a. Administer bicarbonate. b. Complete a head-to-toe assessment. c. Place the patient on high-flow oxygen. d. Obtain repeat arterial blood gases (ABGs).

ANS: C Although the O2 saturation is adequate, the left shift in the oxyhemoglobin dissociation curve will decrease the amount of oxygen delivered to tissues, so high oxygen concentrations should be given. Bicarbonate would worsen the patient's condition. A head-to-toe assessment and repeat ABGs may be implemented. However, the priority intervention is to give high-flow oxygen.

Which laboratory test result will the nurse monitor when evaluating the effects of therapy for a patient who has acute pancreatitis? a. Calcium b. Bilirubin c. Amylase d. Potassium

ANS: C Amylase is elevated in acute pancreatitis. Although changes in the other values may occur, they would not be useful in evaluating whether the prescribed therapies have been effective.

Which diagnostic test will provide the nurse with the most specific information to evaluate the effectiveness of interventions for a patient with ventilatory failure? a. Chest x-ray b. O2 saturation c. Arterial blood gas analysis d. Central venous pressure monitoring

ANS: C Arterial blood gas (ABG) analysis is most useful in this setting because ventilatory failure causes problems with CO2 retention, and ABGs provide information about the PaCO2 and pH. The other tests may also be done to help in assessing oxygenation or determining the cause of the patient's ventilatory failure.

Which assessment finding may indicate that a patient is experiencing adverse effects to a corticosteroid prescribed after kidney transplantation? a. Postural hypotension b. Recurrent tachycardia c. Knee and hip joint pain d. Increased serum creatinine

ANS: C Aseptic necrosis of the weight-bearing joints can occur when patients take corticosteroids over a prolonged period. Increased creatinine level, orthostatic dizziness, and tachycardia are not caused by corticosteroid use. DIF: Cognitive Level: Apply (application) REF: 1096 TOP: Nursing Process: Evaluation MSC: NCLEX: Physiological Integrity

Which nursing action will be included in the plan of care for a patient who has had cerebral angiography? a. Monitor for headache and photophobia. b. Keep patient NPO until gag reflex returns. c. Check pulse and blood pressure frequently. d. Assess orientation to person, place, and time.

ANS: C Because a catheter is inserted into an artery (e.g., the femoral artery) during cerebral angiography, the nurse should assess for bleeding after this procedure that can affect pulse and blood pressure. The other nursing assessments are not needed after angiography. DIF: Cognitive Level: Apply (application) REF: 1310 TOP: Nursing Process: Planning MSC: NCLEX: Physiological Integrity

A patient admitted with possible stroke has been aphasic for 3 hours, and his current blood pressure (BP) is 174/94 mm Hg. Which order by the health care provider should the nurse question? a. Keep head of bed elevated at least 30 degrees. b. Infuse normal saline intravenously at 75 mL/hr. c. Start a labetalol drip to keep BP less than 140/90 mm Hg. d. Administer tissue plasminogen activator (tPA) intravenously per protocol.

ANS: C Because elevated BP may be a protective response to maintain cerebral perfusion, antihypertensive therapy is recommended only if mean arterial pressure (MAP) is greater than 130 mm Hg or systolic pressure is greater than 220 mm Hg. Fluid intake should be 1500 to 2000 mL/day to maintain cerebral blood flow. The head of the bed should be elevated to at least 30 degrees unless the patient has symptoms of poor tissue perfusion. tPA may be administered if the patient meets the other criteria for tPA use.

While the nurse is transporting a patient on a stretcher to the radiology department, the patient begins having a tonic-clonic seizure. Which action should the nurse take? a. Insert an oral airway during the seizure to maintain a patent airway. b. Restrain the patient's arms and legs to prevent injury during the seizure. c. Time and observe and record the details of the seizure and postictal state. d. Avoid touching the patient to prevent further nervous system stimulation.

ANS: C Because the diagnosis and treatment of seizures frequently are based on the description of the seizure, recording the length and details of the seizure is important. Insertion of an oral airway and restraining the patient during the seizure are contraindicated. The nurse may need to move the patient to decrease the risk of injury during the seizure.

A left-handed patient with left-sided hemiplegia has difficulty feeding himself. Which intervention should the nurse include in the plan of care? a. Provide a wide variety of food choices. b. Provide oral care before and after meals. c. Assist the patient to eat with the right hand. d. Teach the patient the "chin-tuck" technique.

ANS: C Because the patient has difficulty feeding himself, the appropriate interventions will focus on teaching the patient to use the right hand for self-feeding. The other interventions are appropriate for patients with other etiologies for the imbalanced nutrition.

The nurse notes thick, white secretions in the endotracheal tube (ET) of a patient who is receiving mechanical ventilation. Which intervention will most directly treat this finding? a. Reposition the patient every 1 to 2 hours. b. Increase suctioning frequency to every hour. c. Add additional water to the patient's enteral feedings. d. Instill 5 mL of sterile saline into the ET before suctioning.

ANS: C Because the patient's secretions are thick, better hydration is indicated. Suctioning every hour without any specific evidence for the need will increase the incidence of mucosal trauma and would not address the etiology of the ineffective airway clearance. Instillation of saline does not liquefy secretions and may decrease the SpO2. Repositioning the patient is appropriate but will not decrease the thickness of secretions. DIF: Cognitive Level: Apply (application)

A patient with cirrhosis and esophageal varices has a new prescription for propranolol (Inderal). Which finding is the best indicator to the nurse that the medication has been effective? a. The patient reports no chest pain. b. Blood pressure is 140/90 mm Hg. c. Stools test negative for occult blood. d. The apical pulse rate is 68 beats/minute.

ANS: C Because the purpose of -blocker therapy for patients with esophageal varices is to decrease the risk for bleeding from esophageal varices, the best indicator of the effectiveness for propranolol is the lack of blood in the stools. Although propranolol is used to treat hypertension, angina, and tachycardia, the purpose for use in this patient is to decrease the risk for bleeding from esophageal varices.

A patient who is on the telemetry unit develops atrial flutter, rate 150, with associated dyspnea and chest pain. Which action that is included in the hospital dysrhythmia protocol should the nurse do first? a. Obtain a 12-lead electrocardiogram (ECG). b. Notify the health care provider of the change in rhythm. c. Give supplemental O2 at 2 to 3 L/min via nasal cannula. d. Assess the patient's vital signs including O2 saturation.

ANS: C Because this patient has dyspnea and chest pain in association with the new rhythm, the nurse's initial actions should be to address the patient's airway, breathing, and circulation (ABC) by starting with O2 administration. The other actions are also important and should be implemented rapidly. DIF: Cognitive Level: Analyze (analysis)

Which finding about a patient who is receiving vasopressin (Pitressin) to treat septic shock is most important for the nurse to communicate to the health care provider? a. The patient's urine output is 18 mL/hr. b. The patient's heart rate is 110 beats/minute. c. The patient is complaining of chest pain. d. The patient's peripheral pulses are weak.

ANS: C Because vasopressin is a potent vasoconstrictor, it may decrease coronary artery perfusion. The other information is consistent with the patient's diagnosis and should be reported to the health care provider but does not indicate a need for a change in therapy.

The nurse is working on a plan of care with her patient which includes turning and positioning and adequate nutrition to help the patient maintain intact skin integrity. The nurse helps the patient to realize that this breaks the chain of infection by eliminating a a. host. b. mode of transmission. c. portal of entry. d. reservoir.

ANS: C Broken or impaired skin creates a portal of entry for pathogens. By maintaining intact tissue, the patient and the nurse have broken the chain of infection by eliminating a portal of entry. Host is incorrect because you are not eliminating the person or organism. Intact tissue does not eliminate the mode of transmission. Skin can still be used to transfer pathogens regardless of it being intact or broken. Intact skin does not eliminate the location for pathogens to live and grow.

A patient who has a right-sided chest tube following a thoracotomy has continuous bubbling in the suction-control chamber of the collection device. Which action by the nurse is most appropriate? a. Document the presence of a large air leak. b. Notify the surgeon of a possible pneumothorax. c. Take no further action with the collection device. d. Adjust the dial on the wall regulator to decrease suction.

ANS: C Continuous bubbling is expected in the suction-control chamber and indicates that the suction-control chamber is connected to suction. An air leak would be detected in the water-seal chamber. There is no evidence of pneumothorax. Increasing or decreasing the vacuum source will not adjust the suction pressure. The amount of suction applied is regulated by the amount of water in this chamber and not by the amount of suction applied to the system.

A patient born in 1955 had hepatitis A infection 1 year ago. According to Centers for Disease Control and Prevention (CDC) guidelines, which action should the nurse include in care when the patient is seen for a routine annual physical examination? a. Start the hepatitis B immunization series. b. Teach the patient about hepatitis A immune globulin. c. Ask whether the patient has been screened for hepatitis C. d. Test for anti-hepatitis-A virus immune globulin M (anti-HAV-IgM).

ANS: C Current CDC guidelines indicate that all patients who were born between 1945 and 1965 should be screened for hepatitis C because many individuals who are positive have not been diagnosed. Although routine hepatitis B immunization is recommended for infants, children, and adolescents, vaccination for hepatitis B is recommended only for adults at risk for blood-borne infections. Because the patient has already had hepatitis A, immunization and anti-HAV IgM levels will not be needed.

Which finding in a patient with a spinal cord tumor requires an immediate report to the health care provider? a. Back pain that increases with coughing b. Depression about the diagnosis of a tumor c. Decreasing sensation and ability to move the legs d. Anxiety about scheduled surgery to remove the tumor

ANS: C Decreasing sensation and leg movement indicates spinal cord compression, an emergency that will require rapid action (such as surgery) to prevent paralysis. The other findings will also require nursing action but are not emergencies

When caring for a patient with a new right-sided homonymous hemianopsia resulting from a stroke, which intervention should the nurse include in the plan of care? a. Apply an eye patch to the right eye. b. Approach the patient from the right side. c. Place needed objects on the patient's left side. d. Teach the patient that the left visual deficit will resolve.

ANS: C During the acute period, the nurse should place objects on the patient's unaffected side. Because there is a visual defect in the right half of each eye, an eye patch is not appropriate. The patient should be approached from the left side. The visual deficit may not resolve, although the patient can learn to compensate for the defect.

The long-term care nurse is evaluating the effectiveness of protein supplements for an older resident who has a low serum total protein level. Which assessment finding indicates that the patient's condition has improved? a. Hematocrit 28% b. Absence of skin tenting c. Decreased peripheral edema d. Blood pressure 110/72 mm Hg

ANS: C Edema is caused by low oncotic pressure in individuals with low serum protein levels. The decrease in edema indicates an improvement in the patient's protein status. Good skin turgor is an indicator of fluid balance, not protein status. A low hematocrit could be caused by poor protein intake. Blood pressure does not provide a useful clinical tool for monitoring protein status

The nurse reviews the patient's complete blood count (CBC) results and notes that the neutrophil levels are elevated, but monocytes are still within normal limits. This indicates _____ inflammatory response. a. chronic b. resolved c. early stage acute d. late stage acute

ANS: C Elevated neutrophils and monocytes within normal limits are findings indicative of early inflammatory response. Neutrophils increase in just a few hours, while it takes the body days to increase the monocyte levels. Chronic inflammation results in varying elevations in WBCs dependent on multiple issues. Elevated neutrophils are not indicative of resolved inflammation. Elevations in monocytes occur later in the inflammatory response.

Which response by the nurse best explains the purpose of ranitidine (Zantac) for a patient admitted with bleeding esophageal varices? a. The medication will reduce the risk for aspiration. b. The medication will inhibit development of gastric ulcers. c. The medication will prevent irritation of the enlarged veins. d. The medication will decrease nausea and improve the appetite.

ANS: C Esophageal varices are dilated submucosal veins. The therapeutic action of H2-receptor blockers in patients with esophageal varices is to prevent irritation and bleeding from the varices caused by reflux of acid gastric contents. Although ranitidine does decrease the risk for peptic ulcers, reduce nausea, and help prevent aspiration pneumonia, these are not the primary purposes for H2-receptor blockade in this patient.

A patient with increased intracranial pressure after a head injury has a ventriculostomy in place. Which action can the nurse delegate to unlicensed assistive personnel (UAP) who regularly work in the intensive care unit? a. Document intracranial pressure every hour. b. Turn and reposition the patient every 2 hours. c. Check capillary blood glucose level every 6 hours. d. Monitor cerebrospinal fluid color and volume hourly.

ANS: C Experienced UAP can obtain capillary blood glucose levels when they have been trained and evaluated in the skill. Monitoring and documentation of cerebrospinal fluid (CSF) color and intracranial pressure (ICP) require registered nurse (RN)-level education and scope of practice. Although repositioning patients is frequently delegated to UAP, repositioning a patient with a ventriculostomy is complex and should be supervised by the RN. DIF: Cognitive Level: Apply (application)

During the neurologic assessment, the patient is unable to respond verbally to the nurse but cooperates with the nurse's directions to move his hands and feet. The nurse will suspect a. cerebellar injury. b. a brainstem lesion. c. frontal lobe damage. d. a temporal lobe lesion.

ANS: C Expressive speech is controlled by Broca's area in the frontal lobe. The temporal lobe contains Wernicke's area, which is responsible for receptive speech. The cerebellum and brainstem do not affect higher cognitive functions such as speech. DIF: Cognitive Level: Apply (application) REF: 1339 TOP: Nursing Process: Assessment MSC: NCLEX: Physiological Integrity

To prevent autonomic hyperreflexia, which nursing action will the home health nurse include in the plan of care for a patient who has paraplegia at the T4 level ? a. Support selection of a high-protein diet. b. Discuss options for sexuality and fertility. c. Assist in planning a prescribed bowel program. d. Use quad coughing to strengthen cough efforts.

ANS: C Fecal impaction is a common stimulus for autonomic hyperreflexia. Dietary protein, coughing, and discussing sexuality and fertility should be included in the plan of care but will not reduce the risk for autonomic hyperreflexia. DIF: Cognitive Level: Apply (application)

A 37-yr-old female patient is hospitalized with acute kidney injury (AKI). Which information will be most useful to the nurse in evaluating improvement in kidney function? a. Urine volume b. Creatinine level c. Glomerular filtration rate (GFR) d. Blood urea nitrogen (BUN) level

ANS: C GFR is the preferred method for evaluating kidney function. BUN levels can fluctuate based on factors such as fluid volume status and protein intake. Urine output can be normal or high in patients with AKI and does not accurately reflect kidney function. Creatinine alone is not an accurate reflection of renal function. DIF: Cognitive Level: Analyze (analysis) REF: 1079 TOP: Nursing Process: Evaluation MSC: NCLEX: Physiological Integrity

A patient with severe burns has crystalloid fluid replacement ordered using the Parkland formula. The initial volume of fluid to be administered in the first 24 hours is 30,000 mL. The initial rate of administration is 1875 mL/hr. After the first 8 hours, what rate should the nurse infuse the IV fluids? a. 350 mL/hour b. 523 mL/hour c. 938 mL/hour d. 1250 mL/hour

ANS: C Half of the fluid replacement using the Parkland formula is administered in the first 8 hours and the other half over the next 16 hours. In this case, the patient should receive half of the initial rate, or 938 mL/hr.

A 55-yr-old patient with end-stage kidney disease (ESKD) is scheduled to receive a prescribed dose of epoetin alfa (Procrit). Which information should the nurse report to the health care provider before giving the medication? a. Creatinine 1.6 mg/dL b. Oxygen saturation 89% c. Hemoglobin level 13 g/dL d. Blood pressure 98/56 mm Hg

ANS: C High hemoglobin levels are associated with a higher rate of thromboembolic events and increased risk of death from serious cardiovascular events (heart attack, heart failure, stroke) when erythropoietin (EPO) is administered to a target hemoglobin of greater than 12 g/dL. Hemoglobin levels higher than 12 g/dL indicate a need for a decrease in epoetin alfa dose. The other information also will be reported to the health care provider but will not affect whether the medication is administered. DIF: Cognitive Level: Apply (application) REF: 1081 TOP: Nursing Process: Assessment MSC: NCLEX: Physiological Integrity

The nurse is reviewing laboratory results on a patient who had a large burn 48 hours ago. Which result requires priority action by the nurse? a. Hematocrit 53% b. Serum sodium 147 mEq/L c. Serum potassium 6.1 mEq/L d. Blood urea nitrogen 37 mg/dL

ANS: C Hyperkalemia can lead to fatal dysrhythmias and indicates that the patient requires cardiac monitoring and immediate treatment to lower the potassium level. The other laboratory values are also abnormal and require changes in treatment, but they are not as immediately life threatening as the elevated potassium level

The nurse is evaluating the effectiveness of the intravenous fluid therapy in a patient with hypernatremia. Which finding indicates goal achievement? a. Urine output increases to 150 mL/hr. b. Systolic and diastolic blood pressure decreases. c. Serum sodium concentration returns to normal. d. Large amounts of emesis and diarrhea decrease.

ANS: C Hypernatremia is diagnosed by elevated serum sodium concentration. Blood pressure is not an accurate indicator of hypernatremia. Emesis and diarrhea will not stop because of intravenous therapy. Urine output is influenced by many factors, including extracellular fluid volume. A large dilute urine output can cause further hypernatremia.

Which stroke risk factor for a 48-yr-old male patient in the clinic is most important for the nurse to address? a. The patient is 25 lb above the ideal weight. b. The patient drinks a glass of red wine with dinner daily. c. The patient's usual blood pressure (BP) is 170/94 mm Hg. d. The patient works at a desk and relaxes by watching television.

ANS: C Hypertension is the single most important modifiable risk factor. People who drink more than 1 (for women) or 2 (for men) alcoholic beverages a day may increase their risk for hypertension. Physical inactivity and obesity contribute to stroke risk but not as much as hypertension. DIF: Cognitive Level: Analyze (analysis) REF: 1347 OBJ: Special Questions: Prioritization TOP: Nursing Process: Assessment MSC: NCLEX: Health Promotion and Maintenance

Before administration of calcium carbonate to a patient with chronic kidney disease (CKD), the nurse should check laboratory results for a. potassium level. b. total cholesterol. c. serum phosphate. d. serum creatinine

ANS: C If serum phosphate is elevated, the calcium and phosphate can cause soft tissue calcification. The calcium carbonate should not be given until the phosphate level is lowered. Total cholesterol, creatinine, and potassium values do not affect whether calcium carbonate should be administered DIF: Cognitive Level: Apply (application) REF: 1081 TOP: Nursing Process: Implementation MSC: NCLEX: Physiological Integrity

A 19-yr-old student comes to the student health center at the end of the semester complaining that, "My heart is skipping beats." An electrocardiogram (ECG) shows occasional unifocal premature ventricular contractions (PVCs). What action should the nurse take next? a. Insert an IV catheter for emergency use. b. Start supplemental O2 at 2 to 3 L/min via nasal cannula. c. Ask the patient about current stress level and caffeine use. d. Have the patient taken to the nearest emergency department (ED).

ANS: C In a patient with a normal heart, occasional PVCs are a benign finding. The timing of the PVCs suggests stress or caffeine as possible etiologic factors. The patient is hemodynamically stable, so there is no indication that the patient needs supplemental O2, an IV, or to be seen in the ED. DIF: Cognitive Level: Apply (application)

When a brain-injured patient responds to nail bed pressure with internal rotation, adduction, and flexion of the arms, the nurse reports the response as a. flexion withdrawal. b. localization of pain. c. decorticate posturing. d. decerebrate posturing.

ANS: C Internal rotation, adduction, and flexion of the arms in an unconscious patient is documented as decorticate posturing. Extension of the arms and legs is decerebrate posturing. Because the flexion is generalized, it does not indicate localization of pain or flexion withdrawal.

The family members of a patient who has been admitted to the intensive care unit (ICU) with multiple traumatic injuries have just arrived in the ICU waiting room. Which action should the nurse take first? a. Explain ICU visitation policies and encourage family visits. b. Escort the family from the waiting room to the patient's bedside. c. Describe the patient's injuries and the care that is being provided. d. Invite the family to participate in an interprofessional care conference.

ANS: C Lack of information is a major source of anxiety for family members and should be addressed first. Family members should be prepared for the patient's appearance and the ICU environment before visiting the patient for the first time. ICU visiting should be individualized to each patient and family rather than being dictated by rigid visitation policies. Inviting the family to participate in a multidisciplinary conference is appropriate but should not be the initial action by the nurse. DIF: Cognitive Level: Analyze (analysis)

The nurse in the dialysis clinic is reviewing the home medications of a patient with chronic kidney disease (CKD). Which medication reported by the patient indicates that patient teaching is required? a. Acetaminophen b. Calcium phosphate c. Magnesium hydroxide d. Multivitamin with iron

ANS: C Magnesium is excreted by the kidneys, and patients with CKD should not use over-the-counter products containing magnesium. The other medications are appropriate for a patient with CKD. DIF: Cognitive Level: Apply (application) REF: 1081 TOP: Nursing Process: Assessment MSC: NCLEX: Physiological Integrity

The nurse will plan to teach the patient diagnosed with acute hepatitis B about a. administering -interferon b. side effects of nucleotide analogs. c. measures for improving the appetite. d. ways to increase activity and exercise.

ANS: C Maintaining adequate nutritional intake is important for regeneration of hepatocytes. Interferon and antivirals may be used for chronic hepatitis B, but they are not prescribed for acute hepatitis B infection. Rest is recommended.

The nurse has administered prescribed IV mannitol (Osmitrol) to an unconscious patient. Which parameter should the nurse monitor to determine the medication's effectiveness? a. Blood pressure b. Oxygen saturation c. Intracranial pressure d. Hemoglobin and hematocrit

ANS: C Mannitol is an osmotic diuretic and will reduce cerebral edema and intracranial pressure. It may initially reduce hematocrit and increase blood pressure, but these are not the best parameters for evaluation of the effectiveness of the drug. Oxygen saturation will not directly improve as a result of mannitol administration

A patient is to receive 1000 mL of 0.9% sodium chloride intravenously at a rate of 125 mL/hr. The nurse is using microdrip gravity drip tubing. Which rate will the nurse calculate for the minute flow rate (drops/min)? a. 12 drops/min b. 24 drops/min c. 125 drops/min d. 150 drops/min

ANS: C Microdrip tubing delivers 60 drops/mL. Calculation for a rate of 125 mL/hr using microdrip tubing: (125 mL/1 hr)(60 drops/1 mL)(1 hr/60 min) = 125 drop/min.

Which assessment finding would the nurse need to report most quickly to the health care provider regarding a patient with acute pancreatitis? a. Nausea and vomiting b. Hypotonic bowel sounds c. Muscle twitching and finger numbness d. Upper abdominal tenderness and guarding

ANS: C Muscle twitching and finger numbness indicate hypocalcemia, which may lead to tetany unless calcium gluconate is administered. Although the other findings should also be reported to the health care provider, they do not indicate complications that require rapid action. DIF: Cognitive Level: Analyze (analysis) REF: 1002 OBJ: Special Questions: Prioritization TOP: Nursing Process: Assessment MSC: NCLEX: Physiological Integrity

A 19-year-old patient with massive trauma and possible spinal cord injury is admitted to the emergency department (ED). Which assessment finding by the nurse will help confirm a diagnosis of neurogenic shock? a. Inspiratory crackles. b. Cool, clammy extremities. c. Apical heart rate 45 beats/min. d. Temperature 101.2° F (38.4° C).

ANS: C Neurogenic shock is characterized by hypotension and bradycardia. The other findings would be more consistent with other types of shock

The nurse is admitting a patient with a neck fracture at the C6 level to the intensive care unit. Which assessment finding(s) indicate(s) neurogenic shock? a. Hyperactive reflex activity below the level of injury b. Involuntary, spastic movements of the arms and legs c. Hypotension, bradycardia, and warm, pink extremities d. Lack of sensation or movement below the level of injury

ANS: C Neurogenic shock is characterized by hypotension, bradycardia, and vasodilation leading to warm skin temperature. Spasticity and hyperactive reflexes do not occur at this stage of spinal cord injury. Lack of movement and sensation indicate spinal cord injury, but not neurogenic shock.

An older patient with cardiogenic shock is cool and clammy and hemodynamic monitoring indicates a high systemic vascular resistance (SVR). Which intervention should the nurse anticipate doing next? a. Increase the rate for the dopamine (Intropin) infusion. b. Decrease the rate for the nitroglycerin (Tridil) infusion. c. Increase the rate for the sodium nitroprusside (Nipride) infusion. d. Decrease the rate for the 5% dextrose in normal saline (D5/.9 NS) infusion.

ANS: C Nitroprusside is an arterial vasodilator and will decrease the SVR and afterload, which will improve cardiac output. Changes in the D5/.9 NS and nitroglycerin infusions will not directly decrease SVR. Increasing the dopamine will tend to increase SVR.

A licensed practical/vocational nurse (LPN/LVN) is caring for a patient with stage 2 chronic kidney disease. Which observation by the RN requires an intervention? a. The LPN/LVN administers the erythropoietin subcutaneously. b. The LPN/LVN assists the patient to ambulate out in the hallway. c. The LPN/LVN administers the iron supplement and phosphate binder with lunch. d. The LPN/LVN carries a tray containing low-protein foods into the patient's room.

ANS: C Oral phosphate binders should not be given at the same time as iron because they prevent the iron from being absorbed. The phosphate binder should be given with a meal and the iron given at a different time. The other actions by the LPN/LVN are appropriate for a patient with renal insufficiency. DIF: Cognitive Level: Apply (application) REF: 1082 OBJ: Special Questions: Delegation TOP: Nursing Process: Implementation MSC: NCLEX: Safe and Effective Care Environment

While reviewing the complete blood count (CBC) of a patient on her unit, the nurse notes elevated basophil and eosinophil readings. The nurse realizes that this is most indicative of a _____ infection. a. bacterial b. fungal c. parasitic d. viral

ANS: C Parasitic infections are frequently indicated on a CBC by elevated basophil and eosinophil levels. Bacterial infections do not lead to elevated basophil and eosinophil levels but elevated B and T lymphocytes, neutrophils, and monocytes. Fungal infections do not lead to elevated basophil and eosinophil levels. Viral infections create elevations in B and T lymphocytes, neutrophils, and monocytes.

Which action by a patient who is using peritoneal dialysis (PD) indicates that the nurse should provide more teaching about PD? a. The patient leaves the catheter exit site without a dressing. b. The patient plans 30 to 60 minutes for a dialysate exchange. c. The patient cleans the catheter while taking a bath each day. d. The patient slows the inflow rate when experiencing abdominal pain.

ANS: C Patients are encouraged to take showers rather than baths to avoid infections at the catheter insertion side. The other patient actions indicate good understanding of peritoneal dialysis. DIF: Cognitive Level: Apply (application) REF: 1086 TOP: Nursing Process: Evaluation MSC: NCLEX: Physiological Integrity

A patient presents to the emergency department with reports of vomiting and diarrhea for the past 48 hours. The health care provider orders isotonic intravenous (IV) therapy. Which IV will the nurse prepare? a. 0.225% sodium chloride (1/4 NS) b. 0.45% sodium chloride (1/2 NS) c. 0.9% sodium chloride (NS) d. 3% sodium chloride (3% NaCl)

ANS: C Patients with prolonged vomiting and diarrhea become hypovolemic. A solution to replace extracellular volume is 0.9% sodium chloride, which is an isotonic solution. 0.225% and 0.45% sodium chloride are hypotonic. 3% sodium chloride is hypertonic. DIF:Apply (application)REF:956

A patient with chronic hepatitis C infection has several medications prescribed. Which medication requires further discussion with the health care provider before administration? a. Ribavirin (Rebetol, Copegus) 600 mg PO bid b. Diphenhydramine 25 mg PO every 4 hours PRN itching c. Pegylated -interferon (PEG-Intron, Pegasys) 1.5 mcg/kg PO daily d. Dimenhydrinate (Dramamine) 50 mg PO every 6 hours PRN nausea

ANS: C Pegylated a-interferon is administered subcutaneously, not orally. The medications are all appropriate for a patient with chronic hepatitis C infection. DIF: Cognitive Level: Understand (comprehension) REF: 981 TOP: Nursing Process: Implementation MSC: NCLEX: Physiological Integrity

Which menu choice by the patient who is receiving hemodialysis indicates that the nurse's teaching has been successful? a. Split-pea soup, English muffin, and nonfat milk b. Oatmeal with cream, half a banana, and herbal tea c. Poached eggs, whole-wheat toast, and apple juice d. Cheese sandwich, tomato soup, and cranberry juice

ANS: C Poached eggs would provide high-quality protein, and apple juice is low in potassium. Cheese is high in salt and phosphate, and tomato soup would be high in potassium. Split-pea soup is high in potassium, and dairy products are high in phosphate. Bananas are high in potassium, and the cream would be high in phosphate DIF: Cognitive Level: Apply (application) REF: 1087 TOP: Nursing Process: Evaluation MSC: NCLEX: Physiological Integrity

A patient had an incisional cholecystectomy 6 hours ago. The nurse will place the highest priority on assisting the patient to a. perform leg exercises hourly while awake. b. ambulate the evening of the operative day. c. turn, cough, and deep breathe every 2 hours. d. choose preferred low-fat foods from the menu.

ANS: C Postoperative nursing care after a cholecystectomy focuses on prevention of respiratory complications because the surgical incision is high in the abdomen and impairs coughing and deep breathing. The other nursing actions are also important to implement but are not as high a priority as ensuring adequate ventilation.

A patient is to receive phototherapy for the treatment of psoriasis after spinal cord injury. What is the nursing priority for this patient? a. Obtaining a complete blood count (CBC) b. Protection from excessive heat c. Protection from excessive UV exposure d. Instructing the patient to take their multivitamin prior to treatment

ANS: C Protection from excessive UV exposure is important to prevent tissue damage. Protection from heat is not the most important priority for this patient. There is no need for vitamins or a CBC for patients with psoriasis.

A patient with left-sided weakness that started 60 minutes earlier is admitted to the emergency department and diagnostic tests are ordered. Which test should be done first? a. Complete blood count (CBC) b. Chest radiograph (chest x-ray) c. Computed tomography (CT) scan d. 12-Lead electrocardiogram (ECG)

ANS: C Rapid screening with a noncontrast CT scan is needed before administration of tissue plasminogen activator (tPA), which must be given within 4.5 hours of the onset of clinical manifestations of the stroke. The sooner the tPA is given, the less brain injury. The other diagnostic tests give information about possible causes of the stroke and do not need to be completed as urgently as the CT scan.

The charge nurse observes an inexperienced staff nurse caring for a patient who has had a craniotomy for resection of a brain tumor. Which action by the inexperienced nurse requires the charge nurse to intervene? a. The staff nurse assesses neurologic status every hour. b. The staff nurse elevates the head of the bed to 30 degrees. c. The staff nurse suctions the patient routinely every 2 hours. d. The staff nurse administers an analgesic before turning the patient.

ANS: C Suctioning increases intracranial pressure, and should only be done when the patient's respiratory condition indicates it is needed. The other actions by the staff nurse are appropriate

A patient admitted with dermal ulcers who has a history of a T3 spinal cord injury tells the nurse, "I have a pounding headache and I feel sick to my stomach." Which action should the nurse take first? a. Check for a fecal impaction. b. Give the prescribed analgesic. c. Assess the blood pressure (BP). d. Notify the health care provider.

ANS: C The BP should be assessed immediately in a patient with an injury at the T6 level or higher who complains of a headache to determine whether autonomic dysreflexia is occurring. Notification of the patient's health care provider is appropriate after the BP is obtained. Administration of an antiemetic is indicated after autonomic dysreflexia is ruled out as the cause of the nausea. After checking the BP, the nurse may assess for a fecal impaction using lidocaine jelly to prevent further increased BP

The nurse observes unlicensed assistive personnel (UAP) taking the following actions when caring for a patient with a urethral catheter. Which action requires that the nurse intervene? a. Taping the catheter to the skin on the patient's upper inner thigh b. Cleaning around the patient's urinary meatus with soap and water c. Disconnecting the catheter from the drainage tube to obtain a specimen d. Using an alcohol-based gel hand cleaner before performing catheter care

ANS: C The catheter should not be disconnected from the drainage tube because this increases the risk for urinary tract infection. The other actions are appropriate and do not require any intervention. DIF: Cognitive Level: Apply (application) REF: 1059 OBJ: Special Questions: Delegation TOP: Nursing Process: Implementation MSC: NCLEX: Safe and Effective Care Environment

The nurse is caring for a patient who has a head injury and fractured right arm after being assaulted. Which assessment information requires rapid action by the nurse? a. The apical pulse is slightly irregular. b. The patient complains of a headache. c. The patient is more difficult to arouse. d. The blood pressure (BP) increases to 140/62 mm Hg.

ANS: C The change in level of consciousness (LOC) is an indicator of increased intracranial pressure (ICP) and suggests that action by the nurse is needed to prevent complications. The change in BP should be monitored but is not an indicator of a need for immediate nursing action. Headache and a slightly irregular apical pulse are not unusual in a patient after a head injury. DIF: Cognitive Level: Apply (application)

Which is best described as the inability of the heart to pump an adequate amount of blood to the systemic circulation at normal filling pressures? a. Pulmonary congestion b. Congenital heart defect c. Heart failure d. Systemic venous congestion

ANS: C The definition of heart failure is the inability of the heart to pump an adequate amount of blood to the systemic circulation at normal filling pressures to meet the body's metabolic demands. Pulmonary congestion is an excessive accumulation of fluid in the lungs. Congenital heart defect is a malformation of the heart present at birth. Systemic venous congestion is an excessive accumulation of fluid in the systemic vasculature.

Which topic is most important to include in patient teaching for a 41-yr-old patient diagnosed with early alcoholic cirrhosis? a. Taking lactulose c. Avoiding alcohol ingestion b. Maintaining good nutrition d. Using vitamin B supplements

ANS: C The disease progression can be stopped or reversed by alcohol abstinence. The other interventions may be used when cirrhosis becomes more severe to decrease symptoms or complications, but the priority for this patient is to stop the progression of the disease.

An older patient receiving iso-osmolar continuous tube feedings develops restlessness, agitation, and weakness. Which laboratory result should the nurse report to the health care provider immediately? a. K+ 3.4 mEq/L (3.4 mmol/L) b. Ca+2 7.8 mg/dL (1.95 mmol/L) c. Na+ 154 mEq/L (154 mmol/L) d. PO4-3 4.8 mg/dL (1.55 mmol/L)

ANS: C The elevated serum sodium level is consistent with the patient's neurologic symptoms and indicates a need for immediate action to prevent further serious complications such as seizures. The potassium and calcium levels vary slightly from normal but do not require immediate action by the nurse. The phosphate level is normal

A patient who has been involved in a motor vehicle crash arrives in the emergency department (ED) with cool, clammy skin; tachycardia; and hypotension. Which intervention ordered by the health care provider should the nurse implement first? a. Insert two large-bore IV catheters. b. Initiate continuous electrocardiogram (ECG) monitoring. c. Provide oxygen at 100% per non-rebreather mask. d. Draw blood to type and crossmatch for transfusions.

ANS: C The first priority in the initial management of shock is maintenance of the airway and ventilation. ECG monitoring, insertion of IV catheters, and obtaining blood for transfusions should also be rapidly accomplished but only after actions to maximize oxygen delivery have been implemented.

Which action should the nurse take first when a patient complains of acute chest pain and dyspnea soon after insertion of a centrally inserted IV catheter? a. Notify the health care provider. b. Offer reassurance to the patient. c. Auscultate the patient's breath sounds. d. Give the prescribed PRN morphine sulfate IV.

ANS: C The initial action should be to assess the patient further because the history and symptoms are consistent with several possible complications of central line insertion, including embolism and pneumothorax. The other actions may be appropriate, but further assessment of the patient is needed before notifying the health care provider, offering reassurance, or administration of morphine

After receiving change-of-shift report, which patient should the nurse assess first? a. Patient with serum potassium level of 5.0 mEq/L who is complaining of abdominal cramping b. Patient with serum sodium level of 145 mEq/L who has a dry mouth and is asking for a glass of water c. Patient with serum magnesium level of 1.1 mEq/L who has tremors and hyperactive deep tendon reflexes d. Patient with serum phosphorus level of 4.5 mg/dL who has multiple soft tissue calcium-phosphate precipitates

ANS: C The low magnesium level and neuromuscular irritability suggest that the patient may be at risk for seizures. The other patients have mild electrolyte disturbances and/or symptoms that require action, but they are not at risk for life-threatening complications

An unconscious patient is admitted to the emergency department (ED) with a head injury. The patient's spouse and teenage children stay at the patient's side and ask many questions about the treatment being given. What action is best for the nurse to take? a. Call the family's pastor or spiritual advisor to take them to the chapel. b. Ask the family to stay in the waiting room until the assessment is completed. c. Allow the family to stay with the patient and briefly explain all procedures to them. d. Refer the family members to the hospital counseling service to deal with their anxiety.

ANS: C The need for information about the diagnosis and care is very high in family members of acutely ill patients. The nurse should allow the family to observe care and explain the procedures unless they interfere with emergent care needs. A pastor or counseling service can offer some support, but research supports information as being more effective. Asking the family to stay in the waiting room will increase their anxiety. DIF: Cognitive Level: Analyze (analysis)

Which action should the nurse in the emergency department take first for a new patient who is vomiting blood? a. Insert a large-gauge IV catheter. b. Draw blood for coagulation studies. c. Check blood pressure and heart rate. d. Place the patient in the supine position.

ANS: C The nurse's first action should be to determine the patient's hemodynamic status by assessing vital signs. Drawing blood for coagulation studies and inserting an IV catheter are also appropriate. However, the vital signs may indicate the need for more urgent actions. Because aspiration is a concern for this patient, the nurse will need to assess the patient's vital signs and neurologic status before placing the patient in a supine position

A patient who is complaining of a "racing" heart and feeling "anxious" comes to the emergency department. The nurse places the patient on a heart monitor and obtains the following electrocardiographic (ECG) tracing. Which action should the nurse take next? a. Prepare to perform electrical cardioversion. b. Have the patient perform the Valsalva maneuver. c. Obtain the patient's vital signs including O2 saturation. d. Prepare to give a -blocker medication to slow the heart rate.

ANS: C The patient has sinus tachycardia, which may have multiple etiologies such as pain, dehydration, anxiety, and myocardial ischemia. Further assessment is needed before determining the treatment. Vagal stimulation or -blockade may be used after further assessment of the patient. Electrical cardioversion is used for some tachydysrhythmias but would not be used for sinus tachycardia. DIF: Cognitive Level: Analyze (analysis)

A patient will attempt oral feedings for the first time after having a stroke. The nurse should assess the gag reflex and then a. order a varied pureed diet. b. assess the patient's appetite. c. assist the patient into a chair. d. offer the patient a sip of juice.

ANS: C The patient should be as upright as possible before attempting feeding to make swallowing easier and decrease aspiration risk. To assess swallowing ability, the nurse should initially offer water or ice to the patient. Pureed diets are not recommended because the texture is too smooth. The patient may have a poor appetite, but the oral feeding should be attempted.

To prepare a patient with ascites for paracentesis, the nurse a. places the patient on NPO status. b. assists the patient to lie flat in bed. c. asks the patient to empty the bladder. d. positions the patient on the right side.

ANS: C The patient should empty the bladder to decrease the risk of bladder perforation during the procedure. The patient would be positioned in Fowler's position and would not be able to lie flat without compromising breathing. Because no sedation is required for paracentesis, the patient does not need to be NPO.

A patient has an incomplete left spinal cord lesion at the level of T7, resulting in Brown-Séquard syndrome. Which nursing action should be included in the plan of care? a. Assessment of the patient for right arm weakness b. Assessment of the patient for increased right leg pain c. Positioning the patient's left leg when turning the patient d. Teaching the patient to look at the right leg to verify its position

ANS: C The patient with Brown-Séquard syndrome has loss of motor function on the ipsilateral side and will require the nurse to move the left leg. Pain sensation will be lost on the patient's right leg. Arm weakness will not be a problem for a patient with a T7 injury. The patient will retain position sense for the right leg

When taking the blood pressure (BP) on the right arm of a patient with severe acute pancreatitis, the nurse notices carpal spasms of the patient's right hand. Which action should the nurse take next? a. Ask the patient about any arm pain. b. Retake the patient's blood pressure. c. Check the calcium level in the chart. d. Notify the health care provider immediately.

ANS: C The patient with acute pancreatitis is at risk for hypocalcemia, and the assessment data indicate a positive Trousseau's sign. The health care provider should be notified after the nurse checks the patient's calcium level. There is no indication that the patient needs to have the BP rechecked or that there is any arm pain.

Which statement by a patient with stage 5 chronic kidney disease (CKD) indicates that the nurse's teaching about management of CKD has been effective? a. "I need to get most of my protein from low-fat dairy products." b. "I will increase my intake of fruits and vegetables to 5 per day." c. "I will measure my urinary output each day to help calculate the amount I can drink." d. "I need to take erythropoietin to boost my immune system and help prevent infection."

ANS: C The patient with end-stage renal disease is taught to measure urine output as a means of determining an appropriate oral fluid intake. Erythropoietin is given to increase the red blood cell count and will not offer any benefit for immune function. Dairy products are restricted because of the high phosphate level. Many fruits and vegetables are high in potassium and should be restricted in the patient with CKD.

A patient with a systemic bacterial infection feels cold and has a shaking chill. Which assessment finding will the nurse expect next? a. Skin flushing b. Muscle cramps c. Rising body temperature d. Decreasing blood pressure

ANS: C The patient's complaints of feeling cold and shivering indicate that the hypothalamic set point for temperature has been increased and the temperature is increasing. Because associated peripheral vasoconstriction and sympathetic nervous system stimulation will occur, skin flushing and hypotension are not expected. Muscle cramps are not expected with chills and shivering or with a rising temperature

After the insertion of an arteriovenous graft (AVG) in the right forearm, a patient complains of pain and coldness of the right fingers. Which action should the nurse take? a. Teach the patient about normal AVG function. b. Remind the patient to take a daily low-dose aspirin tablet. c. Report the patient's symptoms to the health care provider. d. Elevate the patient's arm on pillows to above the heart level.

ANS: C The patient's complaints suggest the development of distal ischemia (steal syndrome) and may require revision of the AVG. Elevation of the arm above the heart will further decrease perfusion. Pain and coolness are not normal after AVG insertion. Aspirin therapy is not used to maintain grafts. DIF: Cognitive Level: Apply (application) REF: 1088 TOP: Nursing Process: Implementation MSC: NCLEX: Physiological Integrity

The nurse assesses a patient who has been hospitalized for 2 days. The patient has been receiving normal saline IV at 100 mL/hr, has a nasogastric tube to low suction, and is NPO. Which assessment finding would be a priority for the nurse to report to the health care provider? a. Oral temperature of 100.1° F b. Serum sodium level of 138 mEq/L (138 mmol/L) c. Gradually decreasing level of consciousness (LOC) d. Weight gain of 2 pounds (1 kg) above the admission weight

ANS: C The patient's history and change in LOC could be indicative of fluid and electrolyte disturbances: extracellular fluid (ECF) excess, ECF deficit, hyponatremia, hypernatremia, hypokalemia, or metabolic alkalosis. Further diagnostic information is needed to determine the cause of the change in LOC and the appropriate interventions. The weight gain, elevated temperature, crackles, and serum sodium level also will be reported, but do not indicate a need for rapid action to avoid complications

The nurse assesses vital signs for a patient admitted 2 days ago with gram-negative sepsis: temperature of 101.2° F, blood pressure of 90/56 mm Hg, pulse of 92 beats/min, and respirations of 34 breaths/min. Which action should the nurse take next? a. Give the scheduled IV antibiotic. b. Give the PRN acetaminophen (Tylenol). c. Obtain oxygen saturation using pulse oximetry. d. Notify the health care provider of the patient's vital signs.

ANS: C The patient's increased respiratory rate in combination with the admission diagnosis of gram-negative sepsis indicates that acute respiratory distress syndrome (ARDS) may be developing. The nurse should check for hypoxemia, a hallmark of ARDS. The health care provider should be notified after further assessment of the patient. Giving the scheduled antibiotic and the PRN acetaminophen will also be done, but they are not the highest priority for a patient who may be developing ARDS.

On admission to the burn unit, a patient with an approximate 25% total body surface area (TBSA) burn has the following initial laboratory results: Hct 58%, Hgb 18.2 mg/dL (172 g/L), serum K+ 4.9 mEq/L (4.8 mmol/L), and serum Na+ 135 mEq/L (135 mmol/L). Which action will the nurse anticipate taking now? a. Monitor urine output every 4 hours. b. Continue to monitor the laboratory results. c. Increase the rate of the ordered IV solution. d. Type and crossmatch for a blood transfusion.

ANS: C The patient's laboratory data show hemoconcentration, which may lead to a decrease in blood flow to the microcirculation unless fluid intake is increased. Because the hematocrit and hemoglobin are elevated, a transfusion is inappropriate, although transfusions may be needed after the emergent phase once the patient's fluid balance has been restored. On admission to a burn unit, the urine output would be monitored more often than every 4 hours; likely every1 hour.

The nurse evaluates that administration of hepatitis B vaccine to a healthy patient has been effective when the patient's blood specimen reveals A. HBsAg. B. anti-HBc IgG. C. anti-HBs. D. anti-HBc IgM.

ANS: C The presence of surface antibody to HBV (anti-HBs) is a marker of a positive response to the vaccine. The other laboratory values indicate current infection with HBV.

The nurse notes that a patient who was admitted with diabetic ketoacidosis has rapid, deep respirations. Which action should the nurse take? a. Give the prescribed PRN lorazepam (Ativan). b. Start the prescribed PRN oxygen at 2 to 4 L/min. c. Administer the prescribed normal saline bolus and insulin. d. Encourage the patient to take deep, slow breaths with guided imagery.

ANS: C The rapid, deep (Kussmaul) respirations indicate a metabolic acidosis and the need for correction of the acidosis with a saline bolus to prevent hypovolemia followed by insulin administration to allow glucose to reenter the cells. Oxygen therapy is not indicated because there is no indication that the increased respiratory rate is related to hypoxemia. The respiratory pattern is compensatory, and the patient will not be able to slow the respiratory rate. Lorazepam administration will slow the respiratory rate and increase the level of acidosis

A current recommendation to prevent neural tube defects is the supplementation of: a. Vitamin A throughout pregnancy. b. Multivitamin preparations as soon as pregnancy is suspected. c. Folic acid for all women of childbearing age. d. Folic acid during the first and second trimesters of pregnancy

ANS: C The widespread use of folic acid among women of childbearing age is expected to decrease the incidence of spina bifida significantly. Vitamin A and multivitamin preparations do not have a relation to the prevention of spina bifida. Folic acid supplementation is recommended for the preconceptual period and during the pregnancy. Only 42% of women actually follow these guidelines. p1582

During change-of-shift report, the nurse learns about the following four patients. Which patient requires assessment first? a. A 40-yr-old patient with chronic pancreatitis who has gnawing abdominal pain b. A 58-yr-old patient who has compensated cirrhosis and is complaining of anorexia c. A 55-yr-old patient with cirrhosis and ascites who has an oral temperature of 102° F (38.8° C) d. A 36-yr-old patient recovering from a laparoscopic cholecystectomy who has severe shoulder pain

ANS: C This patient's history and fever suggest possible spontaneous bacterial peritonitis, which would require rapid assessment and interventions such as antibiotic therapy. The clinical manifestations for the other patients are consistent with their diagnoses and do not indicate complications are occurring.

A nurse is caring for a cancer patient who presents with anorexia, blood pressure 100/60, and elevated white blood cell count. Which primary purpose for starting total parenteral nutrition (TPN) will the nurse add to the care plan? a. Stimulate the patient's appetite to eat. b. Deliver antibiotics to fight off infection. c. Replace fluid, electrolytes, and nutrients. d. Provide medication to raise blood pressure.

ANS: C Total parenteral nutrition is an intravenous solution composed of nutrients and electrolytes to replace the ones the patient is not eating or losing. TPN does not stimulate the appetite. TPN does not contain blood pressure medication or antibiotics.

The patient has type B chronic obstructive pulmonary disease (COPD) exacerbated by an acute upper respiratory infection. Which blood gas values should the nurse expect to see? a. pH high, PaCO2 high, HCO3- high b. pH low, PaCO2 low, HCO3- low c. pH low, PaCO2 high, HCO3- high d. pH low, PaCO2 high, HCO3- normal

ANS: C Type B COPD is a chronic disease that causes impaired excretion of carbonic acid, thus causing respiratory acidosis, with PaCO2 high and pH low. This chronic disease exists long enough for some renal compensation to occur, manifested by high HCO3-. Answers that include low or normal bicarbonate are not correct, because the renal compensation for respiratory acidosis involves excretion of more hydrogen ions than usual, with retention of bicarbonate in the blood. High pH occurs with alkalosis, not acidosis.

During the change of shift report, a nurse is told that a patient has an occluded left posterior cerebral artery. The nurse will anticipate that the patient may have a. dysphasia. b. confusion. c. visual deficits. d. poor judgment.

ANS: C Visual disturbances are expected with posterior cerebral artery occlusion. Aphasia occurs with middle cerebral artery involvement. Cognitive deficits and changes in judgment are more typical of anterior cerebral artery occlusion.

A patient with diabetes who has bacterial pneumonia is being treated with IV gentamicin 60 mg IV BID. The nurse will monitor for adverse effects of the medication by evaluating the patient's a. blood glucose. b. urine osmolality. c. serum creatinine. d. serum potassium.

ANS: C When a patient at risk for chronic kidney disease (CKD) receives a potentially nephrotoxic medication, it is important to monitor renal function with BUN and creatinine levels. The other laboratory values would not be useful in assessing for the adverse effects of the gentamicin. DIF: Cognitive Level: Apply (application) REF: 1083 TOP: Nursing Process: Evaluation MSC: NCLEX: Physiological Integrity

The patient with which diagnosis should have the highest priority for teaching regarding foods that are high in magnesium? a. Severe hemorrhage b. Diabetes insipidus c. Oliguric renal disease d. Adrenal insufficiency

ANS: C When renal excretion is decreased, magnesium intake must be decreased also, to prevent hypermagnesemia. The other conditions are not likely to require adjustment of magnesium intake.

A nurse is conducting discharge teaching to parents about the care of their infant after cardiac surgery. The nurse instructs the parents to notify the physician if what condition occurs? Select all that apply. A. Respiratory rate of 36 at rest B. Appetite slowly increasing C. Temperature above 37.7° C (100° F) D. New, frequent coughing E. Turning blue or bluer than normal

ANS: C, D, E Feedback Correct: The parents should be instructed to notify the physician after their infant's cardiac surgery for a temperature above 37.7° C; new, frequent coughing; and any episodes of the infant turning blue or bluer than normal. Incorrect: A respiratory rate of 36 at rest for an infant is within normal expectations, and it is expected that the appetite will increase slowly.

A patient with a left-brain stroke suddenly bursts into tears when family members visit. The nurse should a. use a calm voice to ask the patient to stop the crying behavior. b. explain to the family that depression is normal following a stroke. c. have the family members leave the patient alone for a few minutes. d. teach the family that emotional outbursts are common after strokes.

ANS: D Patients who have left-sided brain stroke are prone to emotional outbursts that are not necessarily related to the emotional state of the patient. Depression after a stroke is common, but the suddenness of the patient's outburst suggests that depression is not the major cause of the behavior. The family should stay with the patient. The crying is not within the patient's control, and asking the patient to stop will lead to embarrassment. DIF: Cognitive Level: Apply (application) REF: 1364 TOP: Nursing Process: Implementation MSC: NCLEX: Psychosocial Integrity

Which statement best describes a myelomeningocele? a. Fissure in the spinal column that leaves the meninges and the spinal cord exposed. b. Herniation of the brain and meninges through a defect in the skull. c. Hernial protrusion of a sac-like cyst of meninges with spinal fluid but no neural elements. d. Visible defect with an external sac-like protrusion containing meninges, spinal fluid, and nerves.

ANS: D A myelomeningocele is a visible defect with an external sac-like protrusion, containing meninges, spinal fluid, and nerves. Rachischisis is a fissure in the spinal column that leaves the meninges and the spinal cord exposed. Encephalocele is a herniation of brain and meninges through a defect in the skull, producing a fluid-filled sac. Meningocele is a hernial protrusion of a sac-like cyst of meninges with spinal fluid, but no neural elements.

Which assessment finding is of most concern for a patient with acute pancreatitis? a. Absent bowel sounds c. Left upper quadrant pain b. Abdominal tenderness d. Palpable abdominal mass

ANS: D A palpable abdominal mass may indicate the presence of a pancreatic abscess, which will require rapid surgical drainage to prevent sepsis. Absent bowel sounds, abdominal tenderness, and left upper quadrant pain are common in acute pancreatitis and do not require rapid action to prevent further complications.

A nurse is assisting the health care provider in inserting a central line. Which action indicates the nurse is following the recommended bundle protocol to reduce central line-associated bloodstream infections (CLABSI)? a. Preps skin with povidone-iodine solution. b. Suggests the femoral vein for insertion site. c. Applies double gloving without hand hygiene. d. Uses chlorhexidine skin antisepsis prior to insertion.

ANS: D A recommended bundle at insertion of a central line is hand hygiene prior to catheter insertion; use of maximum sterile barrier precautions upon insertion; chlorhexidine skin antisepsis prior to insertion and during dressing changes; avoidance of the femoral vein for central venous access for adults; and daily evaluation of line necessity, with prompt removal of non-essential lines. Povidone-iodine is not recommended.

A patient who has burns on the arms, legs, and chest from a house fire has become agitated and restless 8 hours after being admitted to the hospital. Which action should the nurse take first? a. Stay at the bedside and reassure the patient. b. Administer the ordered morphine sulfate IV. c. Assess orientation and level of consciousness. d. Use pulse oximetry to check the oxygen saturation.

ANS: D Agitation in a patient who may have suffered inhalation injury might indicate hypoxia, and this should be assessed by the nurse first. Administration of morphine may be indicated if the nurse determines that the agitation is caused by pain. Assessing level of consciousness and orientation is also appropriate but not as essential as determining whether the patient is hypoxemic. Reassurance is not helpful to reduce agitation in a hypoxemic patient.

The nurse will ask a patient being admitted with acute pancreatitis specifically about a history of a. diabetes mellitus. c. cigarette smoking. b. high-protein diet. d. alcohol consumption.

ANS: D Alcohol use is one of the most common risk factors for pancreatitis in the United States. Cigarette smoking, diabetes, and high-protein diets are not risk factors.

The nurse analyzes the results of a patient's arterial blood gases (ABGs). Which finding would require immediate action? a. The bicarbonate level (HCO3-) is 31 mEq/L. b. The arterial oxygen saturation (SaO2) is 92%. c. The partial pressure of CO2 in arterial blood (PaCO2) is 31 mm Hg. d. The partial pressure of oxygen in arterial blood (PaO2) is 59 mm Hg.

ANS: D All the values are abnormal, but the low PaO2 indicates that the patient is at the point on the oxyhemoglobin dissociation curve where a small change in the PaO2 will cause a large drop in the O2 saturation and a decrease in tissue oxygenation. The nurse should intervene immediately to improve the patient's oxygenation.

What type of shock is characterized by a hypersensitivity reaction causing massive vasodilation and capillary leaks, which may occur with drug or latex allergy? A. Neurogenic shock B Hypovolemic shock C. Cardiogenic shock D. Anaphylactic shock

ANS: D Anaphylactic shock results from extreme allergy or hypersensitivity to a foreign substance. Neurogenic shock results from loss of neuronal control, such as the interruption of neuronal transmission that occurs from a spinal cord injury. Cardiogenic shock is decreased cardiac output. Hypovolemic shock is a reduction in the size of the vascular compartment, decreasing blood pressure, and low central venous pressure.

Which action will the nurse include in the plan of care for a patient in the rehabilitation phase after a burn injury to the right arm and chest? a. Keep the right arm in a position of comfort. b. Avoid the use of sustained-release narcotics. c. Teach about the purpose of tetanus immunization. d. Apply water-based cream to burned areas frequently.

ANS: D Application of water-based emollients will moisturize new skin and decrease flakiness and itching. To avoid contractures, the joints of the right arm should be positioned in an extended position, which is not the position of comfort. Patients may need to continue the use of opioids during rehabilitation. Tetanus immunization would have been given during the emergent phase of the burn injury

Which is the correct point on the accompanying figure where the nurse will assess for ecchymosis when admitting a patient with a basilar skull fracture? a. A(eye) b. B(upper lip) c. C(ear) d. D(base of skull)

ANS: D Battle's sign (postauricular ecchymosis) and periorbital ecchymoses are associated with basilar skull fracture. DIF: Cognitive Level: Understand (comprehension)

A patient with acute respiratory distress syndrome (ARDS) who is intubated and receiving mechanical ventilation develops a right pneumothorax. Which collaborative action will the nurse anticipate next? a. Increase the tidal volume and respiratory rate. b. Decrease the fraction of inspired oxygen (FIO2). c. Perform endotracheal suctioning more frequently. d. Lower the positive end-expiratory pressure (PEEP).

ANS: D Because barotrauma is associated with high airway pressures, the level of PEEP should be decreased. The other actions will not decrease the risk for another pneumothorax.

Which assessment information is most important for the nurse to obtain to evaluate whether treatment of a patient with anaphylactic shock has been effective? a. Heart rate b. Orientation c. Blood pressure d. Oxygen saturation

ANS: D Because the airway edema that is associated with anaphylaxis can affect airway and breathing, the oxygen saturation is the most critical assessment. Improvements in the other assessments will also be expected with effective treatment of anaphylactic shock.

When analyzing the rhythm of a patient's electrocardiogram (ECG), the nurse will need to investigate further upon finding a(n) a. isoelectric ST segment. b. PR interval of 0.18 second. c. QT interval of 0.38 second. d. QRS interval of 0.14 second.

ANS: D Because the normal QRS interval is less than 0.12 seconds, the patient's QRS interval of 0.14 seconds indicates that the conduction through the ventricular conduction system is prolonged. The PR interval and QT interval are within normal range and ST segment should be isoelectric (flat). DIF: Cognitive Level: Apply (application)

The nurse is caring for a patient with hyperkalemia. Which body system assessment is the priority? a. Gastrointestinal b. Neurological c. Respiratory d. Cardiac

ANS: D Cardiac is the priority. Hyperkalemia places the patient at risk for potentially serious dysrhythmias and cardiac arrest. Potassium balance is necessary for cardiac function. Respiratory is the priority with hypokalemia. Monitoring of gastrointestinal and neurological systems would be indicated for other electrolyte imbalances.

A client has undergone renal angiography by way of the right femoral artery. The nurse determines that the client is experiencing a complication of the procedure on noting A. The pressure is 118/76 B. Respiratory rate of 18 C. Urinary output of 40 ml/hr D. Pallor and coolness of the right leg

ANS: D Complications of renal angiography include allergic reaction to the dye, dye-induced renal damage, and a number of vascular complications, including hemorrhage, thrombosis, and embolism. The nurse detects these complications by monitoring the client for signs and symptoms of allergic reaction, decreased urine output, hematoma or hemorrhage at the insertion site, and signs of diminished circulation to the affected leg. The incorrect options are normal findings.

Components of the Glasgow Coma Scale (GCS) the nurse would use to assess a patient after a head injury include which assessment? a. Blood pressure b. Cranial nerve function c. Head circumference d. Verbal responsiveness

ANS: D Components of the GCS include eye opening, motor responsiveness, and verbal responsiveness. The nurse would want to assess the blood pressure, but this is not a component of the coma scale. Assessment of cranial nerve function is appropriate as alterations such as cranial nerve VI palsies may occur, but this is not part of the coma scale. Increases in head circumference are associated with alterations in intracranial pressure in infants, but this is not part of the coma scale.

A patient is receiving a 3% saline continuous IV infusion for hyponatremia. Which assessment data will require the most rapid response by the nurse? a. The patient's radial pulse is 105 beats/minute. b. There is sediment and blood in the patient's urine. c. The blood pressure increases from 120/80 to 142/94. d. There are crackles audible throughout both lung fields.

ANS: D Crackles throughout both lungs suggest that the patient may be experiencing pulmonary edema, a life-threatening adverse effect of hypertonic solutions. The increased pulse rate and blood pressure and the appearance of the urine also should be reported, but they are not as dangerous as the presence of fluid in the alveoli

The parents of a young child with congestive heart failure tell the nurse that they are "nervous" about giving digoxin. The nurse's response should be based on knowing that: a. It is a safe, frequently used drug. b. It is difficult to either overmedicate or undermedicate with digoxin. c. Parents lack the expertise necessary to administer digoxin. d. Parents must learn specific, important guidelines for administration of digoxin.

ANS: D Digoxin has a narrow therapeutic range. The margin of safety between therapeutic, toxic, and lethal doses is very small. Specific guidelines are available for parents to learn how to administer the drug safely and monitor for side effects. Digoxin is a frequently used drug, but it has a narrow therapeutic range. Very small amounts of the liquid are given to infants, which makes it easy to overmedicate or undermedicate. Parents may lack the necessary expertise to administer the drug at first, but with discharge preparation they should be prepared to administer the drug safely.

The nurse notes premature ventricular contractions (PVCs) while suctioning a patient's endotracheal tube. Which next action by the nurse is indicated? a. Plan to suction the patient more frequently. b. Decrease the suction pressure to 80 mm Hg. c. Give antidysrhythmic medications per protocol. d. Stop and ventilate the patient with 100% oxygen.

ANS: D Dysrhythmias during suctioning may indicate hypoxemia or sympathetic nervous system stimulation. The nurse should stop suctioning and ventilate the patient with 100% O2. There is no indication that more frequent suctioning is needed. Lowering the suction pressure will decrease the effectiveness of suctioning without improving the hypoxemia. Because the PVCs occurred during suctioning, there is no need for antidysrhythmic medications (which may have adverse effects) unless they recur when the suctioning is stopped and patient is well oxygenated. DIF: Cognitive Level: Apply (application)

A patient who is taking a potassium-wasting diuretic for treatment of hypertension complains of generalized weakness. Which action is appropriate for the nurse to take? a. Assess for facial muscle spasms. b. Ask the patient about loose stools. c. Recommend the patient avoid drinking orange juice with meals. d. Suggest that the health care provider order a basic metabolic panel.

ANS: D Generalized weakness is a manifestation of hypokalemia. After the health care provider orders the metabolic panel, the nurse should check the potassium level. Facial muscle spasms might occur with hypocalcemia. Orange juice is high in potassium and would be advisable to drink if the patient is hypokalemic. Loose stools are associated with hyperkalemia.

A patient who is taking a potassium-wasting diuretic for treatment of hypertension complains of generalized weakness. It is most appropriate for the nurse to take which action? a. Assess for facial muscle spasms. b. Ask the patient about loose stools. c. Suggest that the patient avoid orange juice with meals. d. Ask the health care provider to order a basic metabolic panel.

ANS: D Generalized weakness is a manifestation of hypokalemia. After the health care provider orders the metabolic panel, the nurse should check the potassium level. Facial muscle spasms might occur with hypocalcemia. Orange juice is high in potassium and would be advisable to drink if the patient was hypokalemic. Loose stools are associated with hyperkalemia

Esomeprazole (Nexium) is prescribed for a patient who incurred extensive burn injuries 5 days ago. Which nursing assessment would best evaluate the effectiveness of the medication? a. Bowel sounds b. Stool frequency c. Abdominal distention d. Stools for occult blood

ANS: D H2 blockers and proton pump inhibitors are given to prevent Curling's ulcer in the patient who has suffered burn injuries. Proton pump inhibitors usually do not affect bowel sounds, stool frequency, or appetite

A young adult contracts hepatitis from contaminated food. During the acute (icteric) phase of the patient's illness, the nurse would expect serologic testing to reveal a. antibody to hepatitis D (anti-HDV). b. hepatitis B surface antigen (HBsAg). c. anti-hepatitis A virus immunoglobulin G (anti-HAV IgG). d. anti-hepatitis A virus immunoglobulin M (anti-HAV IgM).

ANS: D Hepatitis A is transmitted through the oral-fecal route, and antibody to HAV IgM appears during the acute phase of hepatitis A. The patient would not have antigen for hepatitis B or antibody for hepatitis D. Anti-HAV IgG would indicate past infection and lifelong immunity.

The patient's laboratory report today indicates severe hypokalemia, and the nurse has notified the physician. Nursing assessment indicates that heart rhythm is regular. What is the most important nursing intervention for this patient now? a. Raise bed side rails due to potential decreased level of consciousness and confusion. b. Examine sacral area and patient's heels for skin breakdown due to potential edema. c. Establish seizure precautions due to potential muscle twitching, cramps, and seizures. d. Institute fall precautions due to potential postural hypotension and weak leg muscles.

ANS: D Hypokalemia can cause postural hypotension and bilateral muscle weakness, especially in the lower extremities. Both of these increase the risk of falls. Hypokalemia does not cause edema, decreased level of consciousness, or seizures.

Which laboratory result for a patient with multifocal premature ventricular contractions (PVCs) is most important for the nurse to communicate to the health care provider? a. Blood glucose of 243 mg/dL b. Serum chloride of 92 mEq/L c. Serum sodium of 134 mEq/L d. Serum potassium of 2.9 mEq/L

ANS: D Hypokalemia increases the risk for ventricular dysrhythmias such as PVCs, ventricular tachycardia, and ventricular fibrillation. The health care provider will need to prescribe a potassium infusion to correct this abnormality. Although the other laboratory values also are abnormal, they are not likely to be the etiology of the patient's PVCs and do not require immediate correction

A 62-year-old patient who has Parkinson's disease is taking bromocriptine (Parlodel). Which information obtained by the nurse may indicate a need for a decrease in the dose? a. The patient has a chronic dry cough. b. The patient has four loose stools in a day. c. The patient develops a deep vein thrombosis. d. The patient's blood pressure is 92/52 mm Hg.

ANS: D Hypotension is an adverse effect of bromocriptine, and the nurse should check with the health care provider before giving the medication. Diarrhea, cough, and deep vein thrombosis are not associated with bromocriptine use.

72 hours after cardiac surgery, a young child has a temperature of 37.7 C (101 F). The nurse should: a. Keep the child warm with blankets. b. Apply a hypothermia blanket. c. Record the temperature on nurses' notes. d. Report findings to physician.

ANS: D In the first 24 to 48 hours after surgery, the body temperature may increase to 37.7 C (100 F) as part of the inflammatory response to tissue trauma. If the temperature is higher or an elevated temperature continues after this period, it is most likely a sign of an infection and immediate investigation is indicated. Blankets should be removed from the child to keep the temperature from increasing. A hypothermia blanket is not indicated for this level of temperature. The temperature should be recorded, but the physician must be notified for evaluation.

The nurse should teach a patient that which is a primary prevention strategy to reduce the occurrence of head injuries? a. Blood pressure control b. Smoking cessation c. Maintaining a healthy weight d. Violence prevention

ANS: D Injury prevention measures such as wearing a seat belt, helmet use, firearm safety, and violence prevention programs reduce the risk of traumatic brain injuries. Blood pressure control and exercising can decrease the risk of vascular disease, impacting the cerebral arteries, rather than head injuries. Smoking cessation is one primary prevention strategy which can decrease the risk of vascular disease. Maintaining a healthy weight can decrease the risk of vascular disease.

Which nursing interventions included in the care of a mechanically ventilated patient with acute respiratory failure can the registered nurse (RN) delegate to an experienced licensed practical/vocational nurse (LPN/LVN) working in the intensive care unit? a. Assess breath sounds every hour. b. Monitor central venous pressures. c. Place patient in the prone position. d. Insert an indwelling urinary catheter.

ANS: D Insertion of indwelling urinary catheters is included in LPN/LVN education and scope of practice and can be safely delegated to an LPN/LVN who is experienced in caring for critically ill patients. Placing a patient who is on a ventilator in the prone position requires multiple staff, and should be supervised by an RN. Assessment of breath sounds and obtaining central venous pressures require advanced assessment skills and should be done by the RN caring for a critically ill patient.

A patient complains of leg cramps during hemodialysis. The nurse should a. massage the patient's legs. b. reposition the patient supine. c. give acetaminophen (Tylenol). d. infuse a bolus of normal saline.

ANS: D Muscle cramps during dialysis are caused by rapid removal of sodium and water. Treatment includes infusion of normal saline. The other actions do not address the reason for the cramps. DIF: Cognitive Level: Apply (application) REF: 1091 TOP: Nursing Process: Implementation MSC: NCLEX: Physiological Integrity

A patient with acute pancreatitis is NPO and has a nasogastric (NG) tube to suction. Which information obtained by the nurse indicates that these therapies have been effective? a. Bowel sounds are present. c. Electrolyte levels are normal. b. Grey Turner sign resolves. d. Abdominal pain is decreased.

ANS: D NG suction and NPO status will decrease the release of pancreatic enzymes into the pancreas and decrease pain. Although bowel sounds may be hypotonic with acute pancreatitis, the presence of bowel sounds does not indicate that treatment with NG suction and NPO status has been effective. Electrolyte levels may be abnormal with NG suction and must be replaced by appropriate IV infusion. Although Grey Turner sign will eventually resolve, it would not be appropriate to wait for this to occur to determine whether treatment was effective.

The nurse is reviewing the medication administration record (MAR) on a patient with partial-thickness burns. Which medication is best for the nurse to administer before scheduled wound debridement? a. Ketorolac (Toradol) b. Lorazepam (Ativan) c. Gabapentin (Neurontin) d. Hydromorphone (Dilaudid)

ANS: D Opioid pain medications are the best choice for pain control. The other medications are used as adjuvants to enhance the effects of opioids.

The nurse is caring for a patient who was admitted the previous day with a basilar skull fracture after a motor vehicle crash. Which assessment finding indicates a possible complication that should be reported to the health care provider? a. Complaint of severe headache b. Large contusion behind left ear c. Bilateral periorbital ecchymosis d. Temperature of 101.4° F (38.6° C)

ANS: D Patients who have basilar skull fractures are at risk for meningitis, so the elevated temperature should be reported to the health care provider. The other findings are typical of a patient with a basilar skull fracture.

When the nurse educator is evaluating the skills of a new registered nurse (RN) caring for patients experiencing shock, which action by the new RN indicates a need for more education? a. Placing the pulse oximeter on the ear for a patient with septic shock b. Keeping the head of the bed flat for a patient with hypovolemic shock c. Increasing the nitroprusside (Nipride) infusion rate for a patient with a high SVR d. Maintaining the room temperature at 66° to 68° F for a patient with neurogenic shock

ANS: D Patients with neurogenic shock may have poikilothermia. The room temperature should be kept warm to avoid hypothermia. The other actions by the new RN are appropriate.

Nurses in change-of-shift report are discussing the care of a patient with a stroke who has progressively increasing weakness and decreasing level of consciousness. Which patient problem do they determine has the highest priority for the patient? a. Impaired physical mobility related to weakness b. Disturbed sensory perception related to brain injury c. Risk for impaired skin integrity related to immobility d. Risk for aspiration related to inability to protect airway

ANS: D Protection of the airway is the priority of nursing care for a patient having an acute stroke. The other diagnoses are also appropriate, but interventions to prevent aspiration are the priority at this time

To evaluate the effectiveness of the pantoprazole (Protonix) ordered for a patient with systemic inflammatory response syndrome (SIRS), which assessment will the nurse perform? a. Auscultate bowel sounds. b. Palpate for abdominal pain. c. Ask the patient about nausea. d. Check stools for occult blood.

ANS: D Proton pump inhibitors are given to decrease the risk for stress ulcers in critically ill patients. The other assessments also will be done, but these will not help in determining the effectiveness of the pantoprazole administration.

A patient with a stroke experiences facial drooping on the right side and right-sided arm and leg paralysis. When admitting the patient, which clinical manifestation will the nurse expect to find? a. Impulsive behavior b. Right-sided neglect c. Hyperactive left-sided tendon reflexes d. Difficulty comprehending instructions

ANS: D Right-sided paralysis indicates a left-brain stroke, which will lead to difficulty with comprehension and use of language. The left-side reflexes are likely to be intact. Impulsive behavior and neglect are more likely with a right-side stroke.

The nurse will anticipate teaching a patient with a possible seizure disorder about which test? a. Cerebral angiography b. Evoked potential studies c. Electromyography (EMG) d. Electroencephalography (EEG)

ANS: D Seizure disorders are usually assessed using EEG testing. Evoked potential is used for diagnosing problems with the visual or auditory systems. Cerebral angiography is used to diagnose vascular problems. EMG is used to evaluate electrical innervation to skeletal muscle. DIF: Cognitive Level: Understand (comprehension) REF: 1351 TOP: Nursing Process: Planning MSC: NCLEX: Physiological Integrity

A serum potassium level of 3.2 mEq/L (3.2 mmol/L) is reported for a patient with cirrhosis who has scheduled doses of spironolactone (Aldactone) and furosemide (Lasix) due. Which action should the nurse take? a. Withhold both drugs. c. Administer the furosemide. b. Administer both drugs d. Administer the spironolactone.

ANS: D Spironolactone is a potassium-sparing diuretic and will help increase the patient's potassium level. The nurse does not need to talk with the doctor before giving the spironolactone, although the health care provider should be notified about the low potassium value. The furosemide will further decrease the patient's potassium level and should be held until the nurse talks with the health care provider.

To determine whether there is a delay in impulse conduction through the ventricles, the nurse will measure the duration of the patient's a. P wave. b. Q wave. c. PR interval. d. QRS complex.

ANS: D The QRS complex represents ventricular depolarization. The P wave represents the depolarization of the atria. The PR interval represents depolarization of the atria, atrioventricular node, bundle of His, bundle branches, and the Purkinje fibers. The Q wave is the first negative deflection following the P wave and should be narrow and short.

The nurse obtains a rhythm strip on a patient who has had a myocardial infarction and makes the following analysis: no visible P waves, PR interval not measurable, ventricular rate of 162, R-R interval regular, and QRS complex wide and distorted, and QRS duration of 0.18 second. The nurse interprets the patient's cardiac rhythm as a. atrial flutter. b. sinus tachycardia. c. ventricular fibrillation. d. ventricular tachycardia.

ANS: D The absence of P waves, wide QRS, rate greater than 150 beats/min, and the regularity of the rhythm indicate ventricular tachycardia. Atrial flutter is usually regular, has a narrow QRS configuration, and has flutter waves present representing atrial activity. Sinus tachycardia has P waves. Ventricular fibrillation is irregular and does not have a consistent QRS duration.

Which nursing diagnosis is of highest priority for a patient with Parkinson's disease who is unable to move the facial muscles? a. Activity intolerance b. Self-care deficit: toileting c. Ineffective self-health management d. Imbalanced nutrition: less than body requirements

ANS: D The data about the patient indicate that poor nutrition will be a concern because of decreased swallowing. The other diagnoses may also be appropriate for a patient with Parkinson's disease, but the data do not indicate that they are current problems for this patient.

After the emergency department nurse has received a status report on the following patients who have been admitted with head injuries, which patient should the nurse assess first? a. A 20-yr-old patient whose cranial x-ray shows a linear skull fracture b. A 50-yr-old patient who has an initial Glasgow Coma Scale score of 13 c. A 30-yr-old patient who lost consciousness for a few seconds after a fall d. A 40-yr-old patient whose right pupil is 10 mm and unresponsive to light

ANS: D The dilated and nonresponsive pupil may indicate an intracerebral hemorrhage and increased intracranial pressure. The other patients are not at immediate risk for complications such as herniation. DIF: Cognitive Level: Analyze (analysis)

What is the most appropriate nursing response to the father of a newborn infant with myelomeningocele who asks about the cause of this condition? a. One of the parents carries a defective gene that causes myelomeningocele. b. A deficiency in folic acid in the father is the most likely cause. c. Offspring of parents who have a spinal abnormality are at greater risk for myelomeningocele. d. There may be no definitive cause identified

ANS: D The etiology of most neural tube defects is unknown in most cases. There may be a genetic predisposition or a viral origin, and the disorder has been linked to maternal folic acid deficiency; however, the actual cause has not been determined. There is no evidence that children who have parents with spinal problems are at greater risk for neural tube defects. p1581

The nurse educator is evaluating the care that a new registered nurse (RN) provides to a patient receiving mechanical ventilation. Which action by the new RN indicates the need for more education? a. The RN increases the FIO2 to 100% before suctioning. b. The RN secures a bite block in place using adhesive tape. c. The RN asks for assistance to resecure the endotracheal tube. d. The RN positions the patient with the head of bed at 10 degrees

ANS: D The head of the patient's bed should be positioned at 30 to 45 degrees to prevent ventilator-associated pneumonia. The other actions by the new RN are appropriate. DIF: Cognitive Level: Apply (application)

A patient with acute kidney injury (AKI) has longer QRS intervals on the electrocardiogram (ECG) than were noted on the previous shift. Which action should the nurse take first? a. Notify the patient's health care provider. b. Document the QRS interval measurement. c. Review the chart for the patient's current creatinine level. d. Check the medical record for the most recent potassium level.

ANS: D The increasing QRS interval is suggestive of hyperkalemia, so the nurse should check the most recent potassium and then notify the patient's health care provider. The BUN and creatinine will be elevated in a patient with AKI, but they would not directly affect the electrocardiogram (ECG). Documentation of the QRS interval is also appropriate, but interventions to decrease the potassium level are needed to prevent life-threatening dysrhythmias. DIF: Cognitive Level: Analyze (analysis) REF: 1072 OBJ: Special Questions: Prioritization TOP: Nursing Process: Implementation MSC: NCLEX: Physiological Integrity

Which focused data will the nurse monitor in relation to the 4+ pitting edema assessed in a patient with cirrhosis? a. Hemoglobin b. Temperature c. Activity level d. Albumin level

ANS: D The low oncotic pressure caused by hypoalbuminemia is a major pathophysiologic factor in the development of edema. The other parameters are not directly associated with the patient's edema.

To maintain proper cuff pressure of an endotracheal tube (ET) when the patient is on mechanical ventilation, the nurse should a. inflate the cuff with a minimum of 10 mL of air. b. inflate the cuff until the pilot balloon is firm on palpation. c. inject air into the cuff until a manometer shows 15 mm Hg pressure. d. inject air into the cuff until a slight leak is heard only at peak inflation.

ANS: D The minimal occluding volume technique involves injecting air into the cuff until an air leak is present only at peak inflation. The volume to inflate the cuff varies with the ET and the patient's size. Cuff pressure should be maintained at 20 to 25 mm Hg. An accurate assessment of cuff pressure cannot be obtained by palpating the pilot balloon.

The nurse responds to a ventilator alarm and finds the patient lying in bed gasping and holding the endotracheal tube (ET) in her hand. Which action should the nurse take next? a. Activate the rapid response team. b. Provide reassurance to the patient. c. Call the health care provider to reinsert the tube. d. Manually ventilate the patient with 100% oxygen.

ANS: D The nurse should ensure maximal patient oxygenation by manually ventilating with a bag-valve-mask system. Offering reassurance to the patient, notifying the health care provider about the need to reinsert the tube, and activating the rapid response team are also appropriate after the nurse has stabilized the patient's oxygenation. DIF: Cognitive Level: Analyze (analysis)

A patient who was involved in a motor vehicle crash has had a tracheostomy placed to allow for continued mechanical ventilation. How should the nurse interpret the following arterial blood gas results: pH 7.48, PaO2 85 mm Hg, PaCO2 32 mm Hg, and HCO3 25 mEq/L? a. Metabolic acidosis b. Metabolic alkalosis c. Respiratory acidosis d. Respiratory alkalosis

ANS: D The pH indicates that the patient has alkalosis and the low PaCO2 indicates a respiratory cause. The other responses are incorrect based on the pH and the normal HCO3

The nurse is reviewing the patient's ABG results the PaO2 is 96 pH 7.2 HCO3 25 CO2 is 55. What might the nurse expect to observe on assessment of this patient A. Muscle weakness B. Abdominal pain C. Hyperventilation D. Disorientation

ANS: D The patient is experiencing respiratory acidosis ( pH, and PaCO2 ) which may be manifested by disorientation, tremors, possible seizures, and decreased level of consciousness. Tachycardia and decreased blood pressure are not characteristic of a problem of respiratory acidosis. Increased anxiety and hyperventilation will cause respiratory alkalosis, which is manifested by an increase in pH and a decrease in PaCO2.

A patient develops sinus bradycardia at a rate of 32 beats/min, has a blood pressure (BP) of 80/42 mm Hg, and is complaining of feeling faint. Which action should the nurse take next? a. Recheck the heart rhythm and BP in 5 minutes. b. Have the patient perform the Valsalva maneuver. c. Give the scheduled dose of diltiazem (Cardizem). d. Apply the transcutaneous pacemaker (TCP) pads.

ANS: D The patient is experiencing symptomatic bradycardia and treatment with TCP is appropriate. Continued monitoring of the rhythm and BP is an inadequate response. Calcium channel blockers will further decrease the heart rate and the diltiazem should be held. The Valsalva maneuver will further decrease the rate. DIF: Cognitive Level: Apply (application)

An hour after a thoracotomy, a patient complains of incisional pain at a level 7 (based on 0 to 10 scale) and has decreased left-sided breath sounds. The pleural drainage system has 100 mL of bloody drainage and a large air leak. Which action is best for the nurse to take next? a. Milk the chest tube gently to remove any clots. b. Clamp the chest tube momentarily to check for the origin of the air leak. c. Assist the patient to deep breathe, cough, and use the incentive spirometer. d. Set up the patient controlled analgesia (PCA) and administer the loading dose of morphine.

ANS: D The patient is unlikely to take deep breaths or cough until the pain level is lower. A chest tube output of 100 mL is not unusual in the first hour after thoracotomy and would not require milking of the chest tube. An air leak is expected in the initial postoperative period after thoracotomy.

The nurse is instructing the nursing assistant to prevent pressure ulcers in a frail older patient with spinal cord injury. Which action indicates the nursing assistant has understood the nurse's teaching? a. Bathing and drying the skin vigorously to stimulate circulation b. Keeping the head of the bed elevated 30 degrees c. Limiting intake of fluid and offer frequent snacks d. Turning the patient at least every 2 hours

ANS: D The patient should be turned at least every 2 hours as permanent damage can occur in 2 hours or less. If skin assessment reveals a stage I ulcer while on a 2-hour turning schedule, the patient must be turned more frequently. Limiting fluids will prevent healing; however, offering snacks is indicated to increase healing particularly if they are protein based, because protein plays a role in healing. Use of doughnuts, elevated heads of beds, and overstimulation of skin may all stimulate, if not actually encourage, dermal decline.

After receiving change-of-shift report, which patient should the nurse assess first? a. Patient who is scheduled for the drain phase of a peritoneal dialysis exchange b. Patient with stage 4 chronic kidney disease who has an elevated phosphate level c. Patient with stage 5 chronic kidney disease who has a potassium level of 3.4 mEq/L d. Patient who has just returned from having hemodialysis and has a heart rate of 124/min

ANS: D The patient who is tachycardic after hemodialysis may be bleeding or excessively hypovolemic and should be assessed immediately for these complications. The other patients also need assessments or interventions but are not at risk for life-threatening complications DIF: Cognitive Level: Analyze (analysis) REF: 1091 OBJ: Special Questions: Prioritization | Special Questions: Multiple Patients TOP: Nursing Process: Assessment MSC: NCLEX: Safe and Effective Care Environment

When assessing a patient who has just arrived after an automobile accident, the emergency department nurse notes tachycardia and absent breath sounds over the right lung. For which intervention will the nurse prepare the patient? a. Emergency pericardiocentesis b. Stabilization of the chest wall with tape c. Administration of an inhaled bronchodilator d. Insertion of a chest tube with a chest drainage system

ANS: D The patient's history and absent breath sounds suggest a right-sided pneumothorax or hemothorax, which will require treatment with a chest tube and drainage. The other therapies would be appropriate for an acute asthma attack, flail chest, or cardiac tamponade, but the patient's clinical manifestations are not consistent with these problems

A patient who was found unconscious in a burning house is brought to the emergency department by ambulance. The nurse notes that the patient's skin color is bright red. Which action should the nurse take first? a. Insert two large-bore IV lines. b. Check the patient's orientation. c. Assess for singed nasal hair and dark oral mucous membranes. d. Place the patient on 100% oxygen using a non-rebreather mask.

ANS: D The patient's history and skin color suggest carbon monoxide poisoning, which should be treated by rapidly starting oxygen at 100%. The other actions can be taken after the action to correct gas exchange.

After change-of-shift report, which patient should the nurse assess first? a. 72-year-old with cor pulmonale who has 4+ bilateral edema in his legs and feet b. 28-year-old with a history of a lung transplant and a temperature of 101° F (38.3° C) c. 40-year-old with a pleural effusion who is complaining of severe stabbing chest pain d. 64-year-old with lung cancer and tracheal deviation after subclavian catheter insertion

ANS: D The patient's history and symptoms suggest possible tension pneumothorax, a medical emergency. The other patients also require assessment as soon as possible, but tension pneumothorax will require immediate treatment to avoid death from inadequate cardiac output or hypoxemia.

The nurse performing a focused assessment of left posterior temporal lobe functions will assess the patient for a. sensation on the left side of the body. b. voluntary movements on the right side. c. reasoning and problem-solving abilities. d. understanding written and oral language.

ANS: D The posterior temporal lobe integrates the visual and auditory input for language comprehension. Reasoning and problem solving are functions of the anterior frontal lobe. Sensation on the left side of the body is located in the right postcentral gyrus. Voluntary movement on the right side is controlled in the left precentral gyrus.

A patient with cirrhosis has ascites and 4+ edema of the feet and legs. Which nursing action will be included in the plan of care? a. Restrict daily dietary protein intake. b. Reposition the patient every 4 hours. c. Perform passive range of motion twice daily. d. Place the patient on a pressure-relief mattress.

ANS: D The pressure-relieving mattress will decrease the risk for skin breakdown for this patient. Adequate dietary protein intake is necessary in patients with ascites to improve oncotic pressure. Repositioning the patient every 4 hours will not be adequate to maintain skin integrity. Passive range of motion will not take the pressure off areas such as the sacrum that are vulnerable to breakdown.

Eight hours after a thermal burn covering 50% of a patient's total body surface area (TBSA) the nurse assesses the patient. Which information would be a priority to communicate to the health care provider? a. Blood pressure is 95/48 per arterial line. b. Serous exudate is leaking from the burns. c. Cardiac monitor shows a pulse rate of 108. d. Urine output is 20 mL per hour for the past 2 hours.

ANS: D The urine output should be at least 0.5 to 1.0 mL/kg/hr during the emergent phase, when the patient is at great risk for hypovolemic shock. The nurse should notify the health care provider because a higher IV fluid rate is needed. BP during the emergent phase should be greater than 90 systolic, and the pulse rate should be less than 120. Serous exudate from the burns is expected during the emergent phase

A patient in the emergency department with sudden-onset right-sided weakness is diagnosed with an intracerebral hemorrhage. Which information about the patient is most important to communicate to the health care provider? a. The patient's speech is difficult to understand. b. The patient's blood pressure (BP) is 144/90 mm Hg. c. The patient takes a diuretic because of a history of hypertension. d. The patient has atrial fibrillation and takes warfarin (Coumadin).

ANS: D The use of warfarin probably contributed to the intracerebral bleeding and remains a risk factor for further bleeding. Administration of vitamin K is needed to reverse the effects of the warfarin, especially if the patient is to have surgery to correct the bleeding. The history of hypertension is a risk factor for the patient but has no immediate effect on the patient's care. The BP of 144/90 indicates the need for ongoing monitoring but not for any immediate change in therapy. Slurred speech is consistent with a left-sided stroke, and no change in therapy is indicated

A young adult patient who is in the rehabilitation phase after having deep partial-thickness face and neck burns has a nursing diagnosis of disturbed body image. Which statement by the patient indicates that the problem is resolving? a. "I'm glad the scars are only temporary." b. "I will avoid using a pillow, so my neck will be OK." c. "I bet my boyfriend won't even want to look at me anymore." d. "Do you think dark beige makeup foundation would cover this scar on my cheek?"

ANS: D The willingness to use strategies to enhance appearance is an indication that the disturbed body image is resolving. Expressing feelings about the scars indicates a willingness to discuss appearance, but not resolution of the problem. Because deep partial-thickness burns leave permanent scars, a statement that the scars are temporary indicates denial rather than resolution of the problem. Avoiding using a pillow will help prevent contractures, but it does not address the problem of disturbed body image

The laboratory has just called with the arterial blood gas (ABG) results on four patients. Which result is most important for the nurse to report immediately to the health care provider? a. pH 7.34, PaO2 82 mm Hg, PaCO2 40 mm Hg, and O2 sat 97% b. pH 7.35, PaO2 85 mm Hg, PaCO2 45 mm Hg, and O2 sat 95% c. pH 7.46, PaO2 90 mm Hg, PaCO2 32 mm Hg, and O2 sat 98% d. pH 7.31, PaO2 91 mm Hg, PaCO2 50 mm Hg, and O2 sat 96%

ANS: D These ABGs indicate uncompensated respiratory acidosis and should be reported to the health care provider. The other values are normal or close to normal.

Which assessment finding will the nurse expect for a patient with the following laboratory values: sodium 145 mEq/L, potassium 4.5 mEq/L, calcium 4.5 mg/dL? a. Weak quadriceps muscles b. Decreased deep tendon reflexes c. Light-headedness when standing up d. Tingling of extremities with possible tetany

ANS: D This patient has hypocalcemia because the normal calcium range is 8.4 to 10.5 mg/dL. Hypocalcemia causes muscle tetany, positive Chvostek's sign, and tingling of the extremities. Sodium and potassium values are within their normal ranges: sodium 135 to 145 mEq/L; potassium 3.5 to 5.0 mEq/L. Light-headedness when standing up is a manifestation of ECV deficit or sometimes hypokalemia. Weak quadriceps muscles are associated with potassium imbalances. Decreased deep tendon reflexes are related to hypercalcemia or hypermagnesemia.

After receiving change-of-shift report on a medical unit, which patient should the nurse assess first? a. A patient with cystic fibrosis who has thick, green-colored sputum b. A patient with pneumonia who has crackles bilaterally in the lung bases c. A patient with emphysema who has an oxygen saturation of 90% to 92% d. A patient with septicemia who has intercostal and suprasternal retractions

ANS: D This patient's history of septicemia and labored breathing suggest the onset of ARDS, which will require rapid interventions such as administration of oxygen and use of positive pressure ventilation. The other patients should also be assessed as quickly as possible, but their assessment data are typical of their disease processes and do not suggest deterioration in their status.

Which prescribed intervention will the nurse implement first for a patient in the emergency department who is experiencing continuous tonic-clonic seizures? a. Give phenytoin (Dilantin) 100 mg IV. b. Monitor level of consciousness (LOC). c. Obtain computed tomography (CT) scan. d. Administer lorazepam (Ativan) 4 mg IV.

ANS: D To prevent ongoing seizures, the nurse should administer rapidly acting antiseizure medications such as the benzodiazepines. A CT scan is appropriate, but prevention of any seizure activity during the CT scan is necessary. Phenytoin will also be administered, but it is not rapidly acting. Patients who are experiencing tonic-clonic seizures are nonresponsive, although the nurse should assess LOC after the seizure. DIF: Cognitive Level: Apply (application) REF: 1424 OBJ: Special Questions: Prioritization TOP: Nursing Process: Implementation MSC: NCLEX: Physiological Integrity

The patient is receiving tube feedings due to a jaw surgery. What change in assessment findings should prompt the nurse to request an order for serum sodium concentration? a. Development of ankle or sacral edema b. Increased skin tenting and dry mouth c. Postural hypotension and tachycardia d. Decreased level of consciousness

ANS: D Tube feedings pose a risk for hypernatremia unless adequate water is administered between tube feedings. Hypernatremia causes the level of consciousness to decrease. The serum sodium concentration is a laboratory measure for osmolality imbalances, not ECV imbalances. Edema is a sign of ECV excess, not hypernatremia. Skin tenting, dry mouth, postural hypotension, and tachycardia all can be signs of ECV deficit.

During the emergent phase of burn care, which assessment will be most useful in determining whether the patient is receiving adequate fluid infusion? a. Check skin turgor. b. Monitor daily weight. c. Assess mucous membranes. d. Measure hourly urine output.

ANS: D When fluid intake is adequate, the urine output will be at least 0.5 to 1 mL/kg/hour. The patient's weight is not useful in this situation because of the effects of third spacing and evaporative fluid loss. Mucous membrane assessment and skin turgor also may be used, but they are not as adequate in determining that fluid infusions are maintaining adequate perfusion.

What is an expected assessment finding in a child with coarctation of the aorta? a. Orthostatic hypotension b. Systolic hypertension in the lower extremities c. Blood pressure higher on the left side of the body d. Disparity in blood pressure between the upper and lower extremities

ANS: D Feedback A Orthostatic hypotension is not present with coarctation of the aorta. B Systolic hypertension may be detected in the upper extremities. C The left arm may not accurately reflect systolic hypertension because the left subclavian artery can be involved in the coarctation. D The classic finding in children with coarctation of the aorta is a disparity in pulses and blood pressures between the upper and lower extremities.

Which of the following structural defects constitute tetralogy of Fallot? A. Pulmonary stenosis, ventricular septal defect, overriding aorta, right ventricular hypertrophy B. Aortic stenosis, ventricular septal defect, overriding aorta, right ventricular hypertrophy C. Aortic stenosis, ventricular septal defect, overriding aorta, left ventricular hypertrophy D. Pulmonary stenosis, ventricular septal defect, aortic hypertrophy, left ventricular hypertrophy

ANS:A Pulmonic stenosis, ventricular septal defect, overriding aorta, right ventricular hypertrophy Tetralogy of Fallot has these four characteristics: pulmonary stenosis, ventricular septal defect, overriding aorta, and right ventricular hypertrophy. There is pulmonic stenosis but not aortic stenosis in tetralogy of Fallot. Right ventricular hypertrophy, not left ventricular hypertrophy, is present in tetralogy of Fallot. There is a ventricular septal defect, not an atrial septal defect, and overriding aorta, not aortic hypertrophy, is present.

Which nursing action has the highest priority for a patient who was admitted 16 hours earlier with a C5 spinal cord injury? A. Assessment of respiratory rate and effort B. Application of phennmatic compression devices to legs C. Administration of low doses heparin D. Cardiac monitoring for bradycardia

Assessment of respiratory rate and effort

When a patient comes into the hospital with a diagnosis of metabolic acidosis and a arterial blood pH of 7.30 the nurse will expect which assessment finding of A. Rapid deep respirations B. Skin tenting C. Hot flushed face and neck D. bounding peripheral pulses

B

A male patient who has right-sided weakness after a stroke is making progress in learning to use the left hand for feeding and other activities. The nurse observes that when the patient's wife is visiting, she feeds and dresses him. Which nursing diagnosis is most appropriate for the patient? a. Interrupted family processes related to effects of illness of a family member b. Situational low self-esteem related to increasing dependence on spouse for care c. Disabled family coping related to inadequate understanding by patient's spouse d. Impaired nutrition: less than body requirements related to hemiplegia and aphasia

C

A client with diabetic ketoacidosis is admitted to the intensive care unit and is manifesting respirations that are rapid and deep. Which descriptive term should the nurse use to document the client's breathing pattern? A) Kussmaul respirations. B) Cheyne stokes respirations. C) Apnea. D) Orthopnea.

Kussmaul respirations

The nurse receives the patient's most recent blood work results. Which laboratory value isof greatest concern? a. Sodium of 145 mEq/L b. Calcium of 15.5 mg/dL c. Potassium of 3.5 mEq/L d. Chloride of 100 mEq/L

Normal calcium range is 8.4 to 10.5 mg/dL; therefore, a value of 15.5 mg/dL is abnormally high and of concern. The rest of the laboratory values are within their normal ranges: sodium 136 to 145 mEq/L; potassium 3.5 to 5.0 mEq/L; and chloride 98 to 106 mEq/L.

A client is brought to the emergency department by ambulance, and diabetic ketoacidosis is suspected. Blood samples are taken, and the nurse obtains supplies that will be needed to treat the client. Which type of insulin does the nurse take from the medication supply room for intravenous (IV) administration? A. Nph B. Regular C. Humulin 50/50 D.

Regular

The nurse reviews the arterial blood gas results of an assigned client and notes that the laboratory report indicates a pH of 7.30 (7.30), a Paco2 of 58 mm Hg (58 mm Hg), a Pao2 of 80 mm Hg (80 mm Hg), and an HCO3 of 26 mEq/L (26 mmol/L). The nurse should interpret this to mean that the client has which acid-base disturbance?

respiratory acidosis


Set pelajaran terkait

Ch. 18 (DT w/ Tetracyclines, Sulfonamides, & Urinary Antiseptics)

View Set

SmartBook Assignment Chapter 15: Entropy and Gibbs Energy

View Set

Busn 2190 Unit 2 Chapter 4 Reinforcement Exercises

View Set

Soft Tissue Trauma and Compartment Syndrome

View Set

Business Calculations Mid-Term Exam

View Set